Download as docx, pdf, or txt
Download as docx, pdf, or txt
You are on page 1of 147

Chapter 01: Interprofessional Collaborative Practice: Where We Are Today

MULTIPLE CHOICE
1. A small, rural hospital is part of an Accountable Care Organization (ACO) and is designated
as a Level 1 ACO. What is part of this designation?
a. Bonuses based on achievement of benchmarks
b. Care coordination for chronic diseases
c. Standards for minimum cash reserves
d. Strict requirements for financial reporting
ANS: A
A Level 1 ACO has the least amount of financial risk and requirements, but receives shared
savings bonuses based on achievement of benchmarks for quality measures and
expenditures. Care coordination and minimum cash reserves standards are part of Level 2
ACO requirements. Level 3 ACOs have strict requirements for financial reporting.
2. What was an important finding of the Advisory Board survey of 2014 about primary care
preferences of patients?
a. Associations with area hospitals
b. Costs of ambulatory care
c. Ease of access to care
d. The ratio of providers to patients
ANS: C
As part of the 2014 survey, the Advisory Board learned that patients desired 24/7 access to
care, walk-in settings and the ability to be seen within 30 minutes, and care that is close to
home. Associations with hospitals, costs of care, and the ratio of providers to patients were
not part of these results.
MULTIPLE RESPONSE
1. Which assessments of care providers are performed as part of the value-based purchasing
(VBP) initiative? (Select all that apply.)
a. Appraising costs per case of care for Medicare patients
b. Assessing patients’ satisfaction with hospital care
c. Evaluating available evidence to guide clinical care guidelines
d. Monitoring mortality rates of all patients with pneumonia
e. Requiring advanced IT standards and minimum cash reserves
ANS: A, B, D
Value-based purchasing looks at five domain areas of processes of care, including efficiency
of care (cost per case), experience of care (patient satisfaction measures), and outcomes of
care (mortality rates for certain conditions). Evaluation of evidence to guide clinical care is
part of evidence-based practice. The requirements for IT standards and financial status are
part of Accountable Care Organization standards.
Chapter 02: Translating Research into Clinical Practice
MULTIPLE CHOICE
1. What is the purpose of Level II research?
a. To define characteristics of interest of groups of patients
b. To demonstrate the effectiveness of an intervention or treatment
c. To describe relationships among characteristics or variables
d. To evaluate the nature of relationships between two variables
ANS: C
Level II research is concerned with describing the relationships among characteristics or
variables. Level I research is conducted to define the characteristics of groups of patients.
Level II research evaluates the nature of the relationships between variables. Level IV
research is conducted to demonstrate the effectiveness of interventions or treatments.
2. Which is the most appropriate research design for a Level III research study?
a. Epidemiological studies
b. Experimental design
c. Qualitative studies
d. Randomized clinical trials
ANS: B
The experimental design is the most appropriate design for a Level III study.
Epidemiological studies are appropriate for Level II studies. Qualitative designs are useful
for Level I studies. Randomized clinical trials are used for Level IV studies.
3. What is the purpose of clinical research trials in the spectrum of translational research?
a. Adoption of interventions and clinical practices into routine clinical care
b. Determination of the basis of disease and various treatment options
c. Examination of safety and effectiveness of various interventions
d. Exploration of fundamental mechanisms of biology, disease, or behavior
ANS: C
Clinical research trials are concerned with determining the safety and effectiveness of
interventions. Adoption of interventions and practices is part of clinical implementation.
Determination of the basis of disease and treatment options is part of the preclinical research
phase. Exploration of the fundamental mechanisms of biology, disease, or behavior is part
of the basic research stage.
Chapter 03: Empowering Patients as Collaborative partners: A New Model for
Primary Care
MULTIPLE CHOICE
1. Which statement made by a health care provider demonstrates the most appropriate
understanding for the goal of a performance report?
a. “This process allows me to critique the performance of the rest of the staff.”
b. “Most organizations require staff to undergo a performance evaluation yearly.”
c. “It is hard to be personally criticized but that’s how we learn to change.”
d. “The comments should help me improve my management skills.”
ANS: D
The goal of the performance report is to provide guidance to staff in the areas of
professional development, mentoring, and leadership development. A peer review is written
by others who perform similar skills (peers). The remaining options may be true but do not
provide evidence of understanding of the goal of this professional requirement.
MULTIPLE RESPONSE
1. Which assessment question would a health care provider ask when engaging in the previsit
stage of the new model for primary care? (Select all that apply.)
a. “Are you ready to discuss some of the community resources that are available?”
b. “Are you experiencing any side effects from your newly prescribed medications?”
c. “Do you anticipate any problems with adhering to your treatment plan?”
d. “Are you ready to discuss the results of your laboratory tests?”
e. “Do you have any questions about the lab tests that have been ordered for you?”
ANS: B, C, E
The nursing responsibilities in the previsit stage include assessing the patient’s tolerance of
prescribed medications, understanding of existing treatment plan, and education about
required lab testing. The primary care provider is responsible for screening lab data and
discussing community resources during the actual visit.
Chapter 04: Coordinated Chronic Care
MULTIPLE CHOICE
1. To reduce adverse events associated with care transitions, the Centers for Medicare and
Medicaid Service have implemented which policy?
a. Mandates for communication among primary caregivers and hospitalists
b. Penalties for failure to perform medication reconciliations at time of discharge
c. Reduction of payments for patients readmitted within 30 days after discharge
d. Requirements for written discharge instructions for patients and caregivers
ANS: C
As a component of the Affordable Care Act, the Centers for Medicare and Medicaid Service
developed the Readmissions Reduction Program reducing payments for certain patients
readmitted within 30 days of discharge. The CMS did not mandate communication, institute
penalties for failure to perform medication reconciliations, or require written discharge
instructions.
2. According to multiple research studies, which intervention has resulted in lower costs and
fewer rehospitalizations in high-risk older patients?
a. Coordination of posthospital care by advanced practice health care providers
b. Frequent posthospital clinic visits with a primary care provider
c. Inclusion of extended family members in the outpatient plan of care
d. Telephone follow-up by the pharmacist to assess medication compliance
ANS: A
Research studies provided evidence that high-risk older patients who had posthospital care
coordinated by an APN had reduced rehospitalization rates. It did not include clinic visits
with a primary care provider, inclusion of extended family members in the plan of care, or
telephone follow-up by a pharmacist.
MULTIPLE RESPONSE
1. Which advantages are provided to the chronically ill patient by personal electronic
monitoring devices? (Select all that apply.)
a. Helps provide more patient control their health and lifestyle
b. Eliminates need for regular medical and nursing follow-up visits
c. Helps the early identification of patient health-related problems
d. Helps health care providers in keeping track of the patient’s health status
e. Cost is often covered by Medicare
ANS: A, C, D, E
The explosion in availability of personal electronic monitoring devices is potentially useful
to many patients with chronic disease and others hoping to maintain good health. Data is
recorded and can help people have more control over their health and lifestyle. It can also
help health care providers keep track of their patients’ health status, as information from
these devices can be uploaded into Apps and electronic health records. These devices are
becoming more affordable and some are covered by Medicare. Coupled with telehealth,
e-mail, or other electronic communication with health care providers allows patient
problems to be recognized early. Medical and nursing follow-up is still required as problems
arise.
Chapter 05: An Introduction to Health Care Disparities and Culturally Responsive
Primary Care
MULTIPLE CHOICE
1. A primary care provider administers the “Newest Vital Sign” health literacy test to a patient
newly diagnosed with a chronic disease. What information is gained by administering this
test?
a. Ability to calculate data, along with general knowledge about health
b. Ease of using technology and understanding of graphic data
c. Reading comprehension and reception of oral communication
d. Understanding of and ability to discuss health care concerns
ANS: A
The “Newest Vital Sign” tests asks patients to look at information on an ice cream container
label and answer questions that evaluate ability to calculate caloric data and to grasp general
knowledge about food allergies. It does not test understanding of technology or directly
measure reading comprehension. It does not assess oral communication. The “Ask Me 3”
tool teaches patients to ask three primary questions about their health care and management.
2. What is the main reason for using the REALM-SF instrument to evaluate health literacy?
a. It assesses numeracy skills.
b. It enhances patient–provider communication.
c. It evaluates medical word recognition.
d. It measures technology knowledge.
ANS: C
The Rapid Estimate of Adult Literacy in Medicine–Short Form (REALM-SF) is an easy and
fast tool that measures medical word recognition. It does not evaluate numeracy. The “Ask
Me 3” tool enhances patient–provider communication. This tool does not evaluate
understanding of technology.
3. A female patient who is from the Middle East schedules an appointment in a primary care
office. To provide culturally responsive care, what will the clinic personnel do when
meeting this patient for the first time?
a. Ensure that she is seen by a female provider.
b. Include a male family member in discussions about health care.
c. Inquire about the patient’s beliefs about health and treatment.
d. Research middle eastern cultural beliefs about health care.
ANS: C
It is important not to make assumptions about beliefs and practices associated with health
care and to ask the patient about these. While certain practices are common in some cultural
and ethnic groups, assuming that all members of those groups follow those norms is not
culturally responsive.
Chapter 06: Patient/Family Education and Health Literacy
MULTIPLE CHOICE
1. A primary care provider is providing care for a postsurgical client who recently immigrated
to the United States and speaks English only marginally. What intervention will provide the
most effective means of communicating postdischarge information to the client?
a. Postpone discharge until the client is fully recovered from the surgery.
b. Requesting that a family member who speaks English be present during the
teaching session
c. Providing the necessary information in written form in the client’s native language
d. Requesting the services of a professional interpreter fluent in the client’s native
language
ANS: D
Only approved, professional interpreters experienced in health care interpretation are
appropriate interpreters for patients. Family members or friends should not be used as
interpreters. Use of family members or friends may create misinterpretation or
misunderstanding between the provider and the patient. Family members may not
understand medical terms or may interpret only what they feel is important, or patients
might feel uncomfortable divulging personal information to the person interpreting. Written
information in the client’s native language may be a means of reinforcing instructions but
are not a substitute of person-to-person education. It is neither realistic nor necessary to
postpone discharge for this reason.
MULTIPLE RESPONSE
1. What question asked by the client newly diagnosed with congestive heart failure
demonstrates the effectiveness of previous education concerning the Ask Me 3 health
literacy tool? (Select all that apply.)
a. “Where can I get assistance with the cost of my medications?”
b. “Why is it important for me to take this newly prescribed medication?”
c. “Is it true that high blood pressure isn’t causing my problem?”
d. “Is congestive heart failure curable with appropriate treatment?”
e. “Would watching my intake of salt help me manage this problem?
ANS: B, C, E
While all these questions are appropriate, the Ask Me 3 tool encourages the client to
question what the problem is, what they need to do to manage the problem, and why it is
important to follow the treatment plan. Financial support and curability of the problem is not
directly addressed by this tool.
Chapter 07: Genetic Considerations in Primary Care
MULTIPLE CHOICE
1. A patient expresses concern that she is at risk for breast cancer. To best assess the risk for
this patient, what is the best initial action?
a. Ask if there is a family history of breast cancer.
b. Gather and record a three-generation pedigree.
c. Order a genetic test for the breast cancer gene.
d. Recommend direct-to-consumer genetic testing.
ANS: B
The three-generation pedigree is the best way to evaluate genetic risk. Asking about a
family history is not a systematic risk assessment and does not specify who in the family has
the history or whether there is a pattern. Genetic testing and direct-to-consumer (DTC)
genetic testing are not the initial actions when assessing genetic risk.
2. A patient asks about direct-to-consumer (DTC) genetic testing. What will the provider tell
the patient?
a. It is not useful for identifying genetic diseases.
b. Much of the information does not predict disease risk.
c. The results are shared with the patient’s insurance company.
d. The results must be interpreted by a provider.
ANS: B
DTC testing gives a lot of information, but much of it does not contribute to disease
prediction, since mutations are not necessarily related to specific diseases. The tests are
useful but must be interpreted accurately. The results are confidential and do not have to be
interpreted by a provider.

Chapter 08: Risk Management


MULTIPLE RESPONSE
1. What are some causes for failures or delays in diagnosing patients resulting in malpractice
claims? (Select all that apply.)
a. Failing to recognize a medication complication
b. Failing to request appropriate consultations
c. Improper performance of a treatment
d. Not acting on diagnostic test results
e. Ordering a wrong medication
ANS: B, D
Failing to obtain consultations when indicated or not acting on diagnostic test results can
lead to diagnosis-related failures. Failing to recognize medication complications and
ordering a wrong medication lead to medication prescribing allegations. Improper
performance of a treatment can lead to treatment-related malpractice claims.
Chapter 24: Bites and Stings
MULTIPLE CHOICE
1. A child experiences a snake bite while camping and is seen in the emergency department. The
child’s parents are not able to identify the type of snake. An inspection of the site reveals two
puncture wounds on the child’s arm with no swelling or erythema at the site. The child has
normal vital signs. Which treatment is indicated?
a. Administering antivenom and observing the child for 24 to 48 hours
b. Cleaning the wound, giving tetanus prophylaxis, and observing for 12 hours
c. Performing a type and cross match of the child’s blood
d. Referral to a surgeon for incision and suction of the wound
ANS: B
The child does not have immediate symptoms of envenomation, since there is no swelling or
erythema. Because symptoms may be delayed, and the type of snake is unknown, the child
should be observed in an ED or hospital for 12 hours after providing wound care and tetanus
prophylaxis. Antivenom is not indicated unless envenomation occurs. Type and cross match is
done if envenomation is severe. Incision and suction of the sound is not recommended.
MULTIPLE RESPONSE
1. A patient is seen in the emergency department after experiencing a spider bite. The spider is in
a jar and is less than one inch in size, yellow-brown, and has a violin-shaped marking on its
back. Depending on the patient’s symptoms, which treatments and diagnostic evaluations may
be ordered? (Select all that apply.)
a. Airway management
b. An acute abdominal series
c. Antivenom therapy
d. CBC, BUN, electrolytes, and creatinine
e. Coagulation studies
f. Tetanus prophylaxis
ANS: D, E, F
The spider is a brown recluse. If the patient exhibits systemic symptoms, laboratory workup,
including CBC, BUN, creatinine, electrolytes, and coagulation studies should be performed.
Tetanus prophylaxis is given. Airway management, an acute abdominal series, and antivenom
therapy are used for black widow spider bites.
Chapter 25: Bradycardia and Tachycardia
MULTIPLE CHOICE
1. A patient prescribed a beta blocker medication is in the emergency department with reports of
syncope, shortness of breath, and hypotension. A cardiac monitor reveals a heart rate of 35
beats per minute. Which medication may be used to stabilize this patient?
a. Adenosine
b. Amiodarone
c. Atropine
d. Epinephrine
ANS: D
Epinephrine is indicated if unstable bradycardia is caused by beta blockers. This patient is
symptomatic and unstable and should be treated. Adenosine and amiodarone are used to treat
tachycardia. Atropine is used for some types of bradycardia, but not when induced by beta
blockers.

2. A patient reports heart palpitation but no other symptoms and has no prior history of
cardiovascular disease. The clinic provider performs an electrocardiogram and notes atrial
fibrillation and a heart rate of 120 beats per minute. Which is the initial course of action in
treating this patient?
a. Administer atenolol intravenously.
b. Admit to the hospital for urgent cardioversion.
c. Refer the patient to a cardiologist.
d. Initiate steps to begin anticoagulant therapy.
ANS: C
This patient has no history of serious heart disease and does not have symptoms of chest
pressure, acute MI, or congestive heart failure and may be referred to a cardiologist for
evaluation and treatment but anticoagulant therapy to minimize the risk of clot formation
should be started initially. The 2014 AHA Guidelines for Atrial Fibrillation recommend
shared decision-making in regard to anticoagulation based on relative risk of the patient for
thromboembolic event. Atenolol is given IV for patients who are unstable; the advanced life
support treatment guidelines do not recommend treatment of tachycardia if the patient is
stable. Urgent cardioversion is rarely needed if the heart rate is less than 150 beats per minute
unless there are underlying heart conditions.

3. Which cardiac arrhythmia in an unstable patient requires unsynchronized shocks, or


defibrillation?
a. Atrial fibrillation
b. Atrial flutter
c. Monomorphic ventricular tachycardia
d. Polymorphic ventricular tachycardia
ANS: D
Polymorphic ventricular tachycardia should be treated as ventricular fibrillation with
unsynchronized shocks. The other arrhythmias are treated with synchronized cardioversion.
Chapter 30: Hypotension
1. A young adult patient is being treated for hypertension and is noted to have a resting blood
pressure of 135/88 mm Hg just after finishing a meal. After standing, the patient has a blood
pressure of 115/70 mm Hg. What is the likely cause of this change in blood pressure?
a. A hyperglycemic episode
b. Antihypertensive medications
c. Neurogenic orthostatic hypotension
d. Postprandial hypotension
ANS: B
Medications to treat hypertension may cause orthostatic hypotension. Hypoglycemia may
cause hypotension. Neurogenic orthostatic hypotension is less likely since there is no direct
connection to the neurological system. Postprandial hypotension occurs in elderly patients.

2. An elderly patient who experiences orthostatic hypotension secondary to antihypertensive


medications is noted to have a drop in systolic blood pressure of 25 mm Hg. Which
intervention is important for this patient?
a. Administration of intravenous fluids
b. Close monitoring cardiorespiratory status
c. Initiation of a fall risk protocol
d. Withholding antihypertensive medications
ANS: C
A reduction of systolic blood pressure >20 mm Hg is a risk factor for falls in the elderly, so a
fall risk protocol should be initiated. Unless the patient is dehydrated, IV fluids are not
recommended. Close monitoring of CR status will not prevent falls. Withholding
antihypertensive medications often worsens orthostatic hypotension.

3. An older patient develops orthostatic hypotension secondary to an antihypertensive


medication and asks what measures can be taken to minimize this condition. What will the
provider recommend?
a. Changing from sitting to standing slowly
b. Decreasing the medication dosage
c. Decreasing physical activity
d. Performing the Valsalva maneuver
ANS: A
Changing positions slowly will assist in minimizing the effects of this condition. Decreasing
activity will not help and will have a negative effect on general health. Decreasing or
discontinuation of the medication should not be done without first contracting the prescribing
health care provider. Performing the Valsalva maneuver will increase intrathoracic pressure
and should be avoided.

Chapter 35: Examination of the Skin and Approach to Diagnosis of Skin Disorders
1. A primary care provider is performing a Tzanck test to evaluate possible herpes simplex
lesions. To attain accurate results, the provider will perform what intervention?
a. Blanch the lesions while examining them with a magnifying glass.
b. Gently scrape the lesions with a scalpel onto a slide.
c. Perform a gram stain of exudate from the lesions.
d. Remove the top of the vesicles and obtain fluid from the lesions.
ANS: D
The Tzanck test requires removing the tops from vesicular lesions in order to obtain fresh
fluid from the base of the lesions. Blanching of blue to red lesions under a microscope helps
to evaluate whether blood is in the capillaries of the lesions. Scraping lesions onto a slide is
done to evaluate the presence of hyphae and spores common with candidiasis or fungal
infections. Gram staining is performed to distinguish gram-positive from gram-negative
organisms in suspected bacterial infections.

MULTIPLE RESPONSE
1. When examining a patient’s skin, a practitioner uses dermoscopy for what purpose? (Select all
that apply.)
a. Accentuating changes in color of pathologic lesions by fluorescence
b. Assessing changes in pigmentation throughout various lesions
c. Determining whether lesion borders are regular or irregular
d. Differentiating fluid masses from cystic masses in the epidermis
e. Visualizing skin fissures, hair follicles, and pores in lesions
ANS: B, C, E
Dermoscopy is used to visualize the epidermis and superficial dermis and can reveal changes
in pigmentation throughout lesions, whether borders are regular or irregular, and the various
fissures, follicles, and pores present in lesions. The Wood’s light, or black light, is used to
fluoresce lesions to accentuate changes in color. A direct light source is useful for
differentiating fluid masses from cystic masses.
Chapter 36: Surgical Office Procedures
1. A patient has actinic keratosis and the provider elects to use cryosurgery to remove the
lesions. How will the provider administer this procedure?
a. Applying one or two freeze-thaw cycles to each lesion
b. Applying two or more freeze-thaw cycles to each lesion
c. Applying until the freeze spreads laterally 1 mm from the lesion edges
d. Applying until the freeze spreads laterally 4 mm from the lesion edges
ANS: A
For actinic keratosis, one to two freeze-thaw cycles are usually enough. Two or more
freeze-thaw cycles are generally required for thicker, seborrheic keratosis lesions. The freeze
should spread laterally 3 to 4 mm from the edge of the lesions.

2. A provider is preparing to administer electrocautery to a patient who has several seborrheic


keratoses. The patient tells the provider that he has a pacemaker. Which action is correct?
a. Administer the electrocautery per the usual protocol.
b. Apply electrocautery in short burst at low voltage.
c. Refer the patient to a dermatologist for removal.
d. Suggest another method for removal of the lesions.
ANS: B
Patients with pacemakers or implantable cardioverter-defibrillators may receive electrocautery
if appropriate precautions, such as lower voltage and shorter bursts are taken. It is not
necessary to suggest another method or to refer to a dermatologist.

3. Which type of office surgical procedure warrants sterile technique?


a. Curettage
b. Punch biopsy
c. Scissor excision
d. Shave biopsy
ANS: B
Punch biopsy requires sterile technique. The other procedures require cleaning with alcohol
and clean technique with universal precautions.
Chapter 37: Principles of Dermatologic Therapy
1. When recommending an over-the-counter topical medication to treat a dermatologic
condition, which instruction to the patient is important to enhance absorption of the drug?
a. Apply a thick layer of medication over the affected area.
b. A solution spray preparation will be more effective on hairy areas.
c. Put cool compresses over the affected area after application.
d. Use a lotion or cream instead of an ointment preparation.
ANS: B
Hairy areas are difficult to penetrate, so in these areas, a solution, foam, spray, or gel may
work better. Applying a thicker layer does not increase skin penetration or effectiveness of a
medication. Warm or inflamed skin absorbs medications more readily; cool compresses will
decrease absorption. Lotions and creams are not as readily absorbed as ointments, which have
occlusive properties.

2. A provider is prescribing a topical dermatologic medication for a patient who has open
lesions on a hairy area of the body. Which vehicle type will the provider choose when
prescribing this medication?
a. Cream
b. Gel
c. Ointment
d. Powder
ANS: B
Gels are an excellent vehicle for use on hairy areas of the body. Creams and ointments are not
recommended for hairy areas. Powders should be avoided in open wounds.

3. An adult patient has been diagnosed with atopic dermatitis and seborrheic dermatitis
with lesions on the forehead and along the scalp line. Which is correct when prescribing a
corticosteroid medication to treat this condition?
a. Initiate treatment with 0.1% triamcinolone acetonide.
b. Monitor the patient closely for systemic adverse effects during use.
c. Place an occlusive dressing over the medication after application.
d. Prescribe 0.05% fluocinonide to apply liberally.
ANS: A
Treatment with 0.1% triamcinolone acetonide is appropriate in this case, because it is a class 4
corticosteroid and may be used on the face and is suggested for use for these conditions.
Systemic side effects are rare when topical corticosteroids are used appropriately. Occlusive
dressings increase the risk of adverse effects and are not recommended. 0.05% fluocinonide is
a class III corticosteroid and should not be used on the face.
Chapter 38: Screening for Skin Cancer
1. During a total body skin examination for skin cancer, the provider notes a raised, shiny,
slightly pigmented lesion on the patient’s nose. What will the provider do?
a. Consult with a dermatologist about possible melanoma.
b. Reassure the patient that this is a benign lesion.
c. Refer the patient for possible electrodessication and curettage.
d. Tell the patient this is likely a squamous cell carcinoma.
ANS: C
This lesion is characteristic of basal cell carcinoma, which is treated with electrodessication
and curettage. Melanoma lesions are usually asymmetric lesions with irregular borders,
variable coloration, >6 mm diameter, which are elevated; these should be referred
immediately. All suspicious lesions should be biopsied; until the results are known, the
provider should not reassure the patient that the lesion is benign. Squamous cell carcinoma is
roughened, scaling, and bleeds easily.

2. What is the initial approach when obtaining a biopsy of a potential malignant melanoma
lesion?
a. Excisional biopsy
b. Punch biopsy
c. Shave biopsy
d. Wide excision
ANS: A
A suspected malignant melanoma lesion should be biopsied with excisional biopsy; if
diagnosed, a wide excision should follow. Punch and shave biopsy procedures are appropriate
for diagnostic evaluation of NMSC lesions.
Chapter 39: Adnexal Disease
1. A patient has acne and the provider notes lesions on half of the face, some nodules, and
two scarred areas. Which treatment will be prescribed?
a. Oral clindamycin for 6 to 8 weeks
b. Oral isotretinoin
c. Topical benzoyl peroxide and clindamycin
d. Topical erythromycin

2. A provider is considering an oral contraceptive medication to treat acne in an adolescent


female. Which is an important consideration when prescribing this drug?
a. A progesterone-only contraceptive is most beneficial for treating acne.
b. Combined oral contraceptives are effective for non-inflammatory acne only.
c. Oral contraceptives are effective because of their androgen enhancing effects.
d. Yaz, Ortho Tri-Cyclen, and Estrostep, are approved for acne treatment.
Progesterone-only contraceptives may worsen acne. Combined oral contraceptives
are effective in reducing inflammatory and non-inflammatory acne. Oral contraceptives are
effective because of their antiandrogen effects, since androgen induces sebum production.

3. A female patient is diagnosed with hidradenitis suppurativa and has multiple areas of
swelling, pain, and erythema, along with several abscesses in the right femoral area. When
counseling the patient about this disorder, the practitioner will include which information?
a. Antibiotic therapy is effective in clearing up the lesions.
b. It is often progressive with relapses and permanent scarring.
c. The condition is precipitated by depilatories and deodorants.
d. The lesions are infective, and the disease may be transmitted to others.
ANS: Although lesions may be treated with antibiotics, other medications, and drainage, the
disease is often progressive, with relapses and permanent scarring. Deodorants and depilatories
are not implicated as a cause. The disease is not transmitted to others, although the organisms
may cause other infections in other people.

1. When counseling a patient with rosacea about management of this condition, the
provider may recommend (Select all that apply.)
a. applying a topical steroid.
b. avoiding makeup.
c. avoiding oil-based products.
d. eliminating spicy foods.
e. exposing the skin to sun.
f. using topical antibiotics.

2. Which medications may be used as part of the treatment for a patient with hidradenitis
suppurativa? (Select all that apply.)
a. Chemotherapy
b. Erythromycin
c. Infliximab
d. Isotretinoin
e. Prednisone
Chapter 40: Alopecia
1. A patient is seen in the clinic for patches of hair loss. The provider notes several
well-demarcated patches on the scalp and eyebrows without areas of inflammation and several
hairs within the patch with thinner shafts near the scalp. Based on these findings, which type
of alopecia is most likely?
a. Alopecia areata
b. Anagen effluvium
c. Cicatricial alopecia
d. Telogen effluvium
ANS: A
These findings are characteristic of alopecia areata. Anagen effluvium and telogen effluvium
both result in diffuse hair loss and not discrete patches. Cicatricial alopecia involves
inflammation.
2. A patient diagnosed with alopecia is noted to have scaling on the affected areas of the scalp.
Which confirmatory test(s) will the provider order?
a. Examination of scalp scrapings with potassium hydroxide (KOH)
b. Grasping and pulling on a few dozen hairs
c. Serum iron studies and a complete blood count
d. Venereal Disease Research Laboratory (VDRL) test
ANS: A
Scaling on the scalp is suggestive of tinea capitis. To confirm this, the provider will perform
scalp scraping or test hair samples with KOH preparation to look for hyphae. Grasping and
pulling on hairs is used to identify anagen or telogen hairs by appearance. Serum iron and a
CBC are used if anemia is suspected as a cause. VDRL is performed if syphilis is suspected.
3. A female patient is diagnosed with androgenetic alopecia. Which medication will the primary
health care provider prescribe?
a. Anthralin
b. Cyclosporine
c. Finasteride
d. Minoxidil
ANS: D
Either minoxidil or finasteride are used for androgenetic alopecia, but finasteride is Pregnancy
Category X, so minoxidil is the only medication approved by the FDA for use in women.
Anthralin and cyclosporine are used to treat alopecia areata.
Chapter 41: Animal and Human Bites
1. A young adult has been bitten by a dog resulting in several puncture wounds near the thumb
of one hand but can move all fingers and the bleeding has stopped. What understanding
regarding dog bites should direct the care of this patient?
a. Infection is a likely outcome for a dog bite.
b. Dog bites generally result in serious injury.
c. Neurovascular and destructive soft tissue injuries can occur from such a bite.
d. Oral antibiotics are needed to address the increased risk for the development of
osteomyelitis.
ANS: C
Dog bites account for most of the domestic animal bites that require medical care, yet dog
bites have had the lowest incidence of wound infection (2% to 13%). Even though most dog
bites are relatively minor, severe injuries can occur. These can include crush injuries,
destructive soft tissue injuries, neurovascular injuries, orthopedic injuries, and death.
Osteomyelitis is a risk for human bites.

2. A patient has sustained a human bite on the hand during a fist fight. Which is especially
concerning with this type of bite injury?
a. Possible exposure to rabies virus
b. Potential septic arthritis or osteomyelitis
c. Sepsis from Capnocytophaga canimorsus infection
d. Transmission of human immunodeficiency virus
ANS: B
Clenched-fist injury, or “fight bite,” has a high complication rate from the high penetrating
force with the potential for osteomyelitis, tendinitis, and septic arthritis. Humans do not
transmit rabies unless infected, which is highly unlikely. Humans do not transmit C.
canimorsus. HIV transmission is potential, but the risk is extremely low.

MULTIPLE RESPONSE
1. Which type of bite is generally closed by delayed primary closure? (Select all that apply.)
a. Bites to the face
b. Bites to the hand
c. Deep puncture wounds
d. Dog bites on an arm
e. Wounds 6 hours old or older
ANS: B, C, E
Cat and human bites, deep puncture wounds, clinically infected wounds, wounds more than 6
to 12 hours old, and bites to the hand should be left open and closed by delayed primary
closure. A bite to the face is closed by primary closure. Dog bites do not require delayed or
secondary closure.
Chapter 42: Benign Skin Lesions
1. During a total body skin examination for skin cancer, the provider notes a raised, shiny,
slightly pigmented lesion on the patient’s nose. What will the provider do?
a. Consult with a dermatologist about possible melanoma.
b. Reassure the patient that this is a benign lesion.
c. Refer the patient for possible electrodessication and curettage.
d. Tell the patient this is likely a squamous cell carcinoma.
ANS: C
This lesion is characteristic of basal cell carcinoma, which is treated with electrodessication
and curettage. Melanoma lesions are usually asymmetric lesions with irregular borders,
variable coloration, >6 mm diameter, which are elevated; these should be referred
immediately. All suspicious lesions should be biopsied; until the results are known, the
provider should not reassure the patient that the lesion is benign. Squamous cell carcinoma is
roughened, scaling, and bleeds easily.

2. What is the initial approach when obtaining a biopsy of a potential malignant melanoma
lesion?
a. Excisional biopsy
b. Punch biopsy
c. Shave biopsy
d. Wide excision
ANS: A
A suspected malignant melanoma lesion should be biopsied with excisional biopsy; if
diagnosed, a wide excision should follow. Punch and shave biopsy procedures are appropriate
for diagnostic evaluation of NMSC lesions.
Chapter 44: Burns (Minor)
1. A patient comes to the clinic after being splashed with boiling water while cooking. The
patient has partial thickness burns on both forearms, the neck, and the chin. What will the
provider do?
a. Clean and dress the burn wounds.
b. Order a CBC, glucose, and electrolytes.
c. Perform a chest radiograph.
d. Refer the patient to the emergency department (ED).
ANS: D
Patients with burns on the face, potential circumferential burns, and any patient at risk of
airway compromise should be referred to the ED for evaluation and treatment. The provider
should do this urgently and not clean and dress the wounds or order diagnostic tests.

2. A patient sustains chemical burns on both arms after a spill at work. What is the initial action
by the health care providers in the emergency department (ED)?
a. Begin aggressive irrigation of the site.
b. Contact the poison control center.
c. Remove the offending chemical and garments.
d. Request the Material Safety Data information.
ANS: C
The initial response to a chemical burn is to remove the patient’s clothing and the offending
chemical. Aggressive irrigation is usually recommended next, but providers should first
determine the source to make sure that it is safe to use water. Contacting Poison Control and
getting MSDS information are useful measures after the clothing and chemical is removed.

Chapter 45: Cutaneous Adverse Drug Reactions


1. A patient is taking a sulfonamide antibiotic and develops a rash that begins peeling. Which
type of rash is suspected?
a. Erythema multiforme
b. Stevens-Johnson
c. Urticaria
d. Wheal and flare
ANS: B
The Stevens-Johnson syndrome rash typically peels in sheets. Erythema multiforme, urticaria,
and wheal and flare rashes do not peel.

MULTIPLE RESPONSE
1. Which medication classifications are associated with increasing the risk of developing acute
generalized exanthematous pustulosis (AGEP) (Select all that apply.)
a. Cephalosporins
b. Calcium channel blockers
c. Aminopenicillins
d. Tuberculostatic agents
e. Non-steroidal anti-inflammatory drugs (NSAIDS)
ANS: B, C, E
AGEP is triggered by calcium channel blockers, aminopenicillins, an NSAIDS.
Exanthematous drug eruptions are associated with cephalosporins, and tuberculostatic agents.
Chapter 46: Eczematous Dermatitis
1. Which is the primary symptom causing discomfort in patients with atopic dermatitis (AD)?
a. Dryness
b. Erythema
c. Lichenification
d. Pruritis
ANS: D
Itching is incessant, and patients usually develop other signs at the site of itching. None of the
other options are associated with AD.

2. A patient diagnosed with atopic dermatitis asks what can be done to minimize the recurrence
of symptoms. What will the provider recommend?
a. Calcineurin inhibitors
b. Lubricants and emollients
c. Oral diphenhydramine
d. Prophylactic topical steroids
ANS: B
Emollients and lubricants are used long-term to reduce flare-ups. Calcineurin inhibitors can be
helpful for managing chronic moderate to severe eczema. Oral diphenhydramine helps with
symptoms of itching but is not used to prevent symptoms. Corticosteroids should be used
sparingly to treat symptoms and stopped once the inflammation has subsided.

3. A patient who has atopic dermatitis has recurrent secondary bacterial skin infections. What
will the provider recommend to help prevent these infections?
a. Bleach baths twice weekly
b. Frequent bathing with soap and water
c. Low-dose oral antibiotics
d. Topical antibiotic ointments
ANS: A
Bleach baths and intranasal mupirocin have been shown to reduce bacterial superinfections of
the skin. Frequent bathing with soap and water may increase flare-ups and increase the risk
for superinfections. Oral and topical antibiotic prophylaxes are not recommended.
Chapter 47: Infections and Infestations
1. A previously healthy patient has an area of inflammation on one leg which has
well-demarcated borders and the presence of lymphangitic streaking. Based on these
symptoms, what is the initial treatment for this infection?
a. Amoxicillin-clavulanate
b. Clindamycin
c. Doxycycline
d. Sulfamethoxazole-trimethoprim
ANS: A
This patient has symptoms consistent with erysipelas, which is commonly caused by
staphylococcal or streptococcal bacteria. These may be treated empirically with
penicillinase-resistant penicillin if not allergic. Clindamycin, doxycycline, and
sulfamethoxazole-trimethoprim are used for methicillin-resistant staphylococcus aureus
infections.

2. A patient has vesiculopustular lesions around the nose and mouth with areas of honey-colored
crusts. The provider notes a few similar lesions on the patient’s hands and legs. Which
treatment is appropriate for this patient?
a. Mupirocin, 2% ointment
b. Culture and sensitivity of the lesions
c. Sulfamethoxazole-trimethoprim
d. Surgical referral
ANS: A
This patient has symptoms of impetigo which has spread to the hands and legs. Mupirocin,
2% ointment, should be applied three times a day for 10 days. It is not necessary to obtain a
culture since this can be treated empirically in most cases. MRSA is unlikely, so
sulfamethoxazole-trimethoprim is not indicated. Surgical referrals are generally not indicated.

3. A patient with a purulent skin and soft tissue infection (SSTI). A history reveals a previous
MRSA infection in a family member. The clinician performs an incision and drainage of the
lesion and sends a sample to the lab for culture. What is the next step in treating this patient?
a. Apply moist heat until symptoms resolve.
b. Begin treatment with amoxicillin-clavulanate.
c. Prescribe trimethoprim-sulfamethoxazole.
d. Wait for culture results before ordering an antibiotic.
ANS: C
Because of a history of exposure to MRSA, the patient is likely to be colonized and should be
treated accordingly. Small lesions may be treated with moist heat, but the likelihood of MRSA
requires treatment. Amoxicillin-clavulanate is not effective for MRSA. Treatment should be
started empirically.

4. A patient who has never had an outbreak of oral lesions reports a burning sensation on the oral
mucosa and then develops multiple painful round vesicles at the site. A Tzanck culture
confirms HSV-1 infection. What will the provider tell the patient about this condition?
a. Antiviral medications are curative for oral herpes.
b. The initial episode is usually the most severe.
c. There are no specific triggers for this type of herpesvirus.
d. Transmission to others occurs only when lesions are present.
ANS: B
In herpesvirus outbreaks, the initial episode is generally the most severe. Antiviral
medications may prevent outbreaks, but do not cure the disease. HSV-1 has several specific
triggers. Transmission to others may occur even when lesions are not present.

5. A patient who has had lesions for several days is diagnosed with primary herpes labialis and
asks about using a topical medication. What will the provider tell this patient?
a. Oral antivirals are necessary to treat this type of herpes.
b. Preparations containing salicylic acid are most helpful.
c. Topical medications can have an impact on pain and discomfort.
d. Topical medications will significantly shorten the healing time.
Topical medications may alleviate discomfort, but do not shorten healing time. Oral antivirals
may help shorten healing, but are not necessary as treatment, since the disease is usually
self-limiting. Salicylic acid should not be used because it can erode the skin.

6. A patient who has recurrent, frequent genital herpes outbreaks asks about therapy to minimize
the episodes. What will the provider recommend as first-line treatment?
a. Acyclovir
b. Famciclovir
c. Topical medications
d. Valacyclovir
All three oral antiviral medications help reduce the number of occurrences and the frequency
of asymptomatic shedding. Famciclovir and valacyclovir are more costly and no more
effective, so should not be first-line therapy. Topical medications are not useful with
recurrent, frequent genital herpes.

7. When evaluating scalp lesions in a patient suspected of having tinea capitis, the provider uses
a Wood’s lamp and is unable to elicit fluorescence. What is the significance of this finding?
a. The patient does not have tinea capitis.
b. The patient is less likely to have tinea capitis.
c. The patient is positive for tinea capitis.
d. The patient may have tinea capitis.

8. Although some fungal species causing tinea capitis are fluorescent with a Wood’s lamp,
Trichophyton tonsurans, the most common cause or tinea capitis, does not, so lack of
fluorescence does not rule out the infection, make it less likely, or diagnose it. Which
medication will the provider prescribe as first-line therapy to treat tinea capitis?
a. Oral griseofulvin
b. Oral ketoconazole
c. Topical clotrimazole
d. Topical tolnaftate
A Systemic antifungal medications are used for widespread tinea and always with infections that
involve the nails or scalp. Oral ketoconazole should be avoided due to risks of hepatotoxicity
and serious drug interactions.

9. A patient has a pruritic eczematous dermatitis which has been present for 1 week and reports
similar symptoms in other family members. What will the practitioner look for to help
determine a diagnosis of scabies?
a. Bullous lesions on the soles of the feet and palms of the hands
b. Intraepidermal burrows on the interdigital spaces of the hands
c. Nits and small bugs along the scalp line at the back of the neck
d. Pustular lesions in clusters on the trunk and extremities
ANS: B
The scabies mite typically burrows no deeper than the stratus corneum and burrows may be
found in the interdigital spaces of the hands, among other places. Bullous lesions may occur
with impetigo. Nits and small bugs are characteristic findings with pediculosis. Pustular
lesions represent superficial skin infections.

10. The provider is prescribing 5% permethrin cream for an adolescent patient who has scabies.
What will the provider include in education for this patient?
a. All household contacts will be treated only if symptomatic.
b. Itching 2 weeks after treatment indicates treatment failure.
c. Stuffed animals and pillows should be placed in plastic bags for 1 week.
d. The adolescent’s school friends should be treated.
ANS: C
Bedding and clothing of persons with scabies should be washed in hot water and dried on hot
dryer settings. Items that cannot be washed should be put in plastic bags for 1 week. All
household contacts should be treated. Itching may persist because of the secondary dermatitis
for up to 2 weeks and does not represent treatment failure. Casual contacts do not require
treatment.

11. A patient with intertrigo shows no improvement and persistent redness after treatment with
drying agents and antifungal medications. The patient reports an onset of odor associated with
a low-grade fever. What will the provider do next to manage this condition?
a. Culture the lesions to determine the cause.
b. Evaluate the patient for HIV infection.
c. Order topical nystatin cream.
d. Prescribe a cephalosporin antibiotic.
ANS: A
This patient has symptoms of a secondary bacterial infection. The lesions should be cultured
and the results used to determine the appropriate antibiotic. Patients with recurrent candida
infections should be evaluated for underlying HIV infection, diabetes, and other
immunocompromised states. Topical nystatin cream is used for candida infection and these
symptoms are consistent with bacterial infection. Antibiotics should be chosen based on
culture results.

12. An older patient experiences a herpes zoster outbreak and asks the provider if she is
contagious because she is going to be around her grandchild who is too young to be
immunized for varicella. What will the provider tell her?
a. An antiviral medication will prevent transmission to others.
b. As long as her lesions are covered, there is no risk of transmission.
c. Contagion is possible until all her lesions are crusted.
d. Varicella-zoster and herpes zoster are different infections.
ANS: C
Herpes zoster lesions contain high concentrations of virus that can be spread by contact and
by air; although they are less contagious than primary infections, contagion is possible until
all lesions are crusted. Antiviral medications shorten the course, but do not reduce
transmission. Covering the lesions does not prevent transmission. Herpes zoster and
varicella-zoster are the same.

13. A patient has a unilateral vesicular eruption which is described as burning and stabbing in
intensity. To differentiate between herpes simplex and herpes zoster, which test will the
provider order?
a. Polymerase chain reaction analysis
b. Serum immunoglobulins
c. Tzanck test
d. Viral culture
ANS: A
The PCR is a rapid and sensitive test that can differentiate between the two. Serum Ig levels
are not diagnostic. The Tzanck test identifies the presence of a herpes virus but does not
differentiate between the two types. Viral culture will differentiate, but it is not rapid.

1. What instructions will the primary care provider give to parents of a child who has scabies
who is ordered to use 5% permethrin cream? (Select all that apply.)
a. Apply the cream at bedtime and rinse it off in the morning.
b. It is not necessary to wash bedding or clothing when using this cream.
c. Massage the cream into the skin from head to toe.
d. The rash should disappear within a day or two after using the cream.
e. Use once now and repeat the treatment in 1 to 2 weeks.
ANS: A, E
Permethrin cream should be applied from the neck down in children and rinsed off in 8 to 12
hours. The treatment should be done once and then repeated in 1 to 2 weeks. Bedding and
clothing should be washed thoroughly. Adults should apply from head to toe, since the scabies
can infest the hairline of adults. The rash may still be present for several weeks after
treatment.

2. When recommending ongoing treatment for a patient who has recurrent intertrigo, what will
the provider suggest? (Select all that apply.)
a. Aluminum sulfate solution
b. Burrow’s solution compresses
c. Cornstarch application
d. Nystatin cream
e. Topical steroid cream
ANS: A, B
Aluminum sulfate solution and other drying agents are recommended, and Burrow’s solution
compresses may be soothing. Cornstarch is ineffective and may result in fungal growth.
Nystatin cream is used only for candida intertrigo. Topical steroids may promote infection.
Chapter 48: Nail Disorders
1. A patient is diagnosed with herpetic whitlow and in a 2 weeks follow-up evaluation, is noted
to have paronychial inflammation of the tendon sheath in one finger that has responded to
treatment. What is a priority treatment for this patient?
d. Refer the patient to the emergency department.

When paronychial infection of the tendon sheath is suspected in patients with herpetic
whitlow, they should be immediately referred to the emergency department for a surgical
referral. Oral antiviral medications are given for severe cases and recurrences, but the
emergent situation is a priority. Incision and drainage may lead to superinfection of longer
healing. Creatinine clearance is ordered when beginning oral antiviral therapy.

MULTIPLE RESPONSE
1. A patient diagnosed with recurrent herpetic whitlow is counseled about management of
symptoms and prevention of complications. What will be included in this teaching?
b. Contact the provider if symptoms persist longer than 3 weeks.
c. Cool compresses may help with comfort and decrease erythema.
d. Keep hands away from the mouth and eyes to prevent inoculation.

Patients with herpetic whitlow should be seen by a physician if symptoms are recalcitrant to
treatment after 3 weeks. Cool compresses may help with symptomatic relief. Patients should
avoid touching the mouth and eyes to prevent spread of lesions to these tissues. Antiviral
medications should be given within 48 hours of onset of symptoms to be effective. Wearing
gloves during food preparation is not necessary.

2. A female patient who works with caustic chemicals has developed acute paronychia. What
will the provider include when teaching this patient about her condition?
a. Analgesics may be necessary for comfort.
d. Use protective gloves while working.
e. Wear waterproof gloves when washing dishes.

Patients with paronychia may require analgesics for comfort. They should be instructed to
wear protective gloves while working, if the condition is work-related and to wear waterproof
gloves while washing dishes. Nail polish should be avoided, and nails should be kept trimmed
and clean.
Chapter 49: Maculopapular Skin Disorders
1. A patient with chronic seborrheic dermatitis reports having difficulty remembering to use the
twice daily ketoconazole cream prescribed by the provider. What will the provider order for
this patient?
c. Oral itraconazole (Sporanox)

Itraconazole is effective for moderate to severe symptoms and is an alternative for those who
do not wish to use topical treatment. Burrow’s solution and selenium shampoo rinses are not
indicated. Oral corticosteroids are usually not given.

2. A child has plaques on the extensor surfaces of both elbows and on the face with minimal
scaling and pruritis. What is the likely cause of these lesions?
c. Psoriasis

Children with psoriasis often have lesions on the face and have less scaling than adults.
Psoriasis tends to present on extensor surfaces, while atopic dermatitis occurs on flexor
surfaces. Guttate psoriasis appears as teardrop-shaped lesions that appear on the trunk and
spread to the extremities and are occasionally seen after streptococcal infections in
adolescents. Seborrhea usually occurs on the scalp.

3. A patient diagnosed with psoriasis develops lesions on the intertriginous areas of the skin.
Which treatment is recommended?
d. Topical, low-potency steroids

Patients with intertriginous psoriasis should be treated with low-potency topical steroids.
High-potency steroids usually produce maximum benefit in 2 to 3 weeks and research
suggests combining high-potency steroids with vitamin D analog is best. Oral corticosteroids
are used for recalcitrant symptoms.

4. A patient with severe, recalcitrant psoriasis has tried topical medications, intralesional steroid
injections, and phototherapy with ultraviolet B light without consistent improvement in
symptoms. What is the next step in treating this patient?
c. Methotrexate

Methotrexate has shown good efficacy in treating recalcitrant psoriasis. Cyclosporine and oral
retinoids are effective but have serious side effects. Etanercept and other biologic agents are
effective but expensive and should be tried after all other treatments have failed.

1. An adult patient has greasy, scaling patches on the forehead and eyebrows suggestive of
seborrheic dermatitis. What is included in assessment and management of this condition?
a. Begin first-line treatment with a topical antifungal medication.
b. Evaluate the scalp for dry, flaky scales and treat with selenium sulfide shampoo.
First-line therapy may include topical antifungals or corticosteroids. Adults with symptoms on
the face or eyebrows are likely to have scalp lesions, since this is usually a “top-down”
disorder. The condition is chronic and recurrent. Antibacterial medications are used for
secondary bacterial infections but do not treat Malassezia, which is a fungus. Topical steroids
should be used on a short-term basis.
Chapter 50: Pigmentation Changes
1. A parent reports the appearance of areas of depigmented skin on a child which has spread
rapidly. The provider notes asymmetrically patterned tri-colored, macules in a dermatomal
distribution. What type of vitiligo does the provider suspect?
b. Segmented vitiligo

Segmented, or dermatomal vitiligo, spreads rapidly, is usually asymmetrical, and tends to


occur in children. Inflammatory vitiligo occurs after inflammation of the skin. Type A vitiligo
is non-dermatomal and is generally symmetric. Poliosis occurs when well-defined areas of
white hair occur.

2. A patient who is diagnosed with vitiligo asks the provider what can be done to minimize the
contrast between depigmented and normal skin. What will the provider recommend?
a. Applying a cosmetic cover-up or tanning cream

Cosmetic cover-ups or tanning creams are useful to help darken affected areas. Hydrogen
peroxide is not recommended. Tanning is contraindicated; excessive sunburn can stimulate
depigmentation. Waiting for widespread depigmentation is unpredictable.

3. A patient diagnosed with well-localized vitiligo is referred to a dermatologist for treatment.


What will the initial treatment be?
d. Twice-daily application of a mid-potency steroid cream

The initial treatment for vitiligo is twice-daily mid-potency steroids. UVA and UVB therapy
with psoralens may be used if this isn’t effective and must be performed by a qualified
specialist. Patients with widespread areas of vitiligo may be treated with depigmentation
therapy.
Chapter 51: Wound Management
1. A patient has a pressure ulcer that has been treated with topical medications. During a
follow-up visit, the provider notes an area of red bumps in the lesion. What does this indicate?
a. Healing tissue

Wounds that are healing or have the potential to heel will demonstrate pink or red tissue and
the absence of exudate, infection, or debris and will have bumpy granulation tissue. Perfusion
is assessed by pulse assessment and localized capillary refill. Secondary infection is
characterized by exudate and cellular debris. Tunneling is a secondary wound.

2. A patient has an ulcer on one lower leg just above the medial malleolus. The provider notes
irregular wound edges with granulation tissue and moderate exudate, with ankle edema in that
leg. What is the initial treatment to help treat this wound?
a. Compression therapy

This patient has symptoms consistent with venous ulcers, which are characterized by irregular
borders and granulation tissue. Compression therapy is the initial treatment of choice to
reduce edema and promote venous return. Hyperbaric oxygen therapy, revascularization
procedures, and skin grafting are generally used to treat arterial ulcers.

3. A patient with a wound containing necrotic tissue requires debridement. The practitioner notes
an area of erythema and exudate in the wound. Which type of debridement will most likely be
used?
d. Mechanical debridement

Mechanical debridement may be performed using a syringe with an 18-gauge needle to


remove hyperkeratotic or necrotic tissue. Autolytic and chemical debridement methods
require dressings that retain moisture and are contraindicated in the presence of infection.
Biologic debridement uses maggots and is not widely used in the United States.
Chapter 52: Evaluation of the Eyes

1. A provider performs an eye examination during a health maintenance visit and notes a
difference of 0.5 mm in size between the patient’s pupils. What does this finding indicate?
d. Probable benign, physiologic anisocoria

A difference in diameter of less than 1 mm is usually benign. Afferent pupillary defects are
paradoxical dilations of pupils in response to light. This does not indicate differences in
intraocular pressure. A difference of more than 1 mm is more likely to represent an underlying
neurological abnormality.

2. A patient comes to clinic with diffuse erythema in one eye without pain or history of trauma.
The examination reveals a deep red, confluent hemorrhage in the conjunctiva of that eye.
What is the most likely treatment for this condition?
c. Reassure the patient that this will resolve.

Most subconjunctival hemorrhage, occurring with trauma or Valsalva maneuvers, will


self-resolve and are benign. Lubricating drops are used for chemosis. Antibiotic eye drops are
not indicated. Referral is not indicated.

3. During an eye examination, the provider notes a red-light reflex in one eye but not the other.
What is the significance of this finding?
b. Ocular disease requiring referral

The red reflex should be elicited in normal eyes. Any asymmetry or opacity suggests ocular
disease, potentially retinoblastoma, and should be evaluated immediately.

Chapter 53: Cataracts


1. A primary care provider may suspect cataract formation in a patient with which finding?
a. Asymmetric red reflex

An asymmetric red reflex may be a finding in a patient with cataracts. Corneal opacification,
excessive tearing, and corneal injection are not symptoms of cataracts.

MULTIPLE RESPONSE
1. Which are risk factors for development of cataracts? (Select all that apply.)
a. Advancing age
d. Smoking
e. Ultraviolet light

Most older adults will develop cataracts. Smoking and UV light exposure hasten the
development of cataracts. Cholesterol and conjunctivitis are not risk factors.
Chapter 54: Blepharitis, Hordeolum, and Chalazion

1. A patient has a gradually enlarging nodule on one upper eyelid and reports that the lesion is
painful. On examination, the lesion appears warm and erythematous. The provider knows that
this is likely to be which type of lesion?
c. Hordeolum

Although hordeolum and chalazion lesions both present as gradually enlarging nodules, a
hordeolum is usually painful, while a chalazion generally is not. Blepharitis refers to
generalized inflammation of the eyelids. Meibomian is a type of gland near the eye.

2. A patient reports using artificial tears for comfort because of burning and itching in both eyes
but reports worsening symptoms. The provider notes redness and discharge along the eyelid
margins with clear conjunctivae. What is the recommended treatment?
b. Warm compresses, lid scrubs, and antibiotic ointment

This patient has symptoms of blepharitis without conjunctivitis. Initial treatment involves lid
hygiene and antibiotic ointment may be applied after lid scrubs. Antibiotic solution is used if
conjunctivitis is present. Oral antibiotics are used for severe cases. This disorder is generally
chronic.

3. A child has a localized nodule on one eyelid which is warm, tender, and erythematous. On
examination, the provider notes clear conjunctivae and no discharge. What is the
recommended treatment?
d. Warm compresses and massage of the lesion

This child has a hordeolum, which is generally self-limited and usually spontaneously
improves with conservative treatment. Warm compresses and massage of the lesion are
recommended. Referral is not necessary unless a secondary infection occurs. Surgical
intervention is not indicated. Systemic antibiotics are used to treat secondary cellulitis.
Chapter 55: Conjunctivitis
1. A patient reports bilateral burning and itching eyes for several days. The provider notes a
boggy appearance to the conjunctivae, along with clear, watery discharge. The patient’s
eyelids are thickened and discolored. There are no other symptoms. Which type of
conjunctivitis is most likely?
a. Allergic

Allergic conjunctivitis generally presents simultaneously in both eyes with itching as a


predominant feature. Discharge is generally clear or stringy and white and the patient will
have lid discoloration, thickening, and erythema. Bacterial conjunctivitis is characterized by
acute inflammation of the conjunctivae along with purulent discharge. Chemical conjunctivitis
will not have purulent discharge. Viral conjunctivitis is usually in association with a URI.

2. A patient who has symptoms of a cold develops conjunctivitis. The provider notes erythema
of one eye with profuse, watery discharge and enlarged anterior cervical lymph nodes, along
with a fever. Which treatment is indicated?
b. Artificial tears and cool compresses

Viral conjunctivitis accompanies upper respiratory tract infections and is generally


self-limited, lasting 5 to 14 days. Symptomatic treatment is recommended.
Antihistamine-vasoconstrictor drops are used for allergic conjunctivitis. Topical antibiotic
drops are sometimes used for bacterial conjunctivitis. Topical corticosteroid drops are used for
severe inflammation.

3. A patient diagnosed with allergic conjunctivitis and prescribed a topical


antihistamine-vasoconstrictor medication reports worsening symptoms. What is the provider’s
next step in managing this patient’s symptoms?
b. Determine the duration of treatment with this medication.

Antibiotic-vasoconstrictor agents can have a rebound effect with worsening symptoms if used
longer than 3 to 7 days, so the provider should determine whether this is the cause. Topical
mast cell stabilizers are useful as prophylaxis for recurrent or persistent allergic conjunctivitis
and results do not occur for several weeks. Oral antihistamines may be the next step if it is
determined that the cause of worsening symptoms is related to the allergy. It is not necessary
to refer to ophthalmology at this time.
Chapter 56: Corneal Surface Defects and Ocular Surface Foreign Bodies
1. A patient who works in a furniture manufacturing shop reports a sudden onset of severe eye
pain while sanding a piece of wood and now has copious tearing, redness, and light sensitivity
in the affected eye. On examination, the conjunctiva appears injected, but no foreign body is
visualized. What is the practitioner’s next step?
b. Application of topical fluorescein dye

The practitioner must determine if there is a corneal abrasion and will instill fluorescein dye in
order to examine the cornea under a Wood’s lamp. Antibiotic eye drops are not indicated as
initial treatment. Cycloplegic drops are used occasionally for pain control but should be used
with caution. Irrigation of the eye is indicated for chemical burns.

1. Which patients should be referred immediately to an ophthalmologist after eye injury and
initial treatment? (Select all that apply.)
a. A patient who was sprayed by lawn chemicals
b. A patient who works in a metal fabrication shop
e. A patient with irritation secondary to wood dust

Patients with chemical eye injuries, any with possible metallic foreign bodies, and those with
full-thickness corneal lacerations must have immediate referral. Corneal abrasions and
irritation from wood dust may be managed by primary care providers.

Chapter 57: Dry Eye Syndrome


1. A patient experiencing chronically dry eyes reports having a foreign body sensation, burning,
and itching. A Schirmer test is abnormal. What is the suspected cause of this patient’s
symptoms based on this test finding?
a. Aqueous deficiency

An abnormal Schirmer test, which assesses aqueous production, indicates aqueous-deficient


dry eye. A corneal abrasion usually causes excessive tearing. An evaporative disorder is
determined by an evaluation of tear breakup time. Poor eyelid closure causes increased
corneal exposure and increased evaporation of tears.
MULTIPLE RESPONSE
1. A patient has evaporative dry eye syndrome with eyelid inflammation. What are some
pharmacologic and nonpharmacologic measures the provider can recommend? (Select all that
apply.)
a. Apply over-the-counter artificial tears as needed.
b. Avoid direct exposure to air conditioning.
d. Use nontearing baby shampoo to gently scrub the eyelids.

Patients with dry eye are encouraged to use OTC artificial tears to help moisten the eyes.
Avoiding exposure to fans, air conditioning, and wind is recommended. Nontearing baby
shampoo may be used to cleanse the lids in patients with eyelid inflammation. Topical steroid
eye drops should be used sparingly and for short periods of time. Tetrahydozoline drops
constrict blood vessels and may dry eyes further.
Chapter 58: Nasolacrimal Duct Obstruction and Dacryocystitis
1. An adult patient with a history of recurrent sinusitis and allergic rhinitis reports chronic
tearing in one eye, ocular discharge, and eyelid crusting. The provider suspects nasolacrimal
duct obstruction. Which initial treatment will the provider recommend?
d. Warm compresses

This is most likely acquired nasolacrimal duct obstruction. Initial treatment should include
warm compresses. Antibiotics are only used if infection is present. Nasolacrimal duct probing
is not useful for acquired conditions; definitive treatment usually requires surgery.

2. A patient is diagnosed with dacryocystitis. The provider notes a painful lacrimal sac abscess
that appears to be coming to a head. Which treatment will be useful initially?
b. Incision and drainage

When an abscess is present and coming to a head, incision and drainage may be useful.
Definitive treatment with lacrimal bypass surgery will be performed once the acute episode
has resolved. Eyelid scrubs and topical ointments are not effective.

Chapter 59: Preseptal and Orbital Cellulitis


1. Which is the most common cause of orbital cellulitis in all age groups?
c. Local spread from the ethmoid sinus

Because the membrane separating the ethmoid sinus from the orbit is literally paper-thin, this
is the most common source of orbital infection in all age groups. Bacteremic spread,
inoculation from localized trauma, and paranasal sinus spread all may occur, but are less
common.

2. A child’s optic assessment data include unilateral eyelid edema, warmth, and erythema but no
pain with ocular movement is reported. Which characteristic is most likely true about this
child’s infection?
d. The eye is typically spared without conjunctivitis.

This child has symptoms of preseptal cellulitis in which the eye is typically spared. The other
findings are consistent with orbital cellulitis.
MULTIPLE RESPONSE
1. A patient is experiencing eyelid swelling with erythema and warmth and reports pain with eye
movement. Which diagnostic tests will be performed to confirm a diagnosis of orbital
cellulitis? (Select all that apply.)
b. Complete blood count
c. CT scan of orbits

A complete blood count will help distinguish infectious from noninfectious orbital cellulitis.
A CT scan or the orbits is necessary to confirm the diagnosis. Blood cultures do not confirm
the diagnosis of orbital cellulitis but may be used to evaluate whether septicemia is occurring.
Lumbar puncture is indicated if meningitis is suspected. Visual acuity testing may be used to
monitor recovery.
Chapter 61: Traumatic Ocular Disorders
1. A child sustains an ocular injury in which a shard of glass from a bottle penetrated the eye
wall. The emergency department provider notes that the shard has remained in the eye. Which
term best describes this type of injury?
a. Intraocular foreign body

When a portion of the insulting object enters and remains in the eye, the injury is correctly
referred to as an intraocular foreign body. A penetrating injury occurs when something
penetrates through the eye wall without an exit wound. A perforating injury occurs when the
object has both an entry and an exit wound. A ruptured globe injury occurs when blunt force
causes the eye wall to rupture.

2. A patient experiences a penetrating injury to one eye caused by scissors. The provider notes a
single laceration away from the iris that involves the anterior but not the posterior segment.
What is the prognosis for this injury?
a. Because the posterior segment is not involved, the prognosis is good.

Mechanical energy imparted from sharp objects generally results in lacerations, with
disruption that is more localized. The prognosis is better if the posterior segment is not
involved. The other complications are more common with globe ruptures.

3. Which protective precaution is especially important in a metal fabrication workshop?


d. Polycarbonate goggles

Polycarbonate goggles, which have better side protection, will protect from foreign bodies
that can reach around other lenses and should be used in very high-risk activities, such as
hammering metal on metal or grinding. 2 mm polycarbonate safety glasses are a minimum
safety precaution. Glasses with UVB protection are used in occupations where sunlight
exposure is high. Eyewash stations are necessary where splash injuries or chemical exposures
are possible.

Chapter 62: Auricular Disorders


1. A primary care provider notes painless, hard lesions on a patient’s external ears that expel a
white crystalline substance when pressed. What diagnostic test is indicated?
a. Biopsy of the lesions
d. Uric acid chemical profile

These lesions are consistent with gout and uric acid deposits. The provider should evaluate
this by ordering a uric acid chemical profile. Biopsy is indicated for any small, crusted,
ulcerated, or indurated lesion that does not heal. Rheumatoid nodules indicate a need for
rheumatoid profiles. Endocrine studies are ordered for patients with calcification nodules.

2. During a routine physical examination, a provider notes a shiny, irregular, painless lesion on
the top of one ear auricle and suspects skin cancer. What will the provider tell the patient
about this lesion?
a. A biopsy should be performed.
Chapter 63: Cerumen Impaction
1. A child has recurrent impaction of cerumen in both ears and the parent asks what can be done
to help prevent this. What suggestion will the provider provide?
a. Cleaning the outer ear and canal with a soft cloth

Parents should be instructed to use a soft cloth to clean the outer ear and canal only. Use of a
cotton-tipped swab or any other implement may push cerumen deeper into the canal and risk
damaging the tympanic membrane. Thermal-auricular therapy is not recommended. Oral
irrigation tools have high pressure and a risk of damage to the tympanic membrane.

2. A patient reports symptoms of otalgia and difficulty hearing from one ear. The provider
performs an otoscopic exam and notes a dark brown mass in the lower portion of the external
canal blocking the patient’s tympanic membrane. What is the initial action?
a. Ask the patient about previous problems with that ear.

Before attempting to remove impacted cerumen, the provider must determine whether the
tympanic membrane (TM) is intact and should ask about pressure equalizing ear tubes, a
history of ruptured TM, and previous ear surgeries. Once the TM is determined to be intact,
the other methods may be attempted, although the curette should only be used if the mass is in
the lateral third of the ear canal.

3. A provider is recommending a cerumenolytic for a patient who has chronic cerumen buildup.
The provider notes that the patient has dry skin in the ear canal. Which preparation is US
Food and Drug Administration (FDA) approved for this use?
a. Carbamide peroxide

Any preparation with carbamide peroxide is FDA approved as a cerumenolytic. Patients with
dry skin in the ear canal should not use any product containing hydrogen peroxide. Liquid
docusate sodium and mineral oil are often used, but do not have specific FDA approval.

Chapter 64: Cholesteatoma


1. A young child has a pale, whitish discoloration behind the tympanic membrane. The provider
notes no scarring on the tympanic membrane (TM) and no retraction of the pars flaccida. The
parent states that the child has never had an ear infection. What do these findings most likely
represent?
b. Congenital cholesteatoma

Patients without history of otitis media or perforation of the TM most likely have congenital
cholesteatoma. Primary acquired cholesteatoma will include retraction of the pars flaccida.
MULTIPLE RESPONSE
1. A child is diagnosed as having a congenital cholesteatoma. What is included in management
of this condition? (Select all that apply.)
a. Antibacterial treatment
d. Removal of debris from the ear canal
e. Surgery to remove the lesion
Cholesteatoma is treated with antibiotics, removal of debris from the ear canal, and possibly
surgery. PETs and irrigation of the ear canal are not part of treatment for cholesteatoma.
Chapter 65: Impaired Hearing
1. A child who has recurrent otitis media fails a hearing screen at school. The provider suspects
which type of hearing loss in this child?
b. Conductive

A common cause of conductive loss is fluid in the middle ear as a result of chronic otitis
media with effusion. Central hearing loss is related to CNS disorders. Mixed-type hearing loss
is related to causes of both conductive and sensorineural hearing loss. Sensorineural hearing
loss is caused by damage to the structures in the inner ear, usually caused by infection,
barotrauma, or trauma.

2. A result of screening audiogram on a patient is abnormal. Which test may the primary
provider perform next to further evaluate the cause of this finding?
d. Tympanogram

A screening tympanogram may be performed by a primary provider to determine tympanic


membrane mobility and may help in identifying the presence of infection, fluid, or changes in
middle ear pressure. The other tests are performed by audiologists, not primary care providers.
MULTIPLE RESPONSE
1. Which are risk factors for developing hearing loss caused by presbycusis?
a. Diabetes
c. High blood pressure
e. Smoking

Presbycusis is a gradual degeneration within the cochlea that accompanies aging. Diabetes,
high blood pressure, and smoking may hasten these changes. GERD and liver disease are not
associated with an increased rate of changes.

Chapter 66: Inner Ear Disturbances


1. A patient is suspected of having vestibular neuritis. Which finding on physical examination is
consistent with this diagnosis?
c. Spontaneous horizontal nystagmus

2. A patient reports several episodes of acute vertigo, some lasting up to an hour, associated with
nausea and vomiting. What is part of the initial diagnostic workup for this patient?
a. Audiogram

An audiogram and magnetic resonance imaging (MRI) are part of basic testing for Meniere’s
disease. The other testing may be performed by an otolaryngologist after referral.

MULTIPLE RESPONSE
1. Which symptoms may occur with vestibular neuritis? (Select all that apply.)
a. Disequilibrium
d. Nausea and vomiting
e. Tinnitus
Vestibular neuritis can cause severe vertigo, disequilibrium, nausea, vomiting, and tinnitus,
but not fever or hearing loss.
Chapter 67: Otitis Externa
1. A patient reports a feeling of fullness and pain in both ears and the practitioner elicits
exquisite pain when manipulating the external ear structures. What is the likely diagnosis?
a. Acute otitis externa

This patient’s symptoms are classic for acute otitis externa. Chronic otitis externa more
commonly presents with itching. Acute otitis media is accompanied by fever and tympanic
membrane inflammation, but not external canal inflammation. Otitis media with effusion
causes a sense of fullness but not pain.

2. A patient has an initial episode otitis external associated with swimming. The patient’s ear
canal is mildly inflamed, and the tympanic membrane is not involved. Which medication will
be ordered?
a. Cipro HC

In the absence of a culture, the provider should choose a medication that is effective against
both P. aeruginosa and S. aureus. Cipro HC covers both organisms and also contains a
corticosteroid for inflammation. Fluconazole is an oral antifungal medication used when
fungal infection is present. Neomycin alone does not cover these organisms. Vinegar and
alcohol are used to treat mild fungal infections.
MULTIPLE RESPONSE
1. Which are risk factors for developing otitis externa? (Select all that apply.)
c. Having underlying diabetes mellitus
d. Use of ear plugs and hearing aids
e. Vigorous external canal hygiene

Otitis externa is a cellulitis of the external canal that develops when the integrity of the skin is
compromised. Diabetes mellitus predisposes patients to skin disorders. Using devices that
cause moisture retention and irritation will increase the risk. Vigorous cleansing removes
protective cerumen. Warm, high-humidity environments increase risk. The disease is not
contagious.
Chapter 68: Otitis Media
1. A pediatric patient’s assessment confirms the patient has otalgia, a fever of 38.8°C, and a
recent history of upper respiratory examination. The examiner is unable to visualize the
tympanic membranes in the right ear because of the presence of cerumen in the ear canal. The
left tympanic membrane is dull gray with fluid levels present. What is the correct action?
c. Remove the cerumen and visualize the tympanic membrane.

The AAP 2013 guidelines strongly recommend visualization of the tympanic membrane to
accurately diagnose otitis media and not to treat based on symptoms alone. The practitioner
should attempt to remove the cerumen to visualize the tympanic membrane. A tympanogram
cannot be performed when cerumen is blocking the canal. Because the child may have an
acute ear infection, antibiotics may be necessary.

2. Which patient may be given symptomatic treatment with 24 hours follow-up assessment
without initial antibiotic therapy?
a. A 36-month-old with fever of 38.5°C, mild otalgia, and red, non-bulging TM

Children older than 24 months with fever less than 39°C and nonsevere symptoms may be
watched for 24 hours with symptomatic treatment. Children with otorrhea, those with severe
AOM, and any children with fever greater than 39°C should be given antibiotics.

MULTIPLE RESPONSE
1. Which symptoms in children are evaluated using a parent-reported scoring system to
determine the severity of pain in children with otitis media? (Select all that apply.)
a. Appetite
b. Difficulty sleeping
e. Tugging on ears

Decreased appetite, difficulty sleeping, and tugging on ears are part of the Acute Otitis Media
Severity of Symptom Scale used to evaluate pediatric pain. Children may refuse to cooperate
for reasons other than pain. Poor hearing is not part of the pain assessment.

Chapter 69: Tympanic Membrane Perforation


1. A patient reports ear pain and difficulty hearing. An otoscopic examination reveals a small
tear in the tympanic membrane of the affected ear with purulent discharge. What is the initial
treatment for this patient?
c. Prescribe antibiotic ear drops.
This perforation is most likely due to infection and should be treated with antibiotic ear drops.
Wicks are used for otitis externa. The ear canal should not be irrigated to avoid introducing
fluid into the middle ear. It is not necessary to refer unless the perforation does not heal.

2. A patient reports ear pain after being hit in the head with a baseball. The provider notes a
perforated tympanic membrane. What is the recommended treatment?
d. Refer the patient to an otolaryngologist for evaluation.
Patients with traumatic or blast injuries causing perforations of the tympanic membranes
should be referred to specialists to determine whether damage to inner ear structures has
occurred. For an uncomplicated perforation, the other interventions are all appropriate.
Chapter 70: Chronic Nasal Congestion and Discharge
1. A patient reports persistent nasal blockage, nasal discharge, and facial pain lasting on the right
side for the past 5 months. There is no history of sneezing or eye involvement. The patient has
a history of seasonal allergies and takes a non-sedating antihistamine. What does the provider
suspect is the cause of these symptoms?
c. Chronic rhinosinusitis
Chronic rhinosinusitis is present when symptoms occur longer than 12 weeks. Sneezing and
itchy, watery eyes tend to occur with allergic rhinitis. Autoimmune vasculitides affects upper
and lower respiratory tracts as well as the kidneys. Rhinitis medicamentosa occurs with use of
nasal decongestants and not oral antihistamines.

2. A provider determines that a patient has chronic rhinosinusitis without nasal polyps. What is
the first-line treatment for this condition?
a. Intranasal corticosteroids
Intranasal corticosteroids are the mainstay of treatment for CRS. Oral decongestants should be
used sparingly, only when symptoms are intolerable. Topical decongestants can cause
rebound symptoms. Systemic steroids are not indicated.

3. A pregnant woman develops nasal congestion with chronic nasal discharge. What is the
recommended treatment for this patient?
c. Saline lavage
Saline lavage is recommended for pregnancy rhinitis; the condition will resolve after delivery.
There is no human data on the safety of intranasal corticosteroids during pregnancy.
Prophylactic antibiotics are not indicated; this is not an infectious condition. Topical
decongestants can cause rebound symptoms.

Chapter 71: Epistaxis


1. A patient has bilateral bleeding from the nose with bleeding into the pharynx. What is the
initial intervention for this patient?
b. Assess airway safety and vital signs.
Bilateral epistaxis into the pharynx is more indicative of a posterior bleed which is more likely
to be severe. The most important intervention is to ensure airway safety and determine
stability of vital signs. Other measures are taken as needed.

2. A patient is in the emergency department with unilateral epistaxis that continues to bleed after
15 minutes of pressure on the anterior septum and application of a topical nasal decongestant.
The provider is unable to visualize the site of the bleeding. What is the next measure for this
patient?
c. Nasal packing
Nasal packing is used if bleeding continues after initial measures. Chemical cautery and
electrocautery are used only if the site of bleeding is visualized. Petrolatum ointment is
applied once the bleeding is stopped.

1. A patient has recurrent epistaxis without localized signs of irritation. Which laboratory tests
may be performed to evaluate this condition? (Select all that apply.)
b. CBC with type and crossmatch
d. PT and PTT e. PT/INR
Chapter 72: Nasal Trauma
1. A child is hit with a baseball bat during a game and sustains an injury to the nose, along with a
transient loss of consciousness. A health care provider at the game notes bleeding from the
child’s nose and displacement of the septum. What is the most important intervention
initially?
b. Immobilizing the child’s head and neck and call 911

Nasal trauma resulting in loss of consciousness and possible neck injury are emergencies. The
provider should take cervical spine precautions and call 911 for transport to an emergency
room. The other interventions may be performed once the child’s head and neck are stable.

2. A provider performs a nasal speculum examination on a patient who sustained nasal trauma in
a motor vehicle accident. The provider notes marked swelling of the nose, instability and
crepitus of the nasal septum with no other facial bony abnormalities and observes a rounded
bluish mass against the nasal septum. Which action is necessary initially?
d. Urgent drainage of the mass

A rounded bluish or purplish mass indicates a septal hematoma and must be drained urgently
for cosmetic purposes to prevent loss of nasal cartilage caused by loss of blood supply to this
area. This patient has no signs of facial fractures, so this exam may be deferred. Ice packs are
part of ongoing management, but not a priority. The nasal fracture may be reduced within the
first 3 to 5 days after injury.

3. An alert, irritable 12-month-old child is brought to the emergency department by a parent who
reports that the child fell into a coffee table. The child has epistaxis, periorbital ecchymosis,
and nasal edema. Nares are patent, and the examiner palpates instability and point tenderness
of the nasal septum. The orbital structures appear intact. What is an urgent action for this
patient?
d. Involvement of social services

Young children and infants generally do not engage in activities that cause the high impact
needed to cause a nasal fracture and nasal structures, which have more cartilage than adults,
are at much lower risk of fracture. Child abuse must be suspected in this case. Assessment of
tetanus status and application of symptomatic treatment may be ongoing but are not urgent.
Nasal reduction surgery may be deferred for several days.
Chapter 73: Rhinitis
1. A patient has recurrent sneezing, alterations in taste and smell, watery, itchy eyes, and thin,
clear nasal secretions. The provider notes puffiness around the eyes. The patient’s vital signs
are normal. What is the most likely diagnosis for this patient?
b. Allergic rhinitis

Patients with symptoms described above typically have allergic rhinitis. Sinusitis causes facial
pain, fever, and purulent discharge. Viral rhinitis will also cause purulent discharge and other
symptoms of URI.

2. A patient has seasonal rhinitis symptoms and allergy testing reveals sensitivity to various trees
and grasses. What is the first-line treatment for this patient?
c. Intranasal steroids

Intranasal steroids are the mainstay of treatment and are the most effective medication for
preventing symptoms. Antihistamine sprays are helpful but are not first-line treatments.
Intranasal cromolyn can be effective but must be used four times daily. Oral antihistamines
are used in conjunction with intranasal steroids but are less effective than the steroids.

3. A patient is concerned about frequent nasal stuffiness and congestion that begins shortly after
getting out of bed in the morning. The patient denies itching and sneezing. A physical
examination reveals erythematous nasal mucosa with scant watery discharge. What treatment
will the provider recommend for this patient?
b. Daily intranasal steroids

This patient has symptoms of vasomotor or idiopathic rhinitis. Intranasal steroids are an
effective treatment. Immunotherapy is not effective. This type of rhinitis typically does not
respond to antihistamines. Oral decongestants are effective, but are best used around the
clock, not just prn.
Chapter 74: Sinusitis
1. A patient presenting with nasal congestion, fever, purulent nasal discharge, headache, and
facial pain begins treatment with amoxicillin-clavulanate. At a follow-up visit 10 days after
initiation of treatment, the patient continues to have purulent discharge, congestion, and facial
pain without fever. What is the next course of action for this patient?
c. An antibiotic based on likely resistant organism

Treatment failure is seen in patients who do not have symptom improvement and the provider
has re-confirmed the diagnosis of ABRS and assessed for complications. In these patients, the
choice of antibiotic treatment is based on likely resistant organisms. The lack of fever shows
improvement, so this antibiotic may be used. CT scan is usually not performed in adults
unless other complications are present or suspected. Referral to an otolaryngologist is
necessary if no improvement after the second course of antibiotics. Azithromycin is not used
in adults unless pregnant, due to resistance patterns.

2. A patient with allergic rhinitis develops acute sinusitis and begins treatment with an antibiotic.
Which measure may help with symptomatic relief for patients with underlying allergic
rhinitis?
a. Intranasal steroids

Intranasal steroids should be considered for symptomatic relief for patients with sinusitis,
especially those with allergic rhinitis. Oral mucolytics have little support in efficacy. Saline
solution rinses may provide some relief, but there is no evidence to support their usefulness.
Topical decongestants do decrease nasal congestion and edema, but the potential harm of
rebound congestion requires recommendation with caution.

MULTIPLE RESPONSE
1. Which are potential complications of chronic or recurrent sinusitis? (Select all that apply.)
c. Meningitis
d. Orbital infection
e. Osteomyelitis

Complications of chronic or recurrent sinusitis include spread of infection to other tissues and
may cause meningitis, orbital cellulitis, and osteomyelitis. Allergic rhinitis and asthma are
associated with chronic sinusitis, but not complications of this condition.
Chapter 77: Dental Abscess
Buttaro: Primary Care: A Collaborative Practice, 6th Edition
MULTIPLE CHOICE
1. A patient reports tooth pain in a lower molar and the provider notes a mobile tooth with
erythema and edema of the surrounding tissues without discharge. Which is the initial course
of action by the provider?
b. Prescribe amoxicillin and refer to a dentist in 2 to 3 days.

The primary provider may prescribe antibiotics, especially if the surrounding tissues are
infected. Patients should follow up with a dentist in 2 to 3 days. The primary provider
generally does not perform I&D; this should be done by the dentist. Follow-up should be with
a dentist in 2 to 3 days, not 1 week. Emergency surgery is indicated if there is a question of
airway compromise.

2. A patient has been taking amoxicillin for treatment of a dental abscess. In a follow-up visit,
the provider notes edema of the eyelids and conjunctivae. What is the next action?
a. Hospitalize the patient for an endodontist consultation.

This patient has signs of complications and requires hospitalization with management by a
dentist or endodontist. Changing the antibiotic without consultation is not recommended.
Prompt hospitalization is required.

Chapter 78: Diseases of the Salivary Glands


1. A patient reports painful swelling in the mouth with increased pain at mealtimes. The provider
notes a mass in the salivary gland region. What is the likely cause of these symptoms?
b. Sialolithiasis

Sialolithiasis is a noninfectious salivary gland disorder characterized by pain at mealtimes


caused by blockage of the salivary duct by stones. Basal cell adenoma is a noninfectious cause
of salivary gland inflammation that is generally painless. Sjögren syndrome manifests with
xerostomia and abnormal taste. Warthin’s tumor causes a painless, unilateral mass.

2. A patient has a chronic swelling of the parotid gland that is unresponsive to antibiotics and
which has not increased in size. Which diagnostic test is indicated?
b. Fine-needle aspiration

Chronic lesions may represent tuberculosis or malignancies, so fine-needle aspiration is


indicated to rule out these diseases. Radiological studies are used to identify the extent of
disease but are usually not diagnostic.

3. A patient has parotitis and cultures are positive for actinomycosis. What is the initial treatment
for this condition?
a. Intravenous (IV) penicillin

IV penicillin followed by the oral form (Penicillin V) for several months is indicated for
actinomycosis; specialist consultation is indicated for patients with penicillin allergy.
Clindamycin and erythromycin are used for PCN allergy. Topical antibiotics are not effective.
Chapter 79: Epiglottitis
1. The provider sees a child with a history of high fever and sore throat. When entering the exam
room, the provider finds the child sitting in the tripod position and notes stridor, drooling, and
anxiety. What is the initial action for this patient?
c. Obtain an immediate consultation with an otolaryngologist.

Patients with suspected epiglottitis, with high fever, sore throat, stridor, drooling, and
respiratory distress should be referred immediately to otolaryngology. Starting an IV or
having the child lie down will increase distress and may precipitate laryngospasm. The throat
should not be examined because it may cause laryngospasm.

2. An adult patient is seen in clinic with fever, sore throat, and dysphagia. Which diagnostic test
will the provider order to confirm a diagnosis of epiglottitis?
c. Fiberoptic nasopharyngoscopy

Fiberoptic nasopharyngoscopy allows direct visualization of the epiglottis and is used


increasingly with adult patients suspected of having epiglottitis. Blood cultures and a CBC
may be drawn as part of the workup to help guide antimicrobial therapy but are not diagnostic.
A lateral neck film is not always diagnostic with adults.

3. An adult patient is diagnosed with epiglottitis secondary to a chemical burn. Which


medication will be given initially to prevent complications?
c. Dexamethasone

This case of epiglottitis does not have an infectious cause, so antibiotics are not given unless
there are symptoms of infection. A corticosteroid can decrease the need for intubation.
Chapter 80: Oral Infections
1. A patient reports painful oral lesions 3 days after feeling pain and tingling in the mouth. The
provider notes vesicles and ulcerative lesions on the buccal mucosa. What is the most likely
cause of these symptoms?
c. Herpes simplex virus (HSV)

HSV infections generally start with a prodrome of tingling, pain, and burning followed by
vesicular and ulcerative lesions. Bacterial infection presents with inflammation of the gingiva,
bleeding, and ulceration with or without purulent discharge. Candida albicans appear as
white, cottage cheese-like lesions that may be removed, but may cause bleeding when
removed. HPV manifests as white, verrucous lesions individually or in clusters.

2. A patient diagnosed with gingival inflammation presents with several areas of ulceration and a
small amount of purulent discharge. What is required to diagnose this condition?
c. Physical examination

This patient has symptoms consistent with gingivitis, which may be diagnosed by physical
examination alone. Cultures are not necessary unless systemic disease is present. A
microscopic exam of oral scrapings to look for hyphae may be performed to diagnose candida
infections. A Tzanck smear is performed to confirm a diagnosis of herpes simplex.

3. A patient reports painful oral lesions and the provider notes several white, verrucous lesions in
clusters throughout the mouth. What is the recommended treatment for this patient?
d. Surgical excision

White, verrucous lesions in clusters are diagnostic for human papilloma virus (HPV) infection
which is treated with surgical excision. Nystatin suspension is given for candida infection.
Oral acyclovir is used for herpes simplex virus (HSV) infection. Oral hygiene measures are
used for gingivitis.

Chapter 81: Parotitis


1. Which physical examination finding suggests viral rather than bacterial parotitis?
a. Clear discharge from Stensen’s duct

Viral parotitis generally produces clear discharge. Enlargement and pain of affected glands
may be nonspecific or is associated with tuberculosis (TB) infection. A gradual reduction in
saliva, resulting in xerostomia, is characteristic of human immunodeficiency virus (HIV)
infection. Unilateral edema is more often bacterial.

2. A patient diagnosed with acute suppurative parotitis has been taking amoxicillin-clavulanate
for 4 days without improvement in symptoms. The provider will order an antibiotic for
Methicillin-resistant S. aureus. Which other measure may be helpful?
c. Surgical drainage

If improvement does not occur after 3 to 4 days of antibiotics, surgical drainage is appropriate.
Warm compresses are recommended for comfort. Chewing gum and other methods to
stimulate the production of saliva are recommended. Steroids are questionable and topical
steroids will have little effect.

MULTIPLE RESPONSE
1. What are factors associated with acute suppurative parotitis? (Select all that apply.)
b. Anticholinergic medications
c. Diabetes mellitus
e. Radiotherapy

Anticholinergic medications decrease salivary flow and increase the risk for parotitis. Chronic
diseases, including diabetes mellitus, can increase the risk. Radiotherapy and other procedures
may increase the risk. Allergies and hypervolemia do not increase the risk.

Chapter 82: Peritonsillar Abscess


Buttaro: Primary Care: A Collaborative Practice, 6th Edition
MULTIPLE CHOICE
1. An adolescent presents with fever, chills, and a severe sore throat. On exam, the provider
notes foul-smelling breath and a muffled voice with marked edema and erythema of the
peritonsillar tissue. What will the primary care provider do?
d. Refer the patient to an otolaryngologist.

This patient has clinical signs of peritonsillar abscess, which may be diagnosed on clinical
signs alone. Patients with peritonsillar abscess should be referred to an otolaryngologist for
possible I&D of the abscess and hospitalization for IV antibiotics. A rapid strep and culture
are not indicated. Oral antibiotics generally do not work.

2. A patient is diagnosed with peritonsillar abscess and will be hospitalized for intravenous
antibiotics. What additional treatment will be required?
b. Needle aspiration of the abscess

Needle aspiration, antibiotics, pain medication, and hydration can effectively treat
peritonsillar abscess. Intubation is not performed unless the airway is compromised. Systemic
corticosteroid administration is useful, but not required in all cases. Tonsillectomy alone is
sometimes performed if recurrent tonsillitis or peritonsillar abscess is present.

Chapter 83: Pharyngitis and Tonsillitis


1. A patient reports a sudden onset of sore throat, fever, malaise, and cough. The provider
notes mild erythema of the pharynx and clear rhinorrhea without cervical lymphadenopathy.
What is the most likely cause of these symptoms?
d. Viral pharyngitis

Viral pharyngitis will cause sore throat, fever, and malaise and is often accompanied by URI
symptoms of cough and runny nose. Allergic pharyngitis usually also causes dryness. GAS
causes high fever, cervical adenopathy, and marked erythema with exudate. Infectious
mononucleosis will cause an exudate along with cervical adenopathy.

2. A patient presents with sore throat, a temperature of 38.5°C, tonsillar exudates, and cervical
lymphadenopathy. What will the provider do next to manage this patient’s symptoms?
b. Perform a rapid antigen detection test (RADT).

The RADT is performed initially to determine whether Group A 􀁅-hemolytic Streptococcus


(GAS) is present. The ASO titer is not used during initial diagnostic screening. Penicillin
should not be given empirically. A referral to a specialist is not required for GAS infection.

3. A school-age child has had 5 episodes of tonsillitis in the past year and 2 episodes the
previous year. The child’s parent asks the provider if the child needs a tonsillectomy. What
will the provider tell this parent?
a. Current recommendations do not support tonsillectomy for this child.

Management of chronic pharyngitis or tonsillitis with GAS infection may require


tonsillectomy. Tonsillectomy is not performed as often as in the past due to retrospective
studies that suggest there is little benefit and a chance of significant postsurgical
complications. Radiographic studies are not indicated.

Chapter 84: Acute Bronchitis


1. A patient develops a dry, nonproductive cough and is diagnosed with bronchitis. Several days
later, the cough becomes productive with mucoid sputum. What may be prescribed to help
with symptoms?
b. Antitussive medication

Antitussive medications are occasionally useful for short-term relief of coughing. Antibiotic
therapy is generally not needed and should be avoided unless a bacterial cause is likely.
Bronchodilator medications show no demonstrated reduction in symptoms and are not
recommended. Mucokinetic agents have no evidence to support their use.

2. An adult patient who had pertussis immunizations as a child is exposed to pertussis and
develops a runny nose, low-grade fever, and upper respiratory illness symptoms without a
paroxysmal cough. What is recommended for this patient?
a. A prescription for a macrolides

Adults previously immunized against pertussis may still get the disease without the classic
whooping cough sign seen in children and are contagious from the beginning of the catarrhal
stage of runny nose and common cold symptoms. Macrolide antibiotics are useful for
reducing symptoms and for decreasing shedding of bacteria to limit spread of the disease.
Patients should be isolated for 5 days from the start of treatment. Pertussis vaccine booster
will not alter the course of the disease once exposed. Symptomatic care only will not reduce
symptoms or decrease disease spread.

3. A 35-year old patient develops acute viral bronchitis. Which is the focus for the management
of symptoms in this patient?
c. Supportive care
Chapter 85: Asthma
1. A patient is seen in clinic for an asthma exacerbation. The provider administers three
nebulizer treatments with little improvement, noting a pulse oximetry reading of 90% with 2 L
of oxygen. A peak flow assessment is 70%. What is the next step in treating this patient?
b. Admit to the hospital with specialist consultation.

Patients having an asthma exacerbation should be referred if they fail to improve after three
nebulizer treatments or three epinephrine injections, have a peak flow less than 70% and a
pulse oximetry reading less than 90% on room air. Giving more nebulizer treatments or
administering epinephrine is not indicated. The patient will most likely be given IV
corticosteroids; oral corticosteroids would be given if the patient is managed as an outpatient.

2. An adult develops chronic cough with episodes of wheezing and shortness of breath. The
provider performs chest radiography and other tests and rules out infection, upper respiratory,
and gastroesophageal causes. Which test will the provider order initially to evaluate the
possibility of asthma as the cause of these symptoms?
d. Spirometry

Spirometry is recommended at the time of initial assessment to confirm the diagnosis of


asthma. Allergy testing is performed only if allergies are a possible trigger. The methacholine
challenge test is performed if spirometry is inconclusive. PEFR is generally used to monitor
asthma symptoms.

3. A patient diagnosed with asthma calls the provider to report having a peak flow measure of
75%, shortness of breath, wheezing, and cough, and tells the provider that the symptoms have
not improved significantly after a dose of albuterol. The patient uses an inhaled corticosteroid
medication twice daily. What will the provider recommend?
a. Administering two more doses of albuterol

The patient is experiencing an asthma exacerbation and should follow the asthma action plan
(AAP) which recommends three doses of albuterol before reassessing. The peak flow is above
70%, so ED admission is not indicated. The patient may be instructed to come to the clinic for
oxygen saturation and spirometry evaluation after administering the albuterol. An oral
corticosteroid may be prescribed if the patient will be treated as an outpatient after following
the AAP.
Chapter 86: Chest Pain (Noncardiac)
1. A patient presents to an emergency department reporting chest pain. The patient describes the
pain as being sharp and stabbing and reports that it has been present for several weeks. Upon
questioning, the examiner determines that the pain is worse after eating. The patient reports
getting relief after taking a friend’s nitroglycerin during one episode. What is the most likely
cause of this chest pain?
c. Esophageal pain

Pain that is constant for weeks or is sharp and stabbing is not likely to be cardiac in origin.
Both esophageal and cardiac causes will be attenuated with sublingual nitroglycerin. Aortic
dissection will cause an abrupt onset with the greatest intensity at the beginning of the pain.
Pleural pain is usually related to deep breathing or cough.

2. When a patient reports experiencing chronic chest pain that occurs after meals, the provider
suspects gastroesophageal reflux disease (GERD) and prescribes a proton pump inhibitor.
After 2 months the patient reports improvement in symptoms. What is the next action in
treating this patient?
a. Wean patient from proton pump inhibitor (PPI).

Often the effectiveness of treatment with a PPI is diagnostic and is equal to or better than
more invasive and expensive testing. If the patient continues to show improvement, the patient
is weaned off of the PPI. Most patients do well and there is no need to order tests or refer for
evaluation. If patients do not do well, further testing is needed.

3. A high school athlete reports recent onset of chest pain that is aggravated by deep breathing
and lifting. A 12-lead electrocardiogram in the clinic is normal. The examiner notes localized
pain near the sternum that increases with pressure. What will the provider do next?
c. Recommend an NSAID.

This patient has symptoms consistent with chest wall pain because chest pain occurs with
specific movement and is easily localized. Since the ECG is normal, there is no need to refer
to a cardiologist. The patient does not have symptoms of pneumonia, so a radiograph or
antibiotic is not needed. NSAIDs are recommended for comfort.
Chapter 87: Chronic Cough
1. A patient recovering from a viral infection has a persistent cough 6 weeks after the infection.
What will the provider do?
d. Reassure the patient that this is common after such an infection

Postinfection cough is common after a viral infection and may persist up to 8 weeks after the
infection; this type of cough generally needs no intervention. It is not necessary to perform
chest radiography unless secondary infection is suspected. Antibiotics are not indicated.
Unless the cough persists after 8 weeks, asthma testing is not indicated.

2. A nonsmoking adult with a history of cardiovascular disease reports having a chronic cough
without fever or upper airway symptoms. A chest radiograph is normal. What will the
provider consider initially as the cause of this patient’s cough?
a. ACE inhibitor medication use

About 10% of patients taking ACE inhibitors will develop chronic cough. COPD, GERD, and
psychogenic causes are possible, but given this patient’s cardiovascular history, the possibility
of ACE inhibitor-induced cough should be investigated initially.

3. A young adult patient develops a cough persisting longer than 2 months. The provider
prescribes pulmonary function tests and a chest radiograph, which are normal. The patient
denies abdominal complaints. There are no signs of rhinitis or sinusitis and the patient does
not take any medications. What will the provider evaluate next to help determine the cause of
this cough?
b. Methacholine challenge test
c. Sputum culture

Chronic cough without other symptoms may indicate asthma. If PFTs are normal, a
methacholine challenge test may be performed. 24-hour esophageal pH monitoring is
sometimes performed to evaluate for GERD, but this patient does not have abdominal
symptoms and this test is usually not performed because it is inconvenient. Sputum culture is
not indicated. TB is less likely.
Chapter 88: Chronic Obstructive Pulmonary Disease
1. Which is characteristic of obstructive bronchitis and not emphysema?
c. Mild alteration in lung tissue compliance

Obstructive bronchitis causes much less parenchymal damage than emphysema does, so there
is milder alteration in lung tissue compliance. The other symptoms are characteristic of
emphysema.

2. Which test is the most diagnostic for chronic obstructive pulmonary disease (COPD)?
d. Spirometry for FVC and FEV1

Spirometry testing is the gold standard for diagnosis and assessment of COPD because it is
reproducible and objective. The forced expiratory time maneuver is easy to perform in a clinic
setting and is a good screening to indicate a need for confirmatory spirometry. Lung
radiographs are non-specific but may indicate hyperexpansion of lungs. The COPD
assessment test helps measure health status impairment in persons already diagnosed with
COPD.

3. A patient diagnosed with chronic obstructive pulmonary disease reports daily symptoms of
dyspnea and cough. Which medication will the primary health care provider prescribe?
a. Ipratropium bromide

Ipratropium bromide is an anticholinergic medication and is used as first-line therapy in


patients with daily symptoms. Pirbuterol acetate and salmeterol xinafoate are both
beta2-adrenergics and are used to relieve bronchospasm; pirbuterol is a short-term medication
used for symptomatic relief and salmeterol is a long-term medication useful for reducing
nocturnal symptoms. Theophylline is a third-line agent.
Chapter 89: Dyspnea
1. A young adult patient without a previous history of lung disease has an increased respiratory
rate and reports a feeling of “not getting enough air.” The provider auscultates clear breath
sounds and notes no signs of increased respiratory effort. Which diagnostic test will the
provider perform initially?
b. Complete blood count

This patient has no signs indicating lung disease but does exhibit signs of hypoxia. A CBC
would evaluate for anemia, which is a more common cause of hypoxia in otherwise healthy
adults. Chest radiography is used to evaluate infectious causes. CT is used if interstitial lung
disease is suspected. Spirometry is useful to diagnose asthma and COPD.

2. A patient reports shortness of breath with activity and exhibits increased work of breathing
with prolonged expirations. Which diagnostic test will the provider order to confirm a
diagnosis in this patient?
c. Spirometry

The patient has signs of either asthma or COPD. Spirometry is essential to both the diagnosis
and management of these diseases. ABGs are useful when evaluating severity of
exacerbations but are not specific to these diseases. Blood cultures are drawn if pneumonia is
suspected. A ventilation/perfusion scan is performed to evaluate for pulmonary
thromboembolic disease.

3. An older adult patient diagnosed with chronic obstructive lung disease (COPD) is
experiencing dyspnea and has an oxygen saturation of 89% on room air. The patient has no
history of pulmonary hypertension or congestive heart failure. What will the provider order to
help manage this patient’s dyspnea?
b. Breathing exercises

Formal pulmonary rehabilitation programs, including breathing exercises, are used to manage
long-term disease such as COPD. Anxiolytics and opioids must be used cautiously because of
respiratory depression side effects. Medicare does not approve oxygen supplementation unless
saturations are less than 88% on room air or for patients who have pulmonary hypertension or
CHF who have saturations <89%.
Chapter 90: Hemoptysis
1. A patient with a smoking history of 35 pack years reports having a chronic cough with recent
symptoms of pink, frothy blood on a tissue. The chest radiograph shows a possible nodule in
the right upper lobe. Which diagnostic test is indicated?
a. Coagulation studies
b. Computed tomography (CT)
c. Fiberoptic bronchoscopy
d. Needle biopsy
ANS: B
CT is suggested for initial evaluation of patients at high risk of malignancy, such as a smoker
with >30 pack years, who have suspicious findings on chest radiography. Coagulation studies
are performed for patients taking anticoagulants or a history of coagulopathy. Fiberoptic
bronchoscopy is used with CT but is not the initial test. Needle biopsy is performed if other
tests indicate a tumor.
2. A patient reports coughing up a small amount of blood after a week of cough and fever. The
patient has been previously healthy and does not smoke or work around pollutants or irritants.
What will the provider suspect as the most likely cause of this patient’s symptoms?
a. Infection
b. Lung abscess
c. Malignancy
d. Thromboembolism
ANS: A
In a healthy patient without risk factors who has a cough and fever, infection is the most likely
cause. Lung abscess may occur but is less likely. Malignancy is also less likely.
Thromboembolism is more likely after surgery or with trauma.
3. A patient with hemoptysis and no other symptoms has a normal chest radiograph (CXR),
computed tomography (CT), and fiberoptic bronchoscopy studies. What is the next action in
managing this patient?
a. Observation
b. Prophylactic antibiotics
c. Specialist consultation
d. Surgical intervention
ANS: A
Patients with negative findings on CXR, CT, and bronchoscopy, with no risk factors may be
observed for 3 years. Antibiotics are not indicated, since signs of infection are not present.
Specialty consultation and surgery are not indicated.

Chapter 91: Lung Cancer


Buttaro: Primary Care: A Collaborative Practice, 6th Edition
MULTIPLE CHOICE
1. A patient with a cough has a suspicious lung lesion, a mediastinal lymph mass, and several
bone lesions. What test is indicated to determine histology and staging of this cancer?
a. Biopsy of a bone lesion
b. Bone marrow aspiration and biopsy
c. Bronchoscopy with lung biopsy
d. Thoracentesis and pleural fluid cytology
ANS: A
The diagnosis and stage should be determined in the least invasive manner possible. A single
biopsy of the bone lesion can determine histology and staging. The other procedures are more
invasive and not necessary.
2. A patient with limited stage small cell lung cancer (SCLC) has undergone chemotherapy with
a good initial response to therapy. What will the provider tell this patient about the prognosis
for treating this disease?
a. Surgical resection will improve survival chances dramatically.
b. That relapse is likely with a 2-year overall survival of 50%.
c. There is an 80% chance of 5-year survival.
d. Treatment will proceed with curative intent.
ANS: B
Although SCLC often responds very well initially to chemotherapy, the majority of patients
will relapse and the 2-year survival rates are approximately 50%. Surgical resection does not
play a significant role in the management of SCLC because the majority of patients have
metastatic disease at diagnosis. Treatment is generally palliative.
MULTIPLE RESPONSE
1. When screening for metastatic cancer in a patient with lung cancer, what will the provider
assess for? (Select all that apply.)
a. Reports of headache
b. Increased presence of a cough
c. Diagnostically confirmed low hematocrit
d. Existence of lymph nodes greater than 1 cm
e. Presence of unexplained weight gain greater than 10 pounds
ANS: A, C, D
Headaches may indicate brain metastases. Low hematocrit and lymphadenopathy with nodes
greater than 1 cm also indicate metastasis. Increased cough is a sign of lung cancer itself, not
metastasis. Patients with metastatic cancer have unexplained weight loss of more than 10
pounds.

Chapter 92: Pleural Effusions and Pleurisy


Buttaro: Primary Care: A Collaborative Practice, 6th Edition
MULTIPLE CHOICE
1. A patient reports shortness of breath when in a recumbent position as well as coughing and
pain associated with inspiration. The provider notes distended neck veins during the exam.
What is the likely cause of these findings?
a. Congestive heart failure (CHF)
CHF causes the symptoms described above, with distended neck veins being a significant
finding. Hepatic disease would also cause abdominal distention with ascites and
hepatomegaly. Pulmonary embolus has marked shortness of breath. Pulmonary infection
causes inflammation and a friction rub.
MULTIPLE RESPONSE
1. Which are causes of pleural effusions? (Select all that apply.)
b. Breast cancer
c. Bronchiectasis
d. Congestive heart failure (CHF)
Breast cancer, bronchiectasis, and CHF can all cause pleural effusions. Allergies and
dehydration do not.
Chapter 93: Pneumonia
1. A patient presents with a cough and fever. The provider auscultates rales in both lungs that do
not clear with cough. The patient reports having a headache and sore throat prior to the onset
of coughing. A chest radiograph shows patchy, nonhomogeneous infiltrates. Based on these
findings, which organism is the most likely cause of this patient’s pneumonia?
b. Mycoplasma
c. S. pneumoniae
d. Tuberculosis

Atypical pneumonias, such as those caused by mycoplasma, often present with headache and
sore throat and will have larger areas of infiltrate on chest radiograph. Viral pneumonias show
more diffuse radiographic findings. S. pneumonia will have high fever and cough and distinct
areas of infiltration.

2. A young, previously healthy adult clinic patient reports symptoms of pneumonia including
high fever and cough. Auscultation reveals rales in the left lower lobe. A chest radiograph is
normal. The patient is unable to expectorate sputum. Which treatment is recommended for
this patient?
a. A B-lactam antibiotic plus a fluoroquinolone
b. A respiratory fluoroquinolone antibiotic
c. Empirical treatment with a macrolide antibiotic
d. Hospitalization for intravenous antibiotics

This patient likely has community-acquired pneumonia. The patient has typical symptoms
and, even though the chest radiograph is normal, will require outpatient treatment. For
community-acquired pneumonia in a previously healthy individual, treatment with a
macrolide antibiotic is the recommended first-line therapy. B-lactam plus fluoroquinolone
therapy is used for patients in the ICU. Respiratory fluoroquinolones are used for patients with
underlying disorders who develop pneumonia. Hospitalization is not necessary.

3. A patient was initially treated as an outpatient for pneumonia and then after 2 weeks was
hospitalized after no improvement was evident. The patient continues to show no
improvement after several antibiotic regimens have been attempted. What is the next step in
managing this patient?
a. Administration of the pneumonia vaccine
b. Increasing the dose of the antibiotics
c. Open lung biopsy
d. Performing diagnostic bronchoscopy

Patients who do not respond to antibiotic therapy may have opportunistic fungal or other
infections, bronchogenic carcinoma, or other diseases. Bronchoscopy can exclude or confirm
these. The pneumonia vaccine is preventative for pneumococcal causes and will not help this
patient. Increasing the dose of the antibiotics is not recommended. Open lung biopsy may be
performed if a bronchoscopy is inconclusive.
Chapter 94: Pneumothorax
1. A patient with a central line develops respiratory compromise. What is the initial intervention
for this patient?
a. Lung ultrasonography (US) to determine the cause
b. Obtaining cultures and starting antibiotics
c. Prompt removal of the central line
d. Rapid assessment and resuscitation

Patients with central lines are at increased risk for pneumothorax. Acute respiratory distress is
a medical emergency and assessment and resuscitation should begin immediately. Lung US,
cultures and antibiotics, and removal of the central line may be performed if indicated when
the patient is stabilized.

2. Which method of treatment is used to manage a traumatic pneumothorax?


a. Needle aspiration of the pneumothorax
b. Observation for spontaneous resolution
c. Placement of a small-bore catheter
d. Tube thoracostomy

Traumatic pneumothorax requires tube thoracostomy because of its ability to drain larger
volumes of air along with blood and fluids. Needle aspiration is safe for primary
pneumothorax. Observation for spontaneous resolution is indicated for small pneumothoraces.

Chapter 95: Pulmonary Embolism


1. A patient who has undergone surgical immobilization for a femur fracture reports dyspnea and
chest pain associated with inspiration. The patient has a heart rate of 120 beats per minute.
Which diagnostic test will confirm the presence of a pulmonary embolism (PE)?
b. Computed tomography (CT) angiography

CT angiography is used to diagnose PE. D-dimer assays have good negative predictive value
but have poor positive predictive value, making it useful for excluding but not confirming the
presence of PE. An ECG does not confirm PE but is used to demonstrate comorbid conditions.
ABGs do not confirm PE and are used to identify the degree of respiratory compromise.

2. Which clinical sign is especially worrisome in a patient with a pulmonary embolism (PE)?
c. Hypotension
Hypotension in a patient with PE has a high correlation with acute right ventricular failure and
subsequent death. The other signs are common with PE.

3. A patient develops a pulmonary embolism (PE) after surgery and shows signs of right-sided
heart failure. Which drug will be administered to this patient?
b. Tissue plasminogen activator
Fibrinolytic therapy with recombinant tissue plasminogen activator is given to patients with
hypotension and right-sided heart failure. Heparin is used for its anticoagulant properties in all
patients with PE. Warfarin is not indicated.
Chapter 96: Pulmonary Hypertension
1. A patient with increased left-sided heart pressure will have which type of pulmonary
hypertension?
a. Group 2

Group 2 pulmonary hypertension is associated with increased left-sided heart pressure.

2. A patient who experienced mild pulmonary hypertension with a previously loud second heart
sound on exam now demonstrates edema and jugular vein distension. This indicates which
complication?
b. Right ventricular dysfunction

Right ventricular dysfunction occurs as the disease worsens with manifestations that include
jugular vein distension, edema, and increased liver size. These symptoms do not indicate left
ventricular dysfunction or valvular involvement.

3. A patient diagnosed with pulmonary arterial hypertension (PAH) has increased dyspnea with
activity. Which medication may be prescribed to manage symptom on an outpatient basis?
b. Bosentan

Bosentan helps promote pulmonary artery smooth muscle cell proliferation and improves
exercise capacity. It is also given PO, so is easy to give on an outpatient basis. Inhaled
prostanoids have a short half-life and must be given 6 to 9 times daily. Epoprostenol has a
short half-life and must be given IV. Trepostinil is given IV.

Chapter 97: Sarcoidosis


Buttaro: Primary Care: A Collaborative Practice, 6th Edition
MULTIPLE CHOICE
1. A patient reporting dyspnea and chest pain along with occasional chills and night sweats has a
chest radiograph that shows bilateral hilar lymphadenopathy (BHL) and pulmonary infiltrates.
The provider suspects which classification of sarcoidosis?
b. Stage 2

Stage 1 sarcoidosis is classified based on bilateral hilar lymphadenopathy (BHL) only. Stage 2
presents with BHL and pulmonary infiltrates, stage 3 with pulmonary infiltrates without BHL,
and stage 4 with pulmonary fibrosis.

3. A patient diagnosed with stage 1 sarcoidosis is prescribed a nonsteroidal anti-inflammatory


medication to treat joint discomfort has now developed mild dyspnea and cough. Which
medication will be added to assist in treating this new symptom?
d. An oral corticosteroid

Corticosteroids are begun when pulmonary symptoms develop. Beta-adrenergics are not used.
Antimalarial agents are used to treat chronic skin lesions. Immunosuppressants are used when
corticosteroids are no longer effective or when the disease progresses.
Chapter 98: Cardiac Diagnostic Testing: Noninvasive Assessment of CAD
1. An asymptomatic 63-year-old adult has a low-density lipoprotein level of 135 mg/dL. Which
test is beneficial to assess this patient’s coronary artery disease risk?
b. hsCRP (high-sensitivity CRP)

The hsCRP is useful in asymptomatic men >50 years and women >60 years who have LDL
<160 mg/dL to predict CAD risk. Although the CACS has shown some benefit in patients
with moderate risk, the role for this diagnostic test is unclear. Exercise echocardiography and
myocardial perfusion imaging are not performed initially.

2. Which risk assessment for coronary artery disease is recommended for all female patients?
d. Framingham risk score

The Framingham risk score is a quick method for identifying potential risk for CAD and can
guide providers in choosing subsequent tests based on risk level. The ECG is performed on
women with risk factors. The exercise stress test is useful in symptomatic women who have a
normal ECG. The CACS may be used if moderate risk is present.

Chapter 99: Abdominal Aortic Aneurysm


1. A patient reports abdominal and back pain with anorexia and nausea. During an exam, the
provider notes a pulsatile abdominal mass. What is the initial action?
d. Ultrasound of the mass to determine size (US)

This patient has symptoms consistent with an aortic aneurysm. The initial step is to determine
the size of the aneurysm; this can be done by US. Immediate referral is not necessary. MRI
and CT diagnostic tests are ordered before surgery to evaluate the characteristics of the
aneurysm.

2. A 70-year-old patient presents with an aortic aneurysm measuring 5.0 cm. The patient has
poorly controlled hypertension, and decompensated heart failure. What is the recommendation
for treatment for this patient?
d. Serial ultrasonographic surveillance (US) of the aneurysm

This patient’s aneurysm is less than 5.5 cm and repair is not necessary at this time. Serial US
surveillance is necessary to continue to evaluate size. Repair is risky in patients with
hypertension and heart failure, so avoiding procedures if possible is recommended.
Chapter 100: Cardiac Arrhythmias
1. A patient reports sustained, irregular heart palpitations. What is the most likely cause of
these symptoms?
b. Atrial fibrillation

Atrial fibrillation causes palpitations that are irregular and tend to be sustained. Anemia will
cause rapid palpitations that are regular. Extrasystole causes palpitations or an awareness of
isolated extra beats with a pause. Paroxysmal attacks start and terminate abruptly and are
usually rapid and regular.

2. An adult patient reports frequent episodes of syncope and lightheadedness. The provider
notes a heart rate of 70 beats per minutes. What action will the provider take next?
a. Evaluation of the patient’s orthostatic vital signs

Orthostatic vital signs are helpful to exclude orthostatic hypotension as a cause of syncope
and are easily performed in the clinic. Assessment for vagal bradycardia may be performed
next. ECG and ETT are not recommended as an initial evaluation in a healthy patient, unless
other causes are not determined. Without assessment of the cause of the syncope, cardiac
causes cannot be excluded.
3. A child with a history of asthma is brought to the clinic with a rapid heart rate. A cardiac
monitor shows a heart rate of 225 beats per minute. The provider notifies transport to take
the child to the emergency department. What initial intervention may be attempted in the
clinic?
d. A carotid massage

This child has paroxysmal supraventricular tachycardia (PSVT). Vagal maneuvers or carotid
massage may be attempted to slow the ventricular rate. Adenosine is contraindicated in
patients with asthma. Medications such as beta blockers and digoxin are not used in
emergency treatment of PSVT.
Chapter 101: Carotid Artery Disease
1. During a routine health maintenance examination, the provider auscultates a cervical/carotid
bruit. The patient denies syncope, weakness, or headache. What will the provider do, based on
this finding?
a. Order a carotid duplex ultrasound (US).

1. According to current research, which are associated with a decreased incidence of stroke?
a. Statin therapy for low density lipoproteins (LDL) of <75 mg
c. Glycemic control for patients with diabetes
e. Maintain a body mass index (BMI) of <30 kg/m2
Statin therapy for low density lipoproteins (LDL) of <75 mg, glycemic control for patients
with diabetes, and maintaining a body mass index (BMI) of <30 kg/m2 has shown to lower the
risk of stroke. B-complex vitamins and low-sugar soda have not shown to decreased risk.

Chapter 102: Chest Pain and Coronary Artery Disease


1. A patient reports recurrent chest pain that occurs regardless of activity and is not relieved by
rest. The provider administers a nitroglycerin tablet which does not relieve the discomfort.
What is the next action?
b. Give the patient a beta blocker medication.
Patient with these symptoms who do not respond to nitroglycerin is likely to have
microvascular angina. Treatment is effective with beta blockers. These symptoms are not
characteristic of acute MI, so aspirin is not given. A second nitroglycerin tablet is used for
classic angina. Calcium channel blockers are not indicated.

2. A patient is brought to an emergency department with symptoms of acute ST-segment


elevation MI (STEMI). The nearest hospital that can perform percutaneous coronary
intervention (PCI) is 3 hours away. What is the initial treatment for this patient?
c. Initiate fibrinolytic treatment.

1. Which patient meets the criteria for statin therapy to help prevent atherosclerotic
cardiovascular disease? (Select all that apply.)
b. A 70-year old nondiabetic with a 10-year risk score of 7.5% with an LDL-C of 80
mg/dL
c. An otherwise healthy 25-year old with a low-density lipoprotein (LDL-C) level of
196 mg/dL
d. A 45-year old diabetic with an LDL-C of 150 mg/dL
e. A 60-year old with a history of myocardial infarction
Adults with a history of known cardiovascular disease, including stroke, caused by
atherosclerosis; those with LDL-C level of greater than 190 mg/dL; adults 40 to 75 years, with
diabetes; adults 40 to 75 years, with LDL-C level of 70 to 189 mg/dL and a 5% to 19.9%
10-year risk of developing cardiovascular disease from atherosclerosis, with risk enhancing
factors; adults 40 to 75 years, with LDL-C level of 70 to 189 mg/dL and a 20% or greater
10-year risk of developing cardiovascular disease from atherosclerosis.
Chapter 103: Heart Failure
1. A patient experiencing heart failure with reduced ejection fraction will have which symptoms?
d. Pump failure from left ventricular systolic dysfunction

Heart failure with reduced ejection fraction results in pump failure from ventricular systolic
dysfunction. Heart failure with preserved ejection fraction may have milder symptoms and is
associated with impairment of ventricular filling and relaxation.

2. A patient who has been diagnosed with heart failure for over a year reports being comfortable
while at rest but experiences palpitations and dyspnea when walking to the bathroom. Which
classification of heart failure is appropriate based on these symptoms?
b. Class II

Patients with Class II heart failure (HF) will have slight limitation of activity and will be
comfortable at rest with symptoms occurring with ordinary physical activity. Patients with
Class I HF do not have limitations and ordinary physical activity does not produce symptoms.
With Class III HF, less than usual activity will produce symptoms. With Class IV HF,
symptoms are present even at rest and all physical activity worsens symptoms.

3. A patient who has Class II heart failure is taking an ACE inhibitor and reports a recurrent
cough that does not interfere with sleep or activity. What will the provider do initially to
manage this patient?
c. Provide reassurance that this is a benign side effect

Cough occurs in about 20% of patients who take ACE inhibitors and is not dangerous. The
patient should be reassured that this is the case. If the cough is annoying, alternate therapy
with an ARB may be considered. It is not necessary to evaluate electrolytes, renal function, or
pulmonary function.
Chapter 104: Hypertension
1. A 55-year-old patient has a blood pressure of 138/85 on three occasions. The patient denies
headaches, palpitations, snoring, muscle weakness, and nocturia and does not take any
medications. What will the provider do next to evaluate this patient?
c. Order urinalysis, CBC, BUN, and creatinine

This patient has prehypertension levels and should be evaluated. UA, CBC, BUN, and
creatinine help to evaluate renal function and are in the initial workup. Serum cortisol levels
are performed if pheochromocytoma is suspected, which would cause headache. The patient
does not have snoring, so a sleep study is not indicated at this time. It is not correct to
continue to monitor without assessing possible causes of early hypertension.

2. An African-American patient who is being treated with a thiazide diuretic for chronic
hypertension reports blurred vision and shortness of breath. The provider notes a blood
pressure of 185/115. What is the recommended action for this patient?
b. Admit to the hospital for evaluation and treatment.

Patients with a blood pressure >180/120 or those with signs of target organ symptoms should
be admitted to inpatient treatment with specialist consultation. Changing the medications may
be done with consultation, but a hospitalization and stabilization must be done initially.

MULTIPLE RESPONSE
1. Which are causes of secondary hypertension (HTN)? (Select all that apply.)
c. Nonsteroidal anti-inflammatory (NSAID) drugs
d. Oral contraceptives (OCPs)
e. Sleep apnea

NSAIDs and OCPs can both increase the risk of hypertension. Sleep apnea causes secondary
hypertension. Increased salt intake does not cause HTN, but those with HTN are more
sensitive to sale. Regular isometric exercise can decrease blood pressure.
Chapter 105: Infective Endocarditis
Buttaro: Primary Care: A Collaborative Practice, 6th Edition
MULTIPLE CHOICE
1. A patient who is on renal dialysis is diagnosed with infective endocarditis. What causative
organisms are more likely in this patient?
a. Enterococcal organisms
b. Neisseria gonorrhea
c. Pseudomonas aeruginosa
d. Staphylococcus aureus
ANS: D
This patient is more likely to have a health care–associated endocarditis; most of these are
caused by S. aureus. Enterococcal organisms are the second highest cause in this population.
2. A patient has been diagnosed with infective endocarditis and is being treated with empirical
antibiotics after blood cultures are inconclusive. The patient develops a severe headache along
with transient neurologic changes. What is the likely cause of these symptoms?
a. Extra-cardiac abscess formation
b. Haemophilus infection
c. Mycotic aneurysm
d. Rheumatic heart fever
ANS: C
Patients with mycotic aneurysms will present with symptoms of severe unrelenting headache,
neurological changes, and signs of cranial nerve involvement. Extracardiac abscess formation
depends on the organ involved. Haemophilus infections cause larger vegetations in the heart.
Rheumatic heart fever has a classic group of symptoms involving the skin.
3. A patient has native valve endocarditis (NVE). While blood cultures are pending, which
antibiotics will be ordered as empirical treatment?
a. A beta-lactamase-resistant penicillin and an antifungal drug
b. Imipenem-cilastatin and ampicillin
c. Penicillin G and an aminoglycoside antibiotic
d. Vancomycin and quinupristin-dalfopristin
ANS: C
The most common organism in NVE is S. aureus; until resistance is known, treatment with
penicillin G and an aminoglycoside is needed, although most strains causing NVE are not
penicillin-resistant. Antifungal infections are rare and antifungal medications are not part of
empirical therapy. Imipenem-cilastatin plus ampicillin is given for identified Enterococcus
faecalis infection. Vancomycin and quinupristin-dalfopristin is used, with limited evidence for
benefit, for Enterococcus faecium infection.
Chapter 106: Myocarditis
Buttaro: Primary Care: A Collaborative Practice, 6th Edition
MULTIPLE CHOICE
1. A previously healthy patient presents with sudden onset of dyspnea, fatigue, and orthopnea. A
family history is negative. The provider suspects myocarditis. What is the most likely etiology
for this patient?
a. Autoimmune disorder
b. Bacterial infection
c. Protozoal infection
d. Viral infection
ANS: D
Viral infection is the most common cause of myocarditis. Other infections are less likely.
Although this patient may have an autoimmune disorder, the absence of family history makes
this somewhat less likely.
2. Which test is diagnostic for diagnosing myocarditis?
a. Echocardiogram
b. Electrocardiogram
c. Endomyocardial biopsy
d. Magnetic resonance imaging
ANS: C
Endomyocardial biopsy is the only definitive test to diagnose myocarditis. Other tests are
useful in determining symptoms but are not specific to this diagnosis.
3. A patient who is an avid long-distant runner is diagnosed with viral myocarditis. What will
the provider tell this patient when asked when resuming exercising is permitted?
a. Exercise is contraindicated for life.
b. Exercise may resume when symptoms subside.
c. He may resume exercise in 6 months.
d. He must be symptom-free for 1 year.
ANS: C
Patients with myocarditis should not exercise for 6 months after the onset of symptoms.
Chapter 107: Peripheral Arterial and Venous Insufficiency
MULTIPLE CHOICE
1. An elderly adult patient without prior history of cardiovascular disease reports lower leg
soreness and fatigue when shopping or walking in the neighborhood. The primary care
provider notes decreased pedal pulses bilaterally. Which test will the provider order initially
to evaluate for peripheral arterial disease based on these symptoms?
a. Digital subtraction angiography
b. Doppler ankle, arm index
c. Magnetic resonance angiography
d. Segmental limb pressure measurement
ANS: B
The Doppler study may be performed easily to indicate the likelihood of PAD. Other tests are
performed only if indicated.
2. A 75-year-old patient reports pain and a feeling of tiredness in both legs that only relieves
after sitting for 30 minutes or more. What the does provider suspect as the cause for these
symptoms?
a. Buerger’s disease
b. Cauda equina syndrome
c. Diabetic neuropathy
d. Peripheral arterial disease (PAD)
ANS: B
Patients with cauda equina syndrome, which causes spinal stenosis, will often not get relief
until they sit down for a period of time. Buerger’s disease involves both the upper and lower
extremities. Diabetic neuropathy may mask pain. PAD involves these symptoms that stop
with rest.
3. A patient is diagnosed with peripheral arterial disease (PAD) and elects not to have
angioplasty after an angiogram reveals partial obstruction in lower extremity arteries. What
will the provider recommend to help manage this patient’s symptoms?
a. Daily aspirin therapy to prevent clotting
b. Statin therapy with clopidogrel
c. Walking slowly for 15 to 20 minutes twice daily
d. Walking to the point of pain each day
ANS: D
Studies have demonstrated that an exercise program involving walking to the point of pain is
as effective as angioplasty. Medications are useful to prevent progression of plaque formation
and to prevent myocardial infarction (MI).
Chapter 108: Valvular Heart Disease and Cardiac Murmurs
1. A patient has a cardiac murmur that peaks in mid-systole and is best heard along the left
sternal border. The provider determines that the murmur decreases in intensity when the
patient changes from standing to squatting and increases in intensity with the Valsalva
maneuver. Which will the provider suspect is causing this murmur?
a. Aortic stenosis
b. Hypertrophic cardiomyopathy
c. Mitral valve prolapse
d. Tricuspid regurgitation
ANS: B
These findings occur with hypertrophic cardiomyopathy. With aortic stenosis, the murmur is a
harsh crescendo-decrescendo heard best at the right sternal border that decreases in intensity
with the Valsalva maneuver. With mitral valve prolapse, the murmur is heard in mid- to late
systole, is heard best at the left lower sternal border, and may have a click that moves to later
systole or disappear with the Valsalva maneuver. With tricuspid regurgitation, the murmur
may occur at early, mid, or late systole, is heard at the left lower sternal border, and decreases
with the Valsalva maneuver.
2. A young adult patient is diagnosed with a mitral valve prolapse. During a routine 3-year
health maintenance exam, the provider notes an apical systolic murmur and a mid-systolic
click on auscultation. The patient denies chest pain, syncope, or palpitations. What action will
the provider take?
a. Admit the patient to the hospital for evaluation and treatment.
b. Consult with the cardiologist to determine appropriate diagnostic tests.
c. Continue to monitor the patient every 3 years.
d. Reassure the patient that these findings are expected.
ANS: B
Most patients with mitral valve prolapse are monitored every 3 years unless they have a
systolic murmur. The provider should consult with the cardiologist. Hospital admission is not
necessary since the patient is asymptomatic.
MULTIPLE RESPONSE
1. Which are factors that can cause a heart murmur? (Select all that apply.)
a. Backward flow through a septal defect
b. Backward flow into a normal vessel
c. Forward flow into a dilated vessel
d. High rates of flow through a normal valve
e. Low rates of flow into a cardiac chamber
ANS: A, C, D
High rates of flow into either normal or abnormal vessels can cause murmurs. Backward flow
into septal defects, regurgitant valves, or PDAs can cause murmurs. Forward flow into
constricted or irregular valves or into a dilated vessel can cause murmur. Backward flow into
a normal vessel and low flow rates are not responsible for murmurs.
Chapter 109: Abdominal Pain and Infections

1. An adult patient reports intermittent, crampy abdominal pain with vomiting. The provider
notes marked abdominal distention and hyperactive bowel sounds. What will the provider do
initially?
b. Obtain upright and supine radiologic views of the abdomen

If available, the primary care provider can order radiographic studies of the abdomen and
chest. Once small bowel obstruction is confirmed or suspected, immediate hospitalization
with surgeon referral is necessary. Because small bowel obstruction can have potentially
serious or life-threatening consequences, waiting 24 hours is not recommended.

2. A patient is in clinic for evaluation of sudden onset of abdominal pain. The provider palpates
a pulsatile, painful mass between the xiphoid process and the umbilicus. What is the initial
action?
a. Order a CBC, type and crossmatch, electrolytes, and renal function tests.
b. Perform an ultrasound examination to evaluate the cause.
c. Schedule the patient for an aortic angiogram.
d. Transfer the patient to the emergency department for a surgical consult.

This patient has symptoms and physical findings consistent with a ruptured aortic aneurysm
and should have an immediate surgical consult. Ordering other tests is not necessary by the
primary provider.
MULTIPLE RESPONSE
1. Which symptoms noted in a patient reporting abdominal pain are suggestive of appendicitis?
a. Abdominal rigidity along with pain
b. Pain accompanied by low-grade fever
c. Pain occurring prior to nausea and vomiting
d. Pain that begins in the left lower quadrant
e. Prolonged duration of right lower quadrant pain
ANS: A, B, C
Patients with appendicitis typically have pain that begins in the epigastric or periumbilical
area and migrates to the left lower quadrant. Abdominal rigidity is common, as is low-grade
fever. Pain precedes other symptoms and when the symptoms occur in any other order, the
diagnosis of appendicitis should be questioned. Pain is usually of short duration.
Chapter 110: Anorectal Complaints

1. A patient reports anal pruritis and occasional bleeding with defecation. An examination of the
perianal area reveals external hemorrhoids around the anal orifice as the patient is bearing
down. The provider orders a colonoscopy to further evaluate this patient. What is the
treatment for this patient’s symptoms?
a. A high-fiber diet and increased fluid intake
b. Daily laxatives to prevent straining with stools
c. Infiltration of a local anesthetic into the hemorrhoid
d. Referral for possible surgical intervention
ANS: A
Most hemorrhoids, unless incarcerated or painful, are treated conservatively. A high-fiber diet
and increased fluid intake are recommended first. Daily laxatives are not recommended
because the variation in stool consistency makes hemorrhoid management more difficult.
Infiltration of a local anesthetic is performed for thrombosed external hemorrhoid prior to
removing the clot. Hemorrhoidectomy is performed for severe or very painful hemorrhoids.

MULTIPLE RESPONSE
1. What recommendations are appropriate for patients with chronic pruritus ani?
a. Application of a topical antihistamine
b. Applying a of 1% hydrocortisone cream for several months
c. Avoid tight-fitting or non-breathable clothing
d. Avoiding perfumed soaps and toilet papers
e. Using a hair dryer on the cool setting to control itching

Measures to control itching include avoiding tight-fitting clothing as well as perfumed


products and keeping the area clean and dry and using a cool hair dryer to dry the skin.
Topical antihistamines are not used. Using a topical steroid longer than 2 weeks causes
thinning of the skin.
Chapter 111: Cholelithiasis and Cholecystitis

1. A patient has sudden onset of right upper quadrant (URQ) and epigastric abdominal pain with
fever, nausea, and vomiting. The emergency department provider notes yellowing of the
sclerae. What is the probable cause of these findings?
a. Acute acalculous cholecystitis
b. Chronic cholelithiasis
c. Common bile duct obstruction
d. Infectious cholecystitis
ANS: C
This patient has symptoms of cholecystitis with bile duct obstruction, which causes jaundice.
The common triad of RUQ pain, fever, and jaundice occurs when a stone is lodged in the
common bile duct. Acute acalculous cholecystitis is inflammation without stones. Chronic
cholelithiasis does not cause acute symptoms; jaundice occurs with obstruction. Infectious
cholecystitis may occur without obstruction.

2. A patient presents with fever, nausea, vomiting, anorexia, and right upper quadrant abdominal
pain. An ultrasound is negative for gallstones. Which action is necessary to treat this patient’s
symptoms?
a. Empirical treatment with antibiotics
b. Hospitalization for emergent treatment
c. Prescribing ursodeoxycholic acid
d. Supportive care with close follow-up
ANS: B
This patient has symptoms of acute acalculous cholecystitis and is critically ill.
Hospitalization is required. Empirical treatment with antibiotics and supportive care with
follow-up do not address critical care needs. Ursodeoxycholic acid is a medication that helps
with gallstone dissolution; this patient does not have gallstones.

3. Which diagnostic test will the provider safely order for a 30-year-old woman reporting right
upper quadrant abdominal pain, nausea, and vomiting?
a. Abdominal computed tomography (CT) with contrast
b. Abdominal ultrasound

Women of childbearing age may safely have ultrasound. Until pregnancy is ruled out, the
other studies may be harmful to a developing fetus and should be avoided.
4. A patient with a previous history of liver disease is diagnosed with a bile duct obstruction.
Which procedure will be prescribed for this patient?
a. Chemical dissolution of the gallstone
b. Lithotripsy
c. Open cholecystectomy
d. Laparoscopic cholecystectomy
ANS: C
Patients with possible liver disease should have open cholecystectomy. The other procedures
are contraindicated. Chemical dissolution is not reliable and may take some time.
Chapter 112: Cirrhosis

1. A patient is diagnosed with fibrotic liver disease; a liver biopsy shows micronodular cirrhosis.
What is the most common cause of this form of cirrhosis?
a. Alcoholism

Micronodular cirrhosis is often associated with alcoholic liver disease. Viral causes and
carcinoma usually cause macronodular cirrhosis. Right-sided heart failure occurs with many
other causes as part of the disease development.

2. A patient with a history of chronic alcoholism reports weight loss, pruritis, and fatigue. The
patient’s urine and stools appear normal. What do these findings indicate?
a. Early liver cirrhosis

Early symptoms of cirrhosis are characterized by this patient’s symptoms. As the condition
worsens, stools and urine change color and the patient develops anorexia, nausea, and
vomiting. Liver failure and ascites are late and will include abdominal pain. Viral hepatitis is a
less likely diagnosis in the patient with a history of alcoholism.

3. A patient diagnosed with cirrhosis develops ascites. Which medication will be ordered
initially to improve symptoms?
d. Spironolactone

Spironolactone is the initial diuretic used to improve fluid diuresis in patients with ascites.
Furosemide may be used as adjunctive therapy. Cephalosporin is used when infections occur.
Lactulose is used to increase stools and reduce encephalopathy.
Chapter 113: Constipation

1. A patient diagnosed with chronic constipation uses polyethylene glycol and reports increased
abdominal discomfort with nausea and vomiting. What is the initial action by the provider?
b. Obtain radiographic abdominal studies

Patients with abdominal pain, nausea, and vomiting should have radiologic studies to exclude
obstruction, ileus, megacolon, or volvulus. If those are ruled out, increasing the laxative may
be warranted. Stool culture is indicated if the parasite ascariasis is suspected. Referral for
colonoscopy is needed if alarm symptoms for neoplasm are present.

2. A patient has recurrent constipation which improves with laxative use but returns when
laxatives are discontinued. Which pharmacologic treatment will the provider recommend for
long-term management?
c. Methylcellulose

Methylcellulose is a bulk-forming product and is used initially. The other medications are
used for more severe constipation and not recommended for long-term use. Mineral oil, an
emollient, will soften stool, but it has been associated with aspiration and lipoid pneumonia,
prevents absorption of fat-soluble vitamins, and can cause fecal incontinence; it is not
generally recommended.

MULTIPLE RESPONSE
1. A patient reports a decrease in the frequency of stools and asks about treatment for
constipation. Which findings are part of the Rome IV criteria for diagnosing constipation?
a. Feeling of incomplete evacuation
c. Hard or lumpy stools
e. Symptoms present for 3 months

According to the Rome III criteria, symptoms must have begun 6 months prior and persisted
for at least 3 months and include a feeling of incomplete evacuation, lumpy or hard stools,
fewer than 3 stools per week, and not meeting criteria for irritable bowel syndrome.
Chapter 114: Diarrhea, Noninfectious

1. A patient, who first developed acute diarrhea 2 weeks ago, presents to clinic reporting profuse
watery, bloody diarrheal stools 6 to 8 times daily. The provider notes a toxic appearance with
moderate dehydration. Which test is indicated to diagnose this problem?
c. Stool sample for C. difficile toxin

Patients with acute onset diarrhea lasting more than 2 weeks with profuse, watery, bloody
stools of more than 6 times in a 24-hour period warrants testing for C. difficile toxin.
Qualitative and quantitative fecal fat, 24-hour pH studies, and Wright stain for WBCs are
performed when chronic diarrhea are present.

2. A patient who developed chronic diarrhea after gastric surgery asks what can be done to
mitigate symptoms. What will the provider recommend initially?
b. Avoiding liquids with meals

Initial suggestions for treating postoperative diarrhea will include avoiding fluids during
meals and lying down after meals. Concentrated carbohydrates may trigger symptoms.
Empirical antibiotic therapy is indicated for small intestinal bacterial overgrowth syndrome
with specific symptoms and an association with an elevated folate level. Probiotic
supplements may be used as adjunctive therapy.
MULTIPLE RESPONSE
1. Which types of chronic noninfectious diarrhea will cause fatty stools? (Select all that apply.)
a. Celiac disease
b. Cystic fibrosis
e. Pancreatic insufficiency

Celiac disease, cystic fibrosis, and pancreatic insufficiency all produce malabsorption of fats
and will result in fatty stools. Diabetes results in glucose malabsorption, while lactose
intolerance causes lactose malabsorption.
Chapter 115: Diverticular Disease

1. A patient with a history of diverticular disease asks what can be done to minimize acute
symptoms. What will the provider recommend to this patient?
b. Consuming a diet high in fiber

Increasing dietary fiber reduces constipation and reduces the incidence of acute symptoms.
Avoiding saturated fats and red meats does not reduce the risk of diverticulitis but does
decrease the risk of colon cancer. Anticholinergics and antispasmodics do not prevent attacks
but may help with symptoms. Bran may be used as an adjunct to high-fiber foods but should
not replace other high-fiber sources.

2. A patient with a history of diverticular disease experiences left-sided pain and reports seeing
blood in the stool. What is an important intervention for these symptoms?
c. Referring the patient for a lower endoscopy

Patients with suspected diverticular abscess of rectal bleeding need further evaluation and a
referral for lower endoscopy is warranted. Hemorrhage is more common from the right colon.
The other actions do not have priority over the need to evaluate the cause of the bleeding.
MULTIPLE RESPONSE
1. A patient has intermittent left-sided lower abdominal pain and fever associated with bloating
and constipation alternating with diarrhea. The provider suspects acute diverticulitis. Which
tests will the provider order? (Select all that apply.)
b. Computerized tomography (CT) scan of abdomen and pelvis
e. Stool for occult blood

For symptomatic diverticulosis, the diagnosis of diverticulosis or segmental colitis (as with
SCAD) can be established by direct view on colonoscopy or flexible sigmoidoscopy. A CT
scan of the abdomen can also diagnose diverticulosis. A barium or water-soluble enema
should not be utilized if acute diverticulitis is suspected. Plain abdominal x-ray films will be
normal and are unnecessary, although they are sometimes ordered to exclude the presence of
free air in the abdomen.
Chapter 116: Oropharyngeal Dysphagia in Adults

1. An older adult patient has recently experienced weight loss. The patient’s spouse reports
noticing coughing and choking when eating. What is the likely cause of this presentation?
c. Pharyngeal dysphagia

Pharyngeal dysphagia often results from weakness or poor coordination of the pharyngeal
muscles which can cause delayed swallow and failure of airway protection, leading to
coughing and choking. Esophageal dysphagia is associated with pain after swallowing. Oral
stage disorders are related to poor bolus control and result in drooling or spilling. Xerostomia
is when oral mucous membranes are dry.

2. Which diagnostic study is best to evaluate a swallowing disorder?


d. Videofluoroscopy (VFES)

Videofluoroscopy is the most appropriate because it visualizes the actual swallow.


Electroglottography and electron microscopy may be appropriate but are more limited. CT
evaluation may aid in diagnosis but does not describe the actual swallow mechanism.

3. A patient experiences a feeding disorder after a stroke that causes disordered tongue function
and impaired laryngeal closure. What intervention will be helpful to reduce complications in
this patient?
c. Thickened liquids

Thickening liquids is helpful for patients with disordered tongue function and impaired
laryngeal closure, because there is a reduced tendency for liquids to spill over the tongue base
and cause aspiration. Surface electrical stimulation helps improve strength of muscles but
does not address the problem of aspiration. Teaching head rotation is used for patients with
unilateral laryngeal dysfunction. Thinning liquids is used for patients with weak pharyngeal
contraction.
Chapter 117: Gastroesophageal Reflux Disease

1. A patient experiences a sharp pain just under the sternum with swallowing. This is more
commonly associated with which condition?
b. Infectious esophagitis

A sharp, substernal pain with swallowing is most commonly associated with infectious
esophagitis. Esophageal strictures are highly correlated with hiatal hernia and patients with
stricture will report a feeling of food becoming stuck. A Schatzki ring and peptic stricture are
types of strictures.
MULTIPLE RESPONSE
1. Which medications may cause the greatest increase in the prevalence of gastroesophageal
reflux disease (GERD)? (Select all that apply.)
a. Aspirin
b. Benzodiazepines
c. Calcium antagonists
d. Hormone replacements
e. Oral contraceptives
ANS: A, B, C
Aspirin, benzodiazepines, and calcium antagonists all increase the likelihood of GERD, while
hormone replacement therapy and OCPs are associated with a lower incidence.

Chapter 118: Gastrointestinal Hemorrhage


Buttaro: Primary Care: A Collaborative Practice, 6th Edition
MULTIPLE CHOICE
1. A 50-year-old, previously healthy patient has developed gastritis. What is the most likely
cause of this condition?
a. H. pylori infection

H. pylori accounts for most cases such as gastritis, duodenal ulcers, and gastric ulcers. NSAID
use is an important cause, but not likely in a previously healthy individual. Parasites are the
leading cause worldwide, but not in the United States. Viral gastroenteritis usually does not
cause chronic gastritis and usually has lower GI symptoms.

2. A patient has both occasional “coffee ground” emesis and melena stools. What is the most
probably source of bleeding in this patient?
d. Upper gastrointestinal (GI) tract

Coffee ground emesis is usually old blood from an upper GI source and melena is black,
shiny, foul-smelling as a result of blood degradation and is usually upper GI in origin. Lower
GI and rectal bleeding will cause bright red blood in stools. Hepatic bleeding usually does not
affect the GI tract.

3. What initial action is appropriate when admitting a patient who has a gastrointestinal (GI)
tract bleed, hypotension, and a hematocrit decrease of 6% from baseline?
c. Place two large-bore intravenous lines.
Chapter 119: Hepatitis

1. A patient who is asymptomatic tests positive for the hepatitis C virus (HVC). What will the
provider tell the patient about managing this illness?
c. Several medications are available based on the type of hepatitis C.

The provider should inform the patient that there are several medications available based on
the type of hepatitis C the patient has. HCV rarely has a rapidly fulminant course, although
cirrhosis is likely after years of infection. Immunoglobulin therapy is given for HBV. The
disease is not self-limiting.

2. Which form of hepatitis virus is rapidly spread via the fecal-oral route?
a. Hepatitis A

HAV is rapidly spread, usually through contaminated food, through the fecal-oral route. The
other types have a parenteral transmission via blood and other body fluids.

3. A patient recovering from chronic alcohol abuse reports nausea, vomiting, diarrhea, and
abdominal discomfort. A physical examination is negative for jaundice or ascites. What will
the provider do initially?
b. Order a complete blood count and liver function tests

Patients with alcoholic hepatitis may present initially with signs of gastroenteritis. Based on
the history, even without jaundice and ascites, the provider should order a CBC and LFTs.
Bilirubin and PT levels are performed when a diagnosis is made to determine prognosis and
course of the disease. Reassuring the patient without confirmation of disease is not
recommended. Referral is made if hepatitis is diagnosed.

Chapter 120: Inflammatory Bowel Disease


1. A patient reports lower abdominal cramping and occasional blood in stools. The provider
suspects inflammatory bowel disease. Which test will the provider order to determine whether
the patient has ulcerative colitis (UC) or Crohn’s disease (CD)?
b. Colonoscopy

Colonoscopy is useful in differentiating UC from CD. Barium enema has limited use in
diagnosis, but is used to detect distension, strictures, tumors, fistulas, or obstructions. Genetic
testing may be helpful in the future with further advances. Small bowel series are used
infrequently to determine small bowel involvement.

2. A patient is diagnosed with mild to moderate ulcerative colitis. Which medication will be
prescribed initially to establish remission?
d. Sulfasalazine
Sulfasalazine is a 5-aminosalicyclic acid used to induce remission in UC and is a first-line
medication. Budesonide is a synthetic corticosteroid used for moderate to severe disease, but
not as a first-line agent. Azathioprine is an immunomodulator used to minimize the need for
corticosteroids. Infliximab is a biologic medication and is more useful for treating Crohn’s
disease.
MULTIPLE RESPONSE
1. Which are characteristics of Crohn’s disease (CD)? (Select all that apply.)
a. Fistulous tracts may occur as disease complications.
c. Inflammation affects all layers of the intestinal tract wall.
d. The disease may be limited to the small intestine.

CD may be complicated by fistulous tracts. Inflammation affects all layers of the intestinal
wall tract. The disease may be limited to the small intestine. UC causes inflammation that is
diffuse and continuous and about 50% of patients with UC may never have significant
remission of symptoms.

Chapter 121: Irritable Bowel Syndrome

1. What is the probable underlying pathology of irritable bowel syndrome (IBS), according to
research over the last decade?
a. Alteration in processing of sensory information

Recent research has yielded information about alterations in sensory processing that are
different in persons with IBS. Changes in intestinal mucosa, intestinal tissue disease, and
malabsorption syndromes are structural disorders and this is a functional disease.

2. Which symptom must be present for a diagnosis of irritable bowel syndrome (IBS)?
a. Abdominal pain

3. A patient has irritable bowel syndrome (IBS) with alternating diarrhea and constipation and
asks the provider about dietary changes that may help with symptoms. What will the provider
recommend?
d. Keeping a food and symptom diary

Because all patients with IBS are different and there are no specific foods that cause
symptoms, each patient should keep a diary to determine which foods may trigger symptoms
before adding or eliminating foods.
Chapter 122: Jaundice

1. A patient has an elevated indirect bilirubin. Which condition may be causing this symptom?
c. Hemolytic anemia

Indirect or unconjugated bilirubin is often associated with an increase in the destruction of


RBCs, as with hemolytic anemia. Direct or conjugated bilirubin is elevated when there is liver
dysfunction or obstruction.

2. A patient diagnosed with jaundice has bright orange urine. What is a likely cause of this
jaundice?
a. Bile duct obstruction

Conjugated bilirubin, which is in excess with liver disease, is excreted in the urine, causing a
characteristic orange color. Unconjugated bilirubin is elevated with increased destruction of
RBCs, which occurs with transfusion reactions, defective erythropoiesis, and sickle cell
anemia.
MULTIPLE RESPONSE
1. A patient presenting with jaundice has a bilirubin testing that reveals elevated direct bilirubin.
Which subsequent testing may help determine the cause of these findings?
b. Liver function tests
d. Serologic viral tests
e. Serum iron and ferritin

Since the direct bilirubin is elevated, hepatic causes should be evaluated. These tests will
include liver function tests, viral tests for hepatitis, and serum iron and ferritin. CBC and renal
function tests evaluate the presence of hemolytic disease.

Chapter 123: Nausea and Vomiting

1. A patient has a recent episode of vomiting and describes the vomitus as containing mostly
gastric juice. What does this symptom suggest?
c. Peptic ulcer

The vomitus with peptic ulcer disease contains mostly gastric juice. Bile duct obstruction will
result in bilious vomitus. Gastritis vomitus contains blood and will have a coffee-ground
appearance. Small bowel obstruction produces vomitus that is feculent.

2. A patient has nausea associated with chemotherapy. Which agent will be prescribed to
manage this side effect?
c. Ondansetron

Ondansetron is used to treat chemotherapy-induced nausea and vomiting. The other


medications are used for nausea associated with motion sickness, migraines, and vertigo.
Chapter 124: Pancreatitis

1. Which is the most common cause of pancreatitis in the United States?


b. Gallstones

2. A patient reports a sudden onset of constant, sharp abdominal pain radiating to the back. The
examiner notes both direct and rebound tenderness with palpation of the abdomen. What is the
significance of this finding?
d. Severe acute pancreatitis with peritonitis

Direct and rebound tenderness is an ominous sign suggesting severe peritonitis. Jaundice is
present with compression of the common bile duct. Palpation of a mass suggests the presence
of a pancreatic pseudocyst. Bruising of the periumbilicus or flank suggests retroperitoneal
hemorrhage.

3. The provider suspects that a patient has chronic pancreatitis. Which diagnostic tests will be
most helpful to confirm this diagnosis?
a. Blood glucose and fecal fat

Patients with pancreatic insufficiency will have elevated blood glucose levels and steatorrhea.
The CBC, LFTs, and serum amylase and lipase are typically normal with chronic pancreatitis.

Chapter 125: Tumors of the Gastrointestinal Tract

1. A patient with a history of esophageal reflux reports difficulty swallowing. The provider notes
fixed cervical and axillary lymphadenopathy on exam. What is the significance of these
findings if esophageal carcinoma is suspected?
c. The prognosis for cure is poor.

Supraclavicular, cervical, and axillary lymphadenopathy are signs of advanced disease and
suggestive of metastatic disease. Hepatomegaly and superior vena cava syndrome indicate a
poor prognosis. Esophageal cancer usually has a high mortality rate.

2. A patient is diagnosed with gastric cancer after presenting with cachexia, small bowel
obstruction, hepatomegaly, and ascites. What will the provider tell this patient about treatment
and possible cure?
c. Palliative resection may be performed.

This patient presented with signs of advanced disease, which has a poor prognosis. Palliative
resection may be performed. Curative treatment involves surgery, chemotherapy, and
radiation. Chemotherapy is not the only option and is usually combined with other therapies.
Chemotherapy is preferred for metastatic disease.

3. A patient is diagnosed with cancer of the colon and is scheduled for surgical resection. A
carcinoembryonic antigen (CEA) test prior to surgery is not elevated. What is the significance
of this finding?
c. The test is not informative and will not be repeated.
Chapter 126: Peptic Ulcer Disease
MULTIPLE CHOICE
1. A patient has persistent epigastric pain occurring 2 to 3 hours after a meal. Which test is
definitive for diagnosis peptic ulcer disease (PUD) in this patient?
c. Endoscopy with biopsy of gastric mucosa

Endoscopy provides the most accurate diagnosis of PUD and allows biopsy of multiple areas
to exclude malignancy. Barium swallow may still be performed in patients unwilling to
undergo endoscopy. Breath tests and stool antigen testing for H. pylori can confirm a bacterial
cause. Physical exam generally yields negative findings.

2. A patient who has been taking an NSAID for osteoarthritis pain has been diagnoses with
peptic ulcer disease (PUD). What is the initial step in treating this patient?
a. Discontinue the NSAID.

The first step in treating medication-induced peptic ulcer is to discontinue the medication. H2
receptor antagonists are the first antisecretory medications prescribed. Proton pump inhibitors
are more expensive and are used as second-line treatment. Prostaglandin therapy helps protect
the gastric and duodenal mucosa and is used if NSAIDS cannot be discontinued.

3. Which of the following is the American College of Gastroenterology treatment


recommendation for H. pylori-related peptic ulcer disease?
d. Proton pump inhibitor (PPI), amoxicillin, and clarithromycin for 10 days

The American College of Gastroenterology (ACG) guideline recommendations include a PPI


plus clarithromycin 500 mg po twice a day and amoxicillin 1 gram po daily for 7 to 14 days or
a PPI plus clarithromycin 500 mg po twice a day and metronidazole 500 mg po twice a day
for up to 14 days.
Chapter 127: Incontinence
Buttaro: Primary Care: A Collaborative Practice, 6th Edition
MULTIPLE CHOICE
1. The daughter of an elderly confused patient reports that her parent is having urinary
incontinence several times each day. What will the provider do initially?
a. Obtain a urine sample for urinalysis (UA) and possible culture
b. Order serum creatinine and blood urea nitrogen tests
c. Perform a bladder scan to determine distention and retention
d. Tell the daughter that this is expected given her mother’s age and confusion

When incontinence occurs, UA is performed initially to exclude hematuria, pyuria, glucosuria,


or proteinuria and possible infection. Serum creatinine and BUN may be performed if renal
disease is suspected. Bladder scans may be performed if the UA is normal to evaluate
physiologic causes. It is not correct to offer reassurance without ruling out other causes.

2. The provider is evaluating a patient for potential causes of urinary incontinence and performs
a postvoid residual (PVR) test which yields 30 mL of urine. What is the interpretation of this
result?
a. The patient may have overflow incontinence.
b. The patient probably has a urinary tract infection (UTI).
c. This is a normal result.
d. This represents incomplete emptying.

A PVR less than 50 mL is considered normal and this result does not indicate any
abnormality.

3. The provider is counseling a patient who has stress incontinence about ways to minimize
accidents. What will the provider suggest initially?
a. Increasing fluid intake to dilute the urine
b. Referral to a physical therapist
c. Taking pseudoephedrine daily
d. Voiding every 2 hours during the day

Timed voiding is useful to help minimize stress incontinence and is used initially. Increasing
fluid intake will increase symptoms. PT referral may be done if other measures fail to help
with exercises to strengthen the pelvic floor muscles. Pseudoephedrine is useful, but not an
initial therapy.
Chapter 128: Prostate Cancer
Buttaro: Primary Care: A Collaborative Practice, 6th Edition
MULTIPLE CHOICE
1. An older male patient reports urinary frequency, back pain, and nocturia. A dipstick urinalysis
reveals hematuria. What will the provider do next to evaluate this condition?
a. Order a PSA and perform a digital rectal exam (DRE)
b. Refer for a biopsy
c. Refer the patient to a urologist
d. Schedule a transurethral ultrasound (TRUS)

Patients with symptoms of potential prostate cancer should be screened with PSA and DRE.
Referral to a urologist is the next step even with normal findings, since PSA is occasionally
normal. The urologist may order TRUS or biopsy.

2. An older male patient has a screening prostate-specific antigen (PSA) which is 12 ng/mL.
What does this value indicate?
a. A normal result
b. Benign prostatic hypertrophy
c. Early prostate cancer
d. Prostate cancer
ANS: D
A PSA greater than 10 ng/mL suggests prostate cancer. A level between 4 and 10 ng/mL may
be early prostate cancer or a benign condition. A level less than 4 ng/mL is normal.

3. A patient is diagnosed with prostate cancer and diagnostic testing reveals disease that has
gone past the prostatic capsule without evidence of metastasis. The patient does not wish to
undergo treatment. What will the provider tell this patient?
a. Chemotherapy is indicated to provide cure for this cancer.
b. Monitoring prostate-specific antigen (PSA) with regular digital rectal examination
(DRE) is an acceptable option.
c. Palliative radiation therapy is necessary to improve quality of life.
d. This level of disease requires intervention with hormonal therapy.

This patient has stage T2 prostate cancer which may be managed with watchful waiting which
includes PSA and DRE evaluation. Chemotherapy, palliative radiation therapy, and hormonal
therapy are not required.
Chapter 129: Prostatic Hyperplasia (Benign)
Buttaro: Primary Care: A Collaborative Practice, 6th Edition
MULTIPLE CHOICE
1. A male patient reports nocturia and daytime urinary frequency and urgency without changes
in the force of the urine stream. What is the likely cause of this?
a. Bladder outlet obstruction
b. Lower urinary tract symptoms (LUTS)
c. Prostate cancer
d. Urinary tract infection (UTI)

Lower urinary tract symptoms (LUTS) result from irritative changes in the lower tract.
Bladder outlet obstruction causes hesitancy, decreased caliber and force of the urine stream,
and postvoid dribbling. Diagnosis of prostate cancer and UTI require further testing and are
less likely causes.

2. A 70-year-old male reports urinary hesitancy, postvoid dribbling, and a diminished urine
stream. A digital rectal exam (DRE) reveals an enlarged prostate gland that feels rubbery and
smooth. Which tests will the primary care provider order based on these findings?
a. Bladder scan for postvoid residual
b. Prostate-specific antigen (PSA) and bladder imaging
c. Urinalysis and serum creatinine
d. Urine culture and CBC with differential

The DRE reveals a prostate gland consistent with benign prostatic hyperplasia (BPH). The
primary provider should order a urinalysis and creatinine to evaluate possible infection and
renal function. A bladder scan is ordered at the discretion of the urologist. The prostate exam
isn’t consistent with prostate cancer, so PSA and bladder imaging are not necessary.
Symptoms of prostatitis would indicate a need for evaluation of possible infection.

3. A patient has been taking terazosin daily at bedtime to treat benign prostatic hyperplasia
(BPH) and reports persistent daytime dizziness. What will the provider do?
a. Prescribe finasteride instead of terazosin
b. Recommend taking the medication in the morning
c. Suggest using herbal preparations
d. Switch the prescription to doxazosin

Patients who cannot tolerate the side effect of alpha-adrenergic antagonists, the provider may
initiate therapy with a 5a-reductase inhibitor such as finasteride. Terazosin should be given at
bedtime to minimize these adverse effects. Herbal preparations have not been proven to be
safe or effective. Doxazosin is in the same drug class as terazosin.
Chapter 130: Proteinura and Hematuria
Buttaro: Primary Care: A Collaborative Practice, 6th Edition
MULTIPLE CHOICE
1. A pregnant woman at 30 weeks gestation presents with proteinuria. What will the provider do
next?
a. Evaluate her blood pressure and discuss with OB/GYN
b. Monitor serum glucose for gestational diabetes
c. Perform a 24-hour urine collection
d. Reassure her that this normal at this stage of pregnancy
ANS: A
Proteinuria after 24 weeks gestation is usually a sign of preeclampsia, so her blood pressure
should be evaluated and discussed with the OB/GYN. Serum glucose evaluation for
gestational diabetes is performed as part of routine screening but is not related to the finding
of proteinuria. A 24-hour urine collection is not indicated.
2. An older male patient reports gross hematuria but denies flank pain and fever. What will the
provider do to manage this patient?
a. Monitor blood pressure closely
b. Obtain a urine culture
c. Perform a 24-hour urine collection
d. Refer for cystoscopy and imaging
ANS: D
Gross hematuria in older men denotes a significant risk of malignant disease, so cystoscopy
and imaging are indicated. Proteinuria is concerning for hypertension. The patient does not
have flank pain or fever, so the likelihood of infection is lower. A 24-hour urine collection is
not indicated.
3. A female patient reports hematuria and a urine dipstick and culture indicate a urinary tract
infection. After treatment for the urinary tract infection (UTI), what testing is indicated for
this patient?
a. 24-hour urine collection to evaluate for glomerulonephritis
b. Bladder scan
c. Repeat urinalysis
d. Voiding cystourethrogram
ANS: C
After treatment has been completed, repeated urinalysis is necessary to ensure that the
hematuria has resolved. Failure to follow hematuria to resolution may result in failure to
diagnose a serious condition.
Chapter 131: Renal Failure
Buttaro: Primary Care: A Collaborative Practice, 6th Edition
MULTIPLE CHOICE
1. Which is a prerenal cause of acute kidney injury (AKI)?
a. Hemorrhagic shock
b. Hydronephrosis
c. Hypertension
d. Renal calculi
ANS: A
Hemorrhagic shock interferes with perfusion of the kidney, which is a prerenal cause of AKI.
Hydronephrosis and renal calculi are postrenal causes leading to obstruction to renal pelvis,
ureters, bladder, or urethra. Hypertension is an intrinsic cause.
2. A primary care provider sees a new patient who reports having a diagnosis of chronic kidney
disease for several years. The patient is taking one medication for hypertension which has
been prescribed since the diagnosis was made. The provider orders laboratory tests to evaluate
the status of this patient. Which laboratory finding indicates a need to refer the patient to a
nephrologist?
a. Albumin/creatinine ratio (ACR) of 325 mg/g
b. Blood pressure of 145/85 mm Hg
c. Glomerular filtration rate (eGFR) of 35
d. Urine red blood cell (RBC) count of 15/hpf
ANS: A
An albumin/creatinine ratio greater than 300 mg/g warrants referral. A specialist is necessary
for persistent hypertension refractory to treatment with four or more agents, a GFR of less
than 30, and urine RBC greater than 20/hpf.
MULTIPLE RESPONSE
1. Which tests should be monitored regularly to monitor for complications of chronic renal
disease (CRD)? (Select all that apply.)
a. Liver enzymes
b. Parathyroid hormone levels
c. Serum glucose
d. Serum lipids
e. Vitamin D levels
ANS: B, D, E
CKD can cause hyperparathyroidism, hyperlipidemia, and alterations in vitamin D, calcium,
and phosphorus metabolism, so these should be monitored. Liver function and serum glucose
are not affected by CKD.
Chapter 132: Sexual Dysfunction (Male)
Buttaro: Primary Care: A Collaborative Practice, 6th Edition
MULTIPLE CHOICE
1. Which is true about hypoactive sexual desire in older men?
a. Hypoactive sexual desire in older men is related to sexual aversion.
b. Hypoactive sexual desire is a conscious choice to avoid sexual relations.
c. Men with hypoactive sexual desire may have normal excitement and orgasm.
d. The most common type of sexual dysfunction is hypoactive sexual desire.
ANS: C
Men with hypoactive sexual desire have diminished response in the desire phase of the sexual
response cycle but may still experience normal excitement and orgasm. Sexual aversion and
hypoactive desire are not related. Many people with normal sexual desires choose not to have
sexual relations; hypoactive desire is a physiological condition. Only 16% of men have
hypoactive desire.
2. A 50-year-old man reports having erectile dysfunction (ED). What is an important response
by the provider when developing a plan of care for this patient?
a. Considering testosterone hormone replacement therapy
b. Evaluating the patient for cardiovascular disease
c. Prescribing an oral phosphodiesterase type 5 inhibitor
d. Referring the patient for psychotherapy and counseling
ANS: B
Men under age 60 years with ED are at higher risk for cardiovascular disease, so this patient
should be evaluated for this condition. Until the underlying cause is found, prescribing
medications or hormones is not indicated. Psychotherapy and counseling are used when
psychogenic ED is present.
MULTIPLE RESPONSE
1. The provider prescribes the oral phosphodiesterase type 5 inhibitor sildenafil to treat erectile
dysfunction (ED) in a 65-year-old male patient. What will be included when teaching this
patient about taking this medication? (Select all that apply.)
a. The medication is best taken on an empty stomach.
b. The medication should be taken with a fatty food or meal.
c. The medication’s effects may last for 24 to 36 hours.
d. This medication has a rapid onset and short duration of action.
e. This medication may be taken once daily.
ANS: A, D
Sildenafil has a rapid onset and short duration of action and should be taken on an empty
stomach. Fatty foods may delay or interfere with absorption. This medication is given when
sexual activity is desired and not once daily.
Chapter 133: Testicular Disorders
Buttaro: Primary Care: A Collaborative Practice, 6th Edition
MULTIPLE CHOICE
1. A young adult male reports a dull pain in the right scrotum and the provider notes a bluish
color showing through the skin on the affected side. Palpation reveals a bag of worms on the
proximal spermatic cord. What is an important next step in managing this patient?
a. Anti-infective therapy with ceftriaxone or doxycycline
b. Consideration of underlying causes of this finding
c. Reassurance that this is benign and may resolve spontaneously
d. Referral to an emergency department for surgical consultation
ANS: B
This patient has symptoms of varicocele. Because varicocele is rare on the right side, the
provider should look for underlying causes of these findings. Anti-infective therapy is
indicated for epididymitis. Varicocele requires surgical intervention or ablation to resolve.
Testicular torsion is an emergency.
2. An adolescent male reports severe pain in one testicle. The examiner notes edema and
erythema of the scrotum on that side with a swollen, tender spermatic cord and absence of the
cremasteric reflex. What is the most important intervention?
a. Doppler ultrasound to assess testicular blood flow
b. Immediate referral to the emergency department
c. Prescribing anti-infective agents to treat the infection
d. Transillumination to assess for a “blue dot” sign
ANS: B
This patient has symptoms of testicular torsion, which is a surgical emergency. An immediate
referral is warranted. Doppler US and transillumination are useful in establishing a diagnosis,
but the referral is the most important. Anti-infective agents are used if epididymitis is
suspected.
3. A 3-month-old male infant has edema and painless swelling of the scrotum. On physical
examination, the provider can transilluminate the scrotum. What will the provider
recommend?
a. A Doppler ultrasound to evaluate the scrotal structures
b. A short course of empirical antibiotic therapy
c. Immediate referral to a genitourinary surgeon for repair
d. Observation and reassurance that spontaneous resolution may occur
ANS: D
This infant has symptoms of hydrocele; these disorders often resolve spontaneously during
infancy and do not require treatment unless symptoms, such as pain, occur. It is not necessary
to perform other studies or refer to a surgeon. Antibiotics are not indicated, since this is not
infectious.
Chapter 134: Urinary Calculi
Buttaro: Primary Care: A Collaborative Practice, 6th Edition
MULTIPLE CHOICE
1. A patient diagnosed with diabetes has symptoms consistent with renal stones. Which type of
stone is most likely in this patient?
a. Citrate
b. Cysteine
c. Oxalate
d. Uric acid
ANS: D
Uric acid stones are more prevalent in diabetics. Citrate, cysteine, and oxalate are less
common in all patients.
2. A patient diagnosed with acute renal colic is experiencing nausea and vomiting. A urinalysis
reveals hematuria but is otherwise normal. A radiographic exam shows several radiopaque
stones in the ureter which are less than 1 mm in diameter. What will the primary provider do
initially to manage this patient?
a. Obtain a consultation with a urology specialist
b. Order a narcotic pain medication and increased oral fluids
c. Prescribe desmopressin and a corticosteroid medication
d. Prescribe nifedipine and hospitalize for intravenous antibiotics
ANS: B
Stones that are less than 1 mm in diameter will usually pass spontaneously. The provider
should counsel the patient to increase fluid intake and should prescribe adequate pain
medication. A consultation is not necessary unless initial measures fail. Desmopressin and
corticosteroids have not been shown to be effective. Nifedipine and IV fluids may be used as a
secondary option.
MULTIPLE RESPONSE
1. Which factors increase the risk of renal stones? (Select all that apply.)
a. Excess antacid use
b. Living in a cold climate
c. Obesity
d. History of gout
e. Vitamin D excess
ANS: A, C, D
Excess antacids, obesity, and a history of gout are linked to renal stone risk. Tropical climates
are also linked to renal stone development. Vitamin D excess is not a risk factor.
Chapter 135: Urinary Tract Infections and Sexually Transmitted Infections
Buttaro: Primary Care: A Collaborative Practice, 6th Edition
MULTIPLE CHOICE
1. A 30-year-old male patient has a positive leukocyte esterase and nitrites on a random urine
dipstick during a well patient exam. What type of urinary tract infection does this represent?
a. Complicated
b. Isolation
c. Uncomplicated
d. Unresolved
ANS: A
All urinary tract infections (UTIs) in males are considered complicated, because the infection
source is not secondary to ascending infection.
2. An asymptomatic pregnant woman has a positive leukocyte esterase and positive nitrites on a
urine dipstick screening. What will the provider do next?
a. Admit to the hospital
b. Obtain a urine culture
c. Order a renal ultrasound
d. Prescribe trimethoprim-sulfamethoxazole (TMP-SMZ)
ANS: B
Urine culture is the definitive test and should be obtained in all pregnant women. Admission
to the hospital is usually not necessary. Renal ultrasound is used to identify abnormalities or
obstructions that may be causing recurrent symptoms. TMP-SMZ is contraindicated in
pregnant women.
3. An asymptomatic female is concerned about having come into contact with sexually
transmitted gonorrhea and asks about antibiotics. What will the provider recommend?
a. Amoxicillin-clavulanate for 10 days
b. Cultures and treatment if symptoms appear
c. Empirical ceftriaxone and azithromycin
d. Trimethoprim-sulfamethoxazole
ANS: C
Patients with gonorrhea usually have chlamydia as well, so treatment with both ceftriaxone
and azithromycin is recommended. Amoxicillin-clavulanate and TMP-SMZ are used for
urinary tract infections (UTIs). The patient should be treated empirically. Females are often
asymptomatic.

Chapter 136: Uropathies (Obstructive) and Tumors of the Genitourinary Tract


(Kidneys, Ureters, and Bladder)
Buttaro: Primary Care: A Collaborative Practice, 6th Edition
MULTIPLE CHOICE
1. A patient reports right-sided flank pain and hematuria. The provider palpates a renal mass on
the affected side. What is the probable treatment for this patient’s condition?
a. Biologic response modifiers, including interleukin
b. Ileal conduit urinary diversion surgery
c. Nephron-sparing nephrectomy and chemotherapy
d. Radiotherapy for palliation of metastatic lesions
ANS: D
This patient has the classic triad of symptoms for renal cell carcinoma, which usually do not
present until metastasis has occurred, with poor prognosis for survival. Palliative radiotherapy
is often used to treat metastatic lesions. RCC does not respond well to biologic response
modifiers. Ileal conduit diversion is used for bladder carcinoma. Nephron-sparing
nephrectomy may be used if there is a better chance of survival.
2. A patient has a partial urinary tract obstruction caused by benign prostatic hypertrophy (BPH).
The patient reports increasing difficulty initiating a urine stream and occasional incontinence
but has not experienced a urinary tract infection. Which initial treatment will be ordered?
a. A 5􀁄-reductase inhibitor
b. Surgical intervention
c. Urinary catheterization
d. Urinary diversion procedure
ANS: A
Patients with BPH may not require treatment unless there is retention, recurrent infection, or
unacceptable symptoms. This patient has symptoms, but no retention or infection. The
obstruction is not always progressive, so surgery may be delayed until more severe symptoms
occur. 5􀁄-Reductase inhibitors help reduce the prostate size and may be used initially to see if
there is symptomatic relief. Urinary catheterization is used for an acute obstruction. Urinary
diversion is used when bladder cancer is present.

Chapter 137: Amenorrhea


Buttaro: Primary Care: A Collaborative Practice, 6th Edition
MULTIPLE CHOICE
1. An adolescent female presents with amenorrhea. The provider notes short stature, neck
webbing, and a pigeon chest deformity. Based on these symptoms, what is the underlying
disorder most likely causing this patient’s amenorrhea?
a. Androgen insensitivity
b. Hypothyroidism
c. Polycystic ovarian disease
d. Turner’s syndrome
ANS: D
Turner’s syndrome is characterized by congenital short stature, neck webbing, and a pigeon
chest. Androgen insensitivity is characterized by a complete absence of axillary and pubic
hair. Hypothyroidism will cause changes in vital signs, weight gain, and fatigue. Polycystic
ovarian disease causes metabolic syndrome.
2. A 16-year-old female is diagnosed with primary amenorrhea. A pregnancy test is negative.
Further testing reveals a normal TSH and an elevated prolactin level. Which test will the
provider order next?
a. Clomiphene challenge test
b. Magnetic resonance imaging (MRI)
c. Progesterone challenge test
d. Serum dehydroepiandrosterone (DHEA)
ANS: B
In patients with primary amenorrhea who have a negative pregnancy test, normal TSH, and
elevated prolactin, the provider may order MRI or CT of the sella turcica to identify
microadenomas and macroadenomas. If that is normal, a progesterone challenge test will be
ordered to evaluate estrogen status. Clomiphene challenge is ordered to evaluate
hypergonadotropic amenorrhea. Serum DHEA, if increased, indicates an adrenal origin for
androgen in women with hirsutism.
MULTIPLE RESPONSE
1. A 15-year-old female who participates in high school track and gymnastics is experiencing
amenorrhea after having eight normal periods. A pregnancy test is negative. What may be
included in management of this patient’s amenorrhea to restore normal periods? (Select all
that apply.)
a. Calcium and vitamin D supplementation
b. Clomiphene administration
c. Combination oral contraceptives
d. Dietician consultation
e. Estriol therapy
ANS: A, D
To help maintain bone density, vitamin D and calcium should be given to any women who
have amenorrhea. A dietician should be consulted to assist with healthy weight gain.
Clomiphene has also been used alone management of amenorrhea resulting from
hypogonadotropic syndromes. Estriol, a weak estrogen, can be used to restore normal
endocrine function in functional hypothalamic amenorrhea.

Chapter 138: Bartholin Gland Cysts and Abscesses


Buttaro: Primary Care: A Collaborative Practice, 6th Edition
MULTIPLE CHOICE
1. An adolescent female reports a mass on her genitalia which is becoming increasingly painful.
On exam, the provider notes an erythematous, edematous, tender mass lateral to the vestibule
without discharge. It is determined that the patient has a Bartholin gland abscess. What will
the provider do initially?
a. Obtain a consultation for biopsy of the lesion
b. Perform a speculum examination of the vagina
c. Prescribe a broad-spectrum antibiotic
d. Refer the patient for an incision and drainage and culture
ANS: C
Empirical administration of antibiotics, if done early, can be helpful. Biopsy is performed if
the mass is suspected as being cancerous. Speculum examination is deferred until pain is
relieved. I&D has risks and often results in recurrence.
2. A patient has undergone surgical incision and drainage of a Bartholin’s cyst with insertion of
a drain. What is an important aspect of care for this patient?
a. Administration of antibiotics
b. Education about long-term dyspareunia
c. Teaching about reproductive sequelae
d. Weekly follow-up monitoring
ANS: D
Patients with surgical I&D who have a drain should be monitored weekly. If adequately
treated, long-term function should be normal.

Chapter 139: Breast Disorders


Buttaro: Primary Care: A Collaborative Practice, 6th Edition
MULTIPLE CHOICE
1. A female patient identified as having the BRCA mutation asks which intervention will reduce
the risk of breast cancer the most? What will the provider tell her?
a. Breast magnetic resonance imaging (MRI)
b. Clinical breast examination every 6 months
c. Early childbearing and breastfeeding
d. Prophylactic mastectomy
ANS: D
Although all the options help to reduce breast cancer risk, patients with BRCA mutation are at
extremely high risk and will benefit most from prophylactic mastectomy and oophorectomy.
MRI can help detect invasive cancers earlier.
2. A postmenopausal woman reports unilateral breast pain that she describes as sharp and
burning and localized to one area. A breast examination reveals no dimpling, discharge, or
masses. Which diagnostic test will the provider order?
a. Bilateral mammography
b. Focused ultrasound
c. Hormone levels
d. Needle biopsy
ANS: A
Bilateral mammography is usually performed in postmenopausal women with noncyclic
breast pain, although the likelihood of abnormal findings is low. Focused ultrasound may be
performed in addition to mammography if indicated. Hormone levels are generally normal.
Needle biopsy is performed if a suspicious mass is identified.
3. A woman who has just weaned her infant from breastfeeding develops signs of mastitis and is
treated with antibiotics. At a follow-up visit, the provider notes marked breast edema and
erythema of the affected breast. What will the provider do next?
a. Consult with a surgeon for I&D of the breast
b. Counsel the patient to apply warm compresses
c. Prescribe antibiotics to treat MRSA infection
d. Refer the patient for an immediate biopsy
ANS: D
Patients treated for mastitis who do not respond to antibiotics and who have persistent
erythema and edema are likely to have inflammatory breast carcinoma and should be referred
for a biopsy immediately. MRSA infection is possible, but these symptoms are severe, and the
patient needs immediate evaluation. Warm compresses and surgical K&D are not indicated.

Chapter 140: Chronic Pelvic Pain


Buttaro: Primary Care: A Collaborative Practice, 6th Edition
MULTIPLE CHOICE
1. A woman diagnosed with chronic pelvic pain most likely has a gynecological cause for her
symptoms. Which treatment will the provider recommend initially?
a. Counseling and support
b. Hysterectomy
c. Neurostimulation
d. Oral contraceptives
ANS: D
Oral contraceptives may be helpful and are a good initial choice. Counseling and support are
necessary throughout management, but do not help with symptom relief. Hysterectomy may
be indicated if there are more severe structural problems but does not always alleviate
symptoms. Neurostimulation is used when neuropathic pain is present.
MULTIPLE RESPONSE
1. Which underlying causes are related to chronic pelvic pain in women? (Select all that apply.)
a. Postural problems
b. Hepatitis
c. Interstitial cystitis
d. Physical abuse
e. Gastrointestinal disorders
ANS: A, C, D, E
Problems with posture can cause inflammation in the lower spinal column, leading to chronic
pelvic pain. Interstitial cystitis and physical or sexual abuse are related to CPP. Hepatitis is not
linked to CPP.
Chapter 141: Abnormal Uterine Bleeding
Buttaro: Primary Care: A Collaborative Practice, 6th Edition
MULTIPLE CHOICE
1. Which patient meets the criteria currently presented for a diagnosis of current abnormal
uterine bleeding?
a. A 45-year old who has experienced heavy menstrual void since menses began
b. A 25-year old reporting being “so fatigued after my periods”
c. A 15-year old who reports “using more pads than I did when I first got my period”
d. 35-year old experiencing a significant increase in bleeding over the last 8 months
Chronic AUB has been defined as bleeding from the uterine corpus that is abnormal in
duration, volume, frequency, or regularity and has been present for the majority of the past 6
months. The other options present signs that are not usual or have been confirmed as
abnormal for that particular patient.
MULTIPLE RESPONSE
1. When determining the cause of a patient’s abnormal uterine bleeding (AUB), which
diagnostics would the provider consider initially? (Select all that apply.)
b. Serum level of human chorionic gonadotropin
c. Thyroid-stimulating hormone
d. Total and free testosterone
Diagnostic tests prescribed initially for AUB include serum human chorionic gonadotropin
levels, thyroid-stimulating hormone level and cervical cultures. Liver function tests and total
and free testosterone levels are more advanced tests.
Chapter 142: Dysmenorrhea
Buttaro: Primary Care: A Collaborative Practice, 6th Edition
MULTIPLE CHOICE
1. An adolescent female reports crampy pelvic pain radiating to the back, sacrum, and inner
thighs during the first 2 days of each menstrual period, associated with nausea and loose tools.
She asks about what causes these symptoms. What will the provider tell her?
a. Excess prostaglandins, vasopressin will cause these symptoms.
b. Mechanical cervical obstruction and severe uterine flexion are likely causes.
c. Nulliparity and her young age are correlated with these symptoms.
d. The most likely cause at her age is anovulatory menstrual cycles.
ANS: A
Chemical mediators, including prostaglandins, vasopressin, and other substances originating
from phospholipids, have been shown to produce the symptoms and associated symptoms of
dysmenorrhea. Mechanical obstruction and severe cervical flexion have no substantial
evidence to support their role in causing these symptoms. Nulliparity and age have not shown
correlation with dysmenorrhea. Ovulation causes increased release of chemical mediators, so
anovulatory cycles will result in fewer symptoms.
2. A 35-year-old woman without a previous history of dysmenorrhea reports lower pelvic pain
and irregular bleeding between periods. What is the initial action in managing this patient?
a. Obtaining a pelvic transvaginal ultrasound with saline infusion
b. Ordering a CBC, erythrocyte sedimentation rate, and cultures
c. Performing an abdominal, pelvic, and rectovaginal examination
d. Prescribing a trial of nonsteroidal anti-inflammatory medications (NSAIDs)
ANS: C
Symptoms that occur later in life are more likely to be secondary dysmenorrhea and must be
investigated, beginning with a thorough abdominal, pelvic, and rectovaginal exam. Other tests
may be performed if indicated by findings on the physical exam. NSAIDs are first-line
therapy for primary dysmenorrhea.
3. A patient diagnosed with primary dysmenorrhea has taken nonsteroidal anti-inflammatory
medications (NSAIDs) and COX-2 inhibitors without getting relief from symptoms. What
will the provider suggest?
a. A combined oral contraceptive (OCP)
b. Complementary therapies
c. Intrauterine (IUD) device implantation
d. Narcotic analgesics as needed
ANS: A
OCPs have demonstrated some effectiveness and should be tried in this instance. IUDs are
used in Europe but have not been approved for this use in the United States. Complementary
therapies have not been sufficiently studied. Narcotic analgesics are not recommended for this
level of pain.

Chapter 143: Dyspareunia


Buttaro: Primary Care: A Collaborative Practice, 6th Edition
MULTIPLE CHOICE
1. A previously healthy 22-year-old female reports experiencing pain in the rectovaginal area
with sexual intercourse. What is the most likely cause of this patient’s discomfort?
b. Endometriosis

In the premenopausal woman, endometriosis is the most common cause of dyspareunia,


especially when it involves the rectovaginal area. Atherosclerosis and inadequate lubrication
may occur in older women. Psychological issues are less common.
2. A perimenopausal woman reports a recent onset of moderate to severe pain with sexual
intercourse. Which treatment will the provider prescribe initially to treat this pain?
c. Topical estrogen

If the problem is estrogen-insufficient dryness, which occurs during menopause, topical


estrogen is effective for moderate to severe pain. Botulism injections are useful when spasms
are the cause of pain. Topical corticosteroids are not indicated. Water-based lubrication may
be effective for mild symptoms.

Chapter 144: Ectopic Pregnancy


1. A female patient has lower abdominal pain, nausea, and vomiting and reports missing a period
3 weeks prior. The patient reports using an intrauterine device for contraception. A serum
􀁅-hCG is 1500 mIU/mL. What will the provider do, based on these findings?
a. Perform a transvaginal ultrasound

With elevations in serum 􀁅-hCG greater than 1000 mIU/mL, transvaginal US can usually
detect both viable and nonviable ectopic pregnancies without subjecting women to the risks of
an invasive procedure. A diagnostic laparoscopy is the definitive test for ectopic pregnancy
and should be performed if the US is indeterminate. The level is already high enough that a
fetus would be detectable on US, so rechecking this level is not indicated. Patients who use
IUDs are at higher risk for ectopic pregnancy; telling this woman that a viable pregnancy is
likely should not occur until diagnostic tests are performed.

2. A patient is determined to have a nonruptured ectopic pregnancy within 1 week of a missed


period. Which treatment will the specialist order?
c. Methotrexate
In an early nonruptured ectopic pregnancy, methotrexate is widely used. Leucovorin rescue
and mifepristone are used as adjunctive medications with methotrexate. Laparoscopy is
performed when a ruptured ectopic pregnancy is present.
MULTIPLE RESPONSE
1. A patient has received methotrexate therapy for treatment of an ectopic pregnancy. What
information will the provider include when teaching this patient about this therapy?
a. Avoid sexual intercourse for 3 months
b. Do not consume alcohol for at least 3 months
e. Refrain from taking vitamins containing folic acid

Chapter 145: Fertility Control


Buttaro: Primary Care: A Collaborative Practice, 6th Edition
MULTIPLE CHOICE
1. A woman who is taking oral contraceptive pills (OCPs) to prevent pregnancy calls the
provider to report forgetting to take the pills for 4 days. She has not had sexual intercourse
during that time. What will the provider recommend?
a. Resume the pills and use a backup method the remainder of the cycle
b. Stop the OCP, use an alternative method, and resume OCPs at the next cycle
c. Take 2 pills daily for 4 days and use an alternative method for 4 days
d. Use a morning after pill today and resume the OCPs now
ANS: A
When OCPs are missed longer than 2 days, women should use an alternative method of
fertility control until the end of the cycle and resume taking the pills as prescribed. It is not
recommended that the pills be stopped until a new cycle begins. When pills are missed for 2
or fewer days, women should take 2 pills daily and use a backup method until the end of the
cycle. The morning after pill is used when pregnancy may have occurred.
2. A woman has a history of migraines and requests oral contraceptives to prevent pregnancy.
Which type of contraceptive will the provider recommend?
a. Combination oral contraceptive
b. Contraceptive implant
c. Progestin-only contraception
d. Transdermal combination product
ANS: C
Women with migraine are at increased risk for stroke and should avoid estrogen-containing
contraceptives. The progestin-only pill is recommended. The progestin-only pill can also
cause headaches, but research reveals that healthy women taking progestin-only pills are not
at risk for strokes. The other products all contain estrogen. Altering the method of
administration does not alter the estrogen dose.
3. A female calls the provider to report having unprotected sexual intercourse approximately 4
days prior. Which regimen will the provider recommend?
a. Plan-B One-Step twice daily for 5 days
b. Plan-B One-Step daily for 5 days
c. Ulipristal Acetate (UA) taken one time
d. Ulipristal Acetate (UA) twice daily for 3 days
ANS: C
Although the OTC product Plan B is effective, its efficacy drops significantly after 72 hours.
The prescription product Ulipristal Acetate (UA) is prescribed as a single dose taken within
120 hours after intercourse, but studies have shown greater efficacy for this product than
Plan-B from 72 to 120 hours after unprotected sexual intercourse.

Chapter 146: Genital Tract Cancers


Buttaro: Primary Care: A Collaborative Practice, 6th Edition
MULTIPLE CHOICE
1. A postmenopausal woman who is overweight and who has hyperlipidemia and a history of
infertility develops vaginal bleeding and reports a feeling of pelvic pressure. The provider
suspects a genital tract cancer and refers the patient for diagnostic evaluation. What is the
likely cause of this woman’s symptoms?
a. Cervical cancer
b. Endometrial cancer
c. Ovarian cancer
d. Vaginal cancer
ANS: B
This woman has symptoms characteristic of endometrial cancer, including her health history,
bleeding, and pelvic pressure. Cervical cancer, ovarian cancer, and vaginal cancer typically do
not have symptoms.
2. A woman asks about her risk of ovarian cancer. To best assess risk in this patient, what will
the provider do first?
a. Ask about parity and age of menarche
b. Evaluate age of menopause
c. Obtain a family history
d. Order testing for the BRCA gene
ANS: C
The family history is the best predictor of ovarian cancer risk. The other factors contribute,
but not as significantly as family history. If there is a strong family history, testing for the
BRCA gene is indicated.
3. A woman who has had routine Pap tests all her adult life has an abnormal Pap. What will the
provider tell her about this result?
a. Colposcopy with biopsy is necessary
b. She may have vaginal cancer
c. She most likely has early cervical cancer
d. The result most likely indicates HPV infection
ANS: A
The Pap test is a screen and should be followed by colposcopy with biopsy. Women who have
had Pap tests routinely are less likely to have cervical cancer.

Chapter 147: Infertility


Buttaro: Primary Care: A Collaborative Practice, 6th Edition
MULTIPLE CHOICE
1. A provider is caring for a couple who are trying to conceive. To most accurately evaluate
ovulation and luteal surge in the woman, what test will the provider recommend or perform?
a. Basal body temperature
b. Maintaining a menstrual calendar
c. Plasma mid luteal progesterone concentration level
d. Urinary luteal hormone home kit
ANS: D
A home kit to assess urinary luteal hormone can identify the surge that precedes ovulation by
1 or 2 days; these kits, when done with an afternoon or evening urine sample, correlate well
with peak serum LH. Basal body temperature is an easy, convenient, and inexpensive method
to assess ovulatory function, but the resultant curves may be difficult to interpret. Menstrual
calendars are less accurate. Plasma mid luteal progesterone concentration levels are
presumptive of ovulation but cannot assess the quality of the luteal phase.
MULTIPLE RESPONSE
1. A couple who has been trying to conceive for over 9 months asks the provider about artificial
reproductive therapy (ART). When discussing risks and benefits associated with these
methods, what will the provider include? (Select all that apply.)
a. Higher rates of pregnancy-induced hypertension
b. Increased premature births
c. Likelihood of increased infant birth weight
d. More frequent multiple gestations
e. Reduced risk of ovarian and breast cancers
ANS: A, B, D
ART methods often produce multiple gestation births and increase the likelihood of premature
births, along with higher rates of PIH. Infants born via ART methods more often have lower
birth weights. There is an association between higher rates of ovarian cancer and use of ART.

Chapter 148: Menopause


Buttaro: Primary Care: A Collaborative Practice, 6th Edition
MULTIPLE CHOICE
1. A 45-year-old woman has gone 120 days without a menstrual period. A pregnancy test is
negative. What will the provider do, based on this symptom?
a. Evaluate for various causes of amenorrhea
b. Reassure her that this can be normal
c. Tell her she has developed early menopause
d. Tell her that this is likely perimenopause
ANS: A
Women who have prolonged episodes of amenorrhea, even with FSH levels in menopausal
range, may resume normal menstruation. Other causes of amenorrhea should be evaluated.
2. Shortly after the onset of menopause, a woman begins to show signs of osteopenia. The
provider orders oral bazedoxifene combined with conjugated estrogen (BZA/CE). What will
the provider tell her about the purpose of this treatment?
a. It will improve calcium absorption.
b. It will prevent further bone loss.
c. It will reduce the incidence of fractures.
d. It will reverse bone loss.
ANS: C
HT therapy is given to reduce bone loss, reduce the incidence of osteoporotic fractures, and
reduce height loss. It does not improve calcium absorption and will not prevent further bone
loss or reverse bone loss.
MULTIPLE RESPONSE
1. A woman who has stopped having menstrual periods has vasomotor symptoms but does not
want to use hormone replacement therapy. What measures will the provider recommend?
(Select all that apply.)
a. Avoiding caffeine
b. Engaging in regular exercise
c. Having a glass of wine each evening
d. Keeping the house cool
e. Trying herbal remedies
ANS: A, B, D
Women with vasomotor symptoms should be counseled to avoid caffeine and alcohol, to
engage in regular exercise, and to lower the thermostat. Consuming alcohol is not
recommended to reduce symptoms. Herbal remedies are frequently used but have not been
shown to be beneficial and many may be harmful.

Chapter 149: Cervical Cancer Screening Abnormalities


Buttaro: Primary Care: A Collaborative Practice, 6th Edition
MULTIPLE CHOICE
1. What is true about genital human papillomavirus (HPV) infection in women?
a. A single, isolated HPV infection will lead to cervical cancer.
b. HPV subtypes causing genital warts indicate the likelihood of cancer.
c. Most HPV lesions will resolve spontaneously without causing cancer.
d. Most women with HPV infection will develop cervical cancer.
ANS: C
Most HPV infections resolve spontaneously, especially in younger women. It is the
persistence of HPV infection and not a single infection that increases the risk. The subtypes
causing genital warts are not implicated in the etiology of cervical cancer. Most women with
HPV infection will not develop cancer.
2. What is the benefit of using a liquid-based medium when performing a Pap test?
a. It allows genital human papillomavirus (HPV), gonorrhea, and chlamydia testing
with the same specimen.
b. It is a more accurate test than the slide method of Pap testing.
c. It is not necessary to use endocervical cells to obtain results.
d. It is predictive of the later development of cervical cancer.
ANS: A
The liquid-based medium method allows testing of HPV, gonorrhea, and chlamydia using the
same specimen. It is not necessarily more accurate. It requires use of endocervical cells. It
does not predict the development of cancer.
3. A young female has a Pap test which reveals atypical squamous cells of undetermined
significance. What will the next step be in managing this patient?
a. Colposcopy with biopsy
b. Endometrial sampling
c. Reflex HPV DNA test
d. Repeat cytology in 12 months
ANS: D
Young women with ASC-US may have repeat cytology testing in 12 months. Colposcopy
with biopsy is performed if low-grade intraepithelial lesions or high-grade squamous
intraepithelial lesions are present. Reflex HPV DNA testing is not indicated. Endometrial
sampling is performed with low- or high-grade lesions in women who are older than 35 years.

Chapter 150: Pelvic Inflammatory Disease


Buttaro: Primary Care: A Collaborative Practice, 6th Edition
MULTIPLE CHOICE
1. A female patient reports cramping, dysuria, low back pain, and nausea. A dipstick urinalysis
is normal and a pregnancy test is negative. What will the provider do next?
a. Obtain vaginal secretions for testing
b. Refer for radiological studies
c. Schedule for an endometrial biopsy
d. Treat with empirical antibiotics
ANS: A
The provider should continue to evaluate by getting specimens for testing to rule out active
infection. Radiological studies and endometrial biopsy may be necessary if other tests are not
conclusive. Empirical antibiotics should not be given until other causes are ruled out.
2. A sexually active female has symptoms of pelvic inflammatory disease (PID) with fever.
Cultures are pending. For outpatient treatment, what will the provider order?
a. Ampicillin and doxycycline
b. Cefotetan and doxycycline
c. Clindamycin and gentamicin
d. Rocephin, doxycycline, and metronidazole
ANS: D
For outpatient management, Rocephin is given once IM and then doxycycline and
metronidazole are given for 14 days.
MULTIPLE RESPONSE
1. Which are greatest risk factors for pelvic inflammatory disease? (Select all that apply.)
a. Age under 25 years
b. Cigarette smoking
c. Tubal sterilization
d. Using barrier methods of contraception
e. Vaginal douching
ANS: A, B, E
Age under 25 years, cigarette smoking, and vaginal douching all increase the risk for PID.
Tubal sterilization and barrier contraceptive methods decrease the risk.

Chapter 151: Sexual Dysfunction (Female)


Buttaro: Primary Care: A Collaborative Practice, 6th Edition
MULTIPLE CHOICE
1. What is a possible cause of decreased arousal, libido, and orgasm in a postmenopausal
woman?
a. Diminished testosterone
b. Endometriosis
c. Low estrogen levels
d. Uterine prolapse
ANS: A
Diminished testosterone has been implicated in deceased arousal, libido, and orgasm in
women. Endometriosis is a common cause of deep dyspareunia. Low estrogen causes vaginal
dryness, coital pain, and decreased desire. Uterine prolapse causes deep dyspareunia.
2. A female patient with vaginal atrophy is prescribed a local estrogen therapy product with
conjugated equine estrogen cream. What will the provider recommend as adjunctive therapy
for this patient?
a. Adding progesterone to the regimen
b. Pelvic floor physical therapy
c. Treatment with a testosterone patch
d. Trigger point injections
ANS: A
Conjugated equine estrogen is absorbed systemically, so the patient will need progesterone to
prevent unopposed estrogen stimulation of the endometrium. Pelvic floor PT and trigger point
injections are used for dyspareunia. Testosterone patches are used for decreased arousal and
desire.

Chapter 152: Unplanned Pregnancy


Buttaro: Primary Care: A Collaborative Practice, 6th Edition
MULTIPLE CHOICE
1. To help prevent unplanned pregnancies, which group of providers may make the most impact
by providing contraceptive counseling to women?
a. Counselors in STI clinics
b. Gynecologists
c. Practitioners in walk-in clinics
d. Primary care providers
ANS: D
Women receive most of their preventive care from nongynecologic providers, so PCPs have a
unique opportunity to provide contraceptive counseling to women. Women seek care in STI
clinics and gynecology clinics for specific reasons often after contraceptive counseling is no
longer preventive.
2. An adolescent female calls a primary care clinic to report that she has missed two periods and
is having morning nausea and vomiting. What will the provider suggest?
a. Coming to the clinic for pregnancy testing
b. Making an appointment with a gynecologist
c. Purchasing a home pregnancy test kit
d. Referral to a community health clinic
ANS: A
Patients who feel especially vulnerable and are in denial about a pregnancy may not want to
take a test at home, desiring immediate guidance when a result is discovered. Referring to
another provider will only alienate this adolescent further.
3. A provider sees a woman who has just learned she is pregnant with an unplanned pregnancy.
What is an initial step in helping this woman make decisions about his pregnancy?
a. Actively listen to the woman’s concerns and questions.
b. Ask the mother if the father of the baby will be involved.
c. Make a referral to an obstetrician for more information.
d. Offer information about both adoption and abortion.
ANS: A
The initial step is to actively listen to the woman to determine what her concerns and needs
are. The provider should give unbiased information and answer any questions the woman has.
Asking her about the father’s involvement puts an emphasis on other concerns. Referring her
to an obstetrician conveys a desire to avoid helping her with her problems. Giving information
that she has not asked for conveys a judgment about what she should do.

Chapter 153: Vulvar and Vaginal Disorders


Buttaro: Primary Care: A Collaborative Practice, 6th Edition
MULTIPLE CHOICE
1. A female patient reports vulvar pruritus and dyspareunia. The provider notes white papules on
the vulva with thinning of the epithelium. What condition does the provider suspect?
a. Bartholin’s duct cyst
b. Lichen sclerosis
c. Sexually transmitted infection
d. Vulvar psoriasis
ANS: B
These symptoms are consistent with lichen sclerosis, with white papules and epithelial
thinning being diagnostic. The other conditions may cause pruritus, but not skin lesions.
2. A postmenopausal woman is diagnosed with lichen planus. After several weeks of treatment
with a potent corticosteroid ointment, the woman reports improvement in itching, but states
that she has extreme vaginal dryness. What will the provider recommend?
a. Increasing the frequency of the corticosteroid application
b. Taking diphenhydramine daily at bedtime
c. Using a topical estrogen cream along with the steroid
d. Using petrolatum ointment to minimize drying
ANS: C
Patient with concomitant vulvovaginal atrophy related to menopause may need to use topical
estrogen cream to treat those symptoms. After several weeks, the corticosteroid is tapered to a
lower dose, not increased. Diphenhydramine is given for itching. Petrolatum ointment is
useful for external skin as a barrier but is not used internally.
3. A sexually active young female reports vaginal discharge and moderate vulvovaginal
irritation. The examination reveals a white, noninflammatory discharge adhering to the
vaginal walls, clue cells on microscopic examination, and a positive KOH (potassium
hydroxide) whiff test. What will the provider do next?
a. Obtain cultures of the vaginal discharge
b. Order a gynecologic referral for evaluation and treatment
c. Prescribe oral metronidazole 500 mg twice a day for 7 days
d. Screen the patient for gonorrhea and chlamydia
ANS: C
Bacterial vaginosis may be treated empirically based on the symptoms this woman has and on
the three criteria noted in clinic. Metronidazole is the first-line treatment. It is not necessary to
obtain cultures or to refer to a specialist.

Chapter 154: Ankle and Foot Pain


Buttaro: Primary Care: A Collaborative Practice, 6th Edition
MULTIPLE CHOICE
1. A patient injures an ankle while playing soccer and reports rolling the foot inward while
falling with immediate pain and swelling of the lateral part of the joint. The patient is able to
bear weight and denies hearing an audible sound at the time of injury. What does this history
indicate?
a. Likely ankle sprain with a possible fracture
b. Mild ankle injury without fracture
c. Mild soft tissue injury only
d. Serious ankle injury with certain fracture
ANS: A
Immediate swelling of the joint raises the index of suspicion for a fracture or a substantial
amount of joint involvement. Without radiographs, none of these possibilities can be
confirmed.
2. A patient who is a distance runner reports pain in one heel that is worse in the morning and
seems to improve with exercise. The provider notes localized swelling and a bony prominence
at the heel. What is the initial treatment for this condition?
a. Cessation of all sports activities and exercise
b. Crutches and partial weight bearing
c. Physical therapy for ultrasound therapy
d. Referral to an orthopedist for MRI and evaluation
ANS: A
This patient has symptoms consistent with Achilles tendonitis. Immediate cessation of sports
and exercise is the first step in management. Crutches and partial weight bearing may be
indicated if symptoms do not improve with rest and NSAIDs. Physical therapy is used as
adjunctive therapy. Symptoms that do not improve require referral.
3. A patient has pain on the plantar aspect of the heel with weight bearing after rest. The pain is
worsened with dorsiflexion of the foot. What is the initial treatment for this patient?
a. A series of steroid injections
b. Avoiding all high-impact activities
c. Night splints
d. Wearing flat shoes only
ANS: B
This patient has signs of plantar fasciitis. The initial treatment includes avoiding all
high-impact activities. A single steroid injection may be given with subsequent injections if
no improvement. Night splints are part of second-tier treatment. Flat shoes should be avoided.

Chapter 155: Bone Lesions: Neoplasms and Tumor Mimickers


Buttaro: Primary Care: A Collaborative Practice, 6th Edition
MULTIPLE CHOICE
1. A provider discovers a bone tumor as an incidental finding on a radiograph in a patient who
has sustained an injury to a ligament. The patient has not had pain prior to the injury. What
will the provider do next?
a. Consult with an orthopedic specialist
b. Order a chest CT and full body scan
c. Refer the patient to for a bone biopsy
d. Repeat the radiograph in 3 to 6 months
ANS: D
Latent bone tumors are usually discovered as incidental findings during evaluation for
musculoskeletal injury. If the injury is the source of pain, the radiograph may be repeated in 6
to 12 months to determine whether it is increasing in size. Consultation with an orthopedic
specialist, referral for a biopsy, and further testing with chest CT or full body scanning are
done if there is suspicion of an active tumor.
2. A patient reports persistent lower back pain and constipation. A digital rectal examination
reveals a mass at the sacrum. What will the primary care provider do to manage this patient?
a. Order spinal radiographs in 3 months
b. Perform an MRI of the sacrum
c. Refer the patient to an oncologist
d. Schedule the patient for a biopsy
ANS: C
Patients with chordoma, which is a type of sarcoma with a predilection for the sacrum, will
have these symptoms and a palpable mass coming out of the sacrum. A referral to an
oncologist is necessary. These tumors have a significant risk for malignancy, so waiting 3
months is not an option. The oncologist will order a CT and body scan and possibly biopsy or
surgery.
MULTIPLE RESPONSE
1. Which treatments may be used to manage bone pain in patients with bone tumors? (Select all
that apply.)
a. Bisphosphonates
b. Exercise
c. External beam radiation
d. Massage
e. Vertebroplasty
ANS: A, B, C, E
Bisphosphonates can decrease pain by preventing growth and development of existing and
new bone lesions. Exercise is useful to maintain function and reduce pain. External beam
radiation is useful in most patients. Vertebroplasty involves injecting bone cement to stabilize
bone. Massage is not recommended.

Chapter 156: Bursitis


Buttaro: Primary Care: A Collaborative Practice, 6th Edition
MULTIPLE CHOICE
1. A patient experiencing shoulder pain is seen by an orthopedic specialist who notes erythema,
warmth, and fluctuance of the shoulder joint. What is the next step in treatment for this
patient?
a. Admit to the hospital for intravenous antibiotics.
b. Inject lidocaine into the joint and reassess in 5 to 10 minutes.
c. Order a plain radiograph of the shoulder to identify possible fracture.
d. Perform a shoulder ultrasound to further evaluate the cause.
ANS: A
Immediate referral is indicated for patients who present with symptoms consistent with septic
bursitis, such as with the symptoms above. Lidocaine is injected into a painful joint to
evaluate for improvement to determine whether bursitis or tendonitis is present as a result of
impingement. This exam is not consistent with fracture. An ultrasound is not indicated.
2. A patient with elbow pain without localized erythema or warmth is diagnosed with bursitis of
the elbow and serum laboratory results are pending. What is the initial treatment while waiting
for these results?
a. Aspiration of the bursal sac for culture
b. Corticosteroid injection into the bursal sac
c. Elbow pads, NSAIDs, rest, and ice
d. Physical and occupational therapy
ANS: C
Initial therapy includes conservative measures for comfort. Until infection is suspected, based
on the white blood count and inflammatory markers, and without localized signs of infection,
aspiration of the bursal sac is not indicated. Corticosteroids should not be injected into the
bursal sac until infection has been excluded. Physical and occupational therapy should not
precede comfort measures.
MULTIPLE RESPONSE
1. What are included in the initial management of bursitis of the heel? (Select all that apply.)
a. Activity modification and bracing
b. Closed heel shoes to prevent further injury
c. Corticosteroid injections
d. Nonsteroidal anti-inflammatory medications
e. Rest, ice, compression, and elevation
ANS: A, D, E
Activity modification and bracing, NSAIDs, and RICE are all used initially to treat heel
bursitis. Patients should wear open-heeled shoes. Corticosteroid injections should be used
cautiously to prevent rupture of the Achilles tendon.

Chapter 157: Fibromyalgia and Myofascial Pain Syndrome


Buttaro: Primary Care: A Collaborative Practice, 6th Edition
MULTIPLE CHOICE
1. Which cause is implicated in patients with fibromyalgia syndrome (FMS)?
a. Autoimmune disease
b. Central nervous system dysfunction
c. Muscle dysfunction
d. Viral disease
ANS: B
Although the cause of FMS is unclear, current research suggests a CNS cause and not muscle,
autoimmune, or viral causes.
2. When counseling a patient about the long-term effects of fibromyalgia syndrome, what is
important to include in teaching?
a. A multidisciplinary approach to treatment is most effective.
b. Eventual damage to muscles and joints will occur.
c. Exercise may cause discomfort and damage to muscles.
d. Medications are useful for controlling and preventing symptoms.
ANS: A
A multidisciplinary approach to FMS management can help with pain management, stress,
and exercise. Although patients experience pain, damage to tissues does not occur. Exercise
may be painful but does not cause damage. Medications help alleviate some, but not all
symptoms.
MULTIPLE RESPONSE
1. Which are symptoms associated with fibromyalgia? (Select all that apply.)
a. Gastrointestinal complaints
b. Hepatosplenomegaly
c. Musculoskeletal pain
d. Nonrestorative sleep
e. Renal complications
ANS: A, C, D
Fibromyalgia may cause GI complaints, musculoskeletal pain, and nonrestorative sleep.
Hepatosplenomegaly and renal complications are not associated with fibromyalgia.

Chapter 158: Gout


Buttaro: Primary Care: A Collaborative Practice, 6th Edition
MULTIPLE CHOICE
1. A postmenopausal female patient has a blood test that reveals hyperuricemia, although the
patient has no symptoms of gout. What will the provider do initially?
a. Ask the patient about medications and medical history
b. Begin therapy with colchicine and an NSAID
c. Recommend a low-purine, alcohol-restricted diet
d. Treat for gout prophylactically to prevent a flare
ANS: A
Patients without symptoms of gout but with hyperuricemia do not need treatment, since most
of these patients will never have a gout flare. It is important, however, to determine the cause
of this finding and correct it if possible, since it is a risk factor for gout. Certain medications
and medical conditions can predispose patients to gout. Colchicine and NSAIDs are used to
treat symptoms of gout. Dietary changes are not necessary and are difficult to follow.
Prophylaxis for prevention of flares is for patients who have gout and who are between flares.
2. A patient with gout and impaired renal function who uses urate-lowering therapy (ULT) is
experiencing an acute gout flare involving one joint. What is the recommended treatment?
a. Administration of intraarticular corticosteroid
b. Discontinuing ULT while treating the flare
c. Oral colchicine for 5 days
d. Therapy with NSAIDs begun within 24 hours
ANS: A
Intraarticular steroids are practical and beneficial when only one or two joints are involved
and are safe for patients who cannot use NSAIDs or colchicine. NSIADs are contraindicated
in patients with renal disease and colchicine should not be used in those with low glomerular
filtration rates. It is not necessary to discontinue urate-lowering therapy during an acute
attack.
3. A patient experiences a second gouty flare and the provider decides to begin urate-lowering
therapy (ULT). How should this be prescribed?
a. Begin with a high-loading dose and gradually decrease.
b. Start ULT during the current flare for best results.
c. Start ULT in 5 weeks along with an anti-inflammatory drug.
d. ULT should be suspended during future gouty flares.
ANS: C
Beginning therapy with a urate-lowering drug during an acute flare will prolong the flare.
Typically, ULT is begun 5 to 6 weeks after a flare and should be given with an
anti-inflammatory drug, since the initial period of ULT administration is associated with
flares. ULT dosing should start low and gradually increase. It is not recommended to stop
ULT during future flares, but to treat those flares while continuing the ULT.

Chapter 159: Septic Arthritis


Buttaro: Primary Care: A Collaborative Practice, 6th Edition
MULTIPLE CHOICE
1. A patient reports the sudden onset of pain, redness, and swelling in one knee joint but denies a
fever. The provider elicits exquisite pain with manipulation of the joint and notes no decrease
in pain when the joint is at rest. Which is the likely cause of this arthritis?
a. Bacterial infection
b. Gout
c. Lyme disease
d. Rheumatoid arthritis
ANS: A
Septic arthritis is usually painful both with movement and at rest and is accompanied by
swelling and erythema. Fever is not always present. The other causes of arthritis are not
painful at rest.
2. An adolescent patient reports intermitted pain and swelling in various joints on the right side
including the knee, elbow, wrist, and ankle. A physical examination reveals tenosynovitis and
a maculopapular rash. Which diagnostic tests will be most helpful in determining a diagnosis
in this patient?
a. Blood cultures and a complete blood count
b. Cultures of the urethra, pharynx, cervix, and rectum
c. Skin lesion scrapings and cultures
d. Urine cultures and renal function studies
ANS: B
This patient has signs of gonococcal arthritis. Cultures of the urethra, pharynx, cervix, and
rectum will be positive in 80% of patients with this infection. Blood cultures are likely to be
negative. Culturing skin lesions is not helpful. Renal involvement is not part of this infection.
3. A patient has marked swelling of a shoulder joint with erythema and severe pain. The
provider suspects a bacterial cause. Which culture will be most helpful to determine the cause
of these symptoms?
a. Blood culture
b. Synovial fluid culture
c. Urethral culture
d. Urine culture
ANS: B
Synovial fluid culture is the most important exam for diagnosis of septic arthritis. Blood
culture may be positive in only 10% of cases. Urethral culture is performed if gonococcal
arthritis is suspected. Urine culture is not helpful.

Chapter 160: Low Back Pain


Buttaro: Primary Care: A Collaborative Practice, 6th Edition
MULTIPLE CHOICE
1. A patient reports severe back pain located in the lumbar spine. To evaluate whether the patient
has axial pain or radicular pain, which assessment is necessary?
a. Asking the patient to perform the Valsalva maneuver
b. Assessing reflexes and asking about tingling or numbness
c. Determining whether the pain is present with prolonged sitting
d. Noting whether pain is mitigated with frequent position shifts
ANS: A
Associated neurological signs are present with radicular pain and include numbness, tingling,
weakness, and reflex changes. The other symptoms occur with both axial and radicular pain.
2. A patient has an acute onset of lower back pain associated with lifting heavy objects at work.
A physical examination reveals no loss of lower extremity function or neurological symptoms.
What is the initial intervention for this patient?
a. Magnetic resonance imaging (MRI) to evaluate soft tissue involvement
b. Plain radiographs to evaluate the extent of the injury
c. Traction therapy to minimize complications
d. Treatment with a nonsteroidal anti-inflammatory drug (NSAID)
ANS: D
NSAIDs are appropriate as first-line treatment in patients without potential complications.
Radiologic studies are performed if improvement does not occur in 4 to 6 weeks. Traction
may be used for patients with radicular symptoms to help resolve neurological deficits,
although systematic review of research has not clearly identified a benefit to this therapy.
3. A patient has recurrent lumbar pain which is sometimes severe. The patient reports that
prescription of nonsteroidal anti-inflammatory drugs (NSAIDs) is no longer effective for pain
relief. What will the provider recommend?
a. Adjunctive treatment with physical therapy
b. Beginning treatment with opioid analgesics
c. Complementary and alternative therapies
d. Referral to an interventional spine physician
ANS: D
Patients with recurrent or chronic lower back pain may benefit from lumbar epidural
corticosteroid injection performed by an interventional spine physician. Physical therapy is
often used for acute injury if no improvement in 4 to 6 weeks. Opioid analgesics are not
usually effective.

Chapter 161: Hip Pain


Buttaro: Primary Care: A Collaborative Practice, 6th Edition
MULTIPLE CHOICE
1. An adult patient who has been taking high-dose corticosteroids reports a dull, aching pain in
the groin and presents with a limp. What condition does the provider suspect, based on this
history?
a. Avascular necrosis of the femoral head
b. Infectious arthritis of the hip
c. Osteoarthritis of the hip
d. Slipped capital femoral epiphysis (SCFE)
ANS: A
Avascular necrosis has the symptoms listed above and is common among patients who have
been taking corticosteroids. Infectious arthritis will typically be accompanied by fever and
intense pain. Osteoarthritis causes progressively worsening pain with activity and
improvement with rest. SCFE is common in adolescents.
2. A patient with chronic hip pain cannot take nonsteroidal anti-inflammatory drugs (NSAIDs)
and tells the provider that acetaminophen is minimally helpful. What might the provider
recommend initially to improve pain relief?
a. A fentanyl patch
b. Capsaicin
c. Glucosamine
d. Lidocaine patches
ANS: B
Topical capsaicin has been shown to provide short-term pain relief and has fewer side effects
than oral agents. It is an appropriate initial therapy. Fentanyl is a narcotic analgesic and should
be reserved for more severe pain. Glucosamine and lidocaine may be helpful for some
patients.

Chapter 162: Knee Pain


Buttaro: Primary Care: A Collaborative Practice, 6th Edition
MULTIPLE CHOICE
1. A high school soccer player sustains a knee injury when kicked on the lateral side of the knee
by another player. The provider notes significant swelling of the knee, with pain at the joint
line on the medial aspect of the knee. What will the provider do to treat this injury?
a. Instruct about RICE management and follow up in 1 week
b. Refer for a same-day orthopedic consultation
c. Schedule a magnetic resonance imaging (MRI) exam
d. Splint the knee and refer for orthopedic consultation in 1 to 2 weeks
ANS: B
This patient has an injury caused by a traumatic event associated with swelling and should
have a same-day orthopedic consultation. Simple sprains may be managed with RICE. MRI
may be ordered by the orthopedist.
2. Which maneuver during a physical examination is used to assess the anterior cruciate
ligament (ACL)?
a. Anterior drawer test
b. Posterior drawer test
c. Valgus stress on knee joint
d. Varus stress on knee joint
ANS: A
The anterior drawer test, in which the examiner pulls the tibia forward while the knee is
flexed, is used to assess anterior cruciate ligament laxity. The posterior drawer test is used to
determine posterior cruciate ligament laxity. The valgus stress test assesses the medial
collateral ligament (MCL) laxity and the varus stress test assesses the lateral collateral
ligament (LCL) laxity.
3. A previously healthy patient reports a sensation of one knee locking or feeling like it will give
way when descending stairs. The patient has no recollection of injury to the knee and denies
pain. What is the most likely treatment for this disorder?
a. Conservative management with RICE and activity modification
b. Immediate referral to an orthopedic surgeon for possible repair
c. Intraarticular injections of corticosteroids three times yearly
d. Restricting participation in sports and strenuous workouts indefinitely
ANS: A
This patient has symptoms consistent with chronic degenerative meniscal injury and is
without pain or significant disability. Conservative management is indicated. Immediate
referral is indicated for severe pain or disability. Intra-articular injections of corticosteroids
are used for patients with concomitant osteoarthritis. Patients should be encouraged to
continue sports and exercise to improve overall muscle tone and minimize disability.

Chapter 163: Metabolic Bone Disease: Osteoporosis and Paget’s Disease of the Bone
Buttaro: Primary Care: A Collaborative Practice, 6th Edition
MULTIPLE CHOICE
1. The primary care provider is assessing a 45-year-old postmenopausal woman who has a
family history of osteoporosis. Which test will be most useful to screen for this disease in this
patient?
a. Biochemical markers of bone resorption and bone formation
b. Bone densitometry of the hip and posteroanterior lumbar spine
c. Plain radiographs of the hips and lumbar and thoracic spine
d. Serum calcium and serum 25-hydroxyvitamin D
ANS: B
Postmenopausal women are candidates for bone densitometry to assess for osteopenia and
osteoporosis. Biochemical markers are generally ordered by specialists; their role in primary
care is uncertain. Plain radiographs are used to determine fracture. Serum calcium and vitamin
D levels are useful in the general population as a preventive measure.
2. A patient is diagnosed with osteoporosis. What is the recommended treatment once the
diagnosis is made?
a. Bisphosphonate therapy
b. Calcium and vitamin D
c. Estrogen replacement
d. Yoga and weight-bearing exercises
ANS: A
Bisphosphonates are FDA-approved treatment for osteoporosis and will help improve bone
density and reduce the risk of fractures. Calcium and vitamin D may help prevent osteoporosis
but must be taken from an early age. Estrogen replacement is used to prevent osteoporosis.
Yoga and exercise help with balance and muscle strength to help prevent falls.
3. A patient has bone pain and laboratory testing reveals an elevated serum alkaline phosphatase
(SAP). Which test can help distinguish Paget’s from malignant bone disease?
a. Bone densitometry
b. Bone marrow biopsy
c. Bone radiograph
d. Bone scan
ANS: C
A plain bone radiograph will show changes pathognomonic of Paget’s disease. The other tests
are not necessary.

Chapter 164: Neck Pain


Buttaro: Primary Care: A Collaborative Practice, 6th Edition
MULTIPLE CHOICE
1. A patient comes to a provider with reports of unilateral arm pain and weakness with mild neck
pain. The provider notes that the patient prefers holding the affected arm crossed in front of
the throat. A history reveals a recent onset of sexual dysfunction. What does the provider
suspect based on this history?
a. Axial neck pain
b. Cervical myelopathy
c. Diabetic neuropathy
d. Facet joint pain
ANS: B
Patients with neurological symptoms have radicular neck pain, which is usually greater in one
arm and involves neurological findings. Patients with concurrent lower extremity findings
may have cervical myelopathy and should be evaluated immediately. Axial neck pain does not
involve neurological findings. Diabetic neuropathy typically does not include neck pain. Facet
joint pain is a cause of axial neck pain, associated with injuries and headaches.
2. A provider suspects degenerative disk disease in a patient with chronic neck pain. Which
diagnostic test will be performed?
a. Computerized tomography (CT)
b. Magnetic resonance imaging (MRI)
c. Plain radiograph
d. Radionuclide bone scintigraphy
ANS: B
MRI is usually performed to diagnose degenerative disk disease. CT testing is used to identify
bone and degenerative changes, but the exposure to radiation is high. Plain radiographs are
used to identify fractures or when trauma or cancer is present. Radionuclide bone scintigraphy
is used for osteomyelitis, metastatic disease, or occult fracture.
3. A patient has chronic radicular neck pain that no longer responds to over-the-counter NSAIDs
and physical therapy measures and reports having difficulty sleeping. Which medication will
the provider order?
a. A skeletal muscle relaxant
b. A tricyclic antidepressant
c. An opioid analgesic
d. Gabapentin
ANS: D
Gabapentin is useful for central pain syndromes and radiculopathy and can help to restore
sleep. Skeletal muscle relaxants are useful for muscle spasms. A tricyclic antidepressant is
useful for some chronic neck pain, although gabapentin is more specific to this patient’s
symptoms. Opioid analgesics should be used cautiously.

Chapter 165: Osteoarthritis


Buttaro: Primary Care: A Collaborative Practice, 6th Edition
MULTIPLE CHOICE
1. A 50-year-old woman reports pain in one knee upon awakening each morning that goes away
later in the morning. A knee radiograph is negative for pathology and serum inflammatory
markers are normal. What will the provider tell this patient?
a. A magnetic resonance imaging study is necessary for diagnosis
b. That the lack of findings indicates no disease process
c. To take acetaminophen 1 gram three times daily for pain
d. To use a cyclooxygenase 2-selective NSAIDs to reduce inflammation
ANS: C
Acetaminophen is the mainstay for initial treatment of osteoarthritis. Radiologic findings are
often negative in the early stages of the disease. There are no serologic markers for OA. A
COX-2-selective inhibitor has cardiovascular side effects and should not be used unless
necessary. These agents are used more for pain than for inflammation.
2. A patient who has osteoarthritis in the carpometacarpal joints of both thumbs asks about
corticosteroid injections to treat symptoms. What will the provider tell this patient about this
therapy?
a. Corticosteroid therapy reduces inflammation and improves joint mobility
b. Injections may be administered as needed up to 6 times per year
c. Intraarticular injections provide significant pain relief for 3 to 4 months
d. This treatment may cause a temporary increase in pain, warmth, and redness
ANS: D
Intraarticular injections of corticosteroids are helpful in decreasing pain, but may cause a
transient increase in pain, warmth, and redness. This therapy does not improve inflammation
and joint mobility. Injections are not recommended more than 3 to 4 times per year. The
duration of pain relief is variable.
3. A 45-year-old patient has mild osteoarthritis in both knees and asks about nonpharmacologic
therapies. What will the provider recommend?
a. Aerobic exercise
b. Glucosamine with chondroitin
c. Therapeutic magnets or copper bracelets
d. Using a cane or walker
ANS: A
Aerobic exercise helps with cardiovascular conditioning and weight reduction as well as
improved range of motion, decreased pain, and strengthening of supporting structures.
Randomized controlled studies have failed to demonstrate significant pain relief with
glucosamine. Therapeutic magnets and copper have not been proven to be effective. A young
patient with mild symptoms will not need assistive devices and should focus on conditioning.

Chapter 166: Osteomyelitis


Buttaro: Primary Care: A Collaborative Practice, 6th Edition
MULTIPLE CHOICE
1. A 50-year-old patient with diabetes mellitus has a low-grade fever and pain on one foot. The
provider notes erythema and swelling at the site along with several superficial skin ulcers
without necrosis and suspects osteomyelitis. Which type of diagnostic study will the provider
order?
a. Biopsy of bone or debridement cultures
b. Blood cultures and serologic markers of inflammation
c. Magnetic resonance imaging of the foot
d. Plain radiograph of the foot
ANS: D
A patient with diabetic foot infection suspected of having osteomyelitis should have a plain
radiograph to identify bony abnormality or soft tissue changes. MRI may be performed if
more specific evaluation is needed or if abscess is suspected. Blood cultures are not diagnostic
of osteomyelitis. Biopsy and debridement cultures increase the risk of further infection if poor
healing at the site occurs.
2. A 3-year-old child has marked pain in one leg localized to the upper tibia with refusal to bear
weight. The child has a high fever and a toxic appearance. Which type of osteomyelitis is
most likely?
a. Chronic osteomyelitis
b. Hematogenous osteomyelitis
c. Osteomyelitis from a contiguous focus
d. Peripheral vascular disease osteomyelitis
ANS: B
Young children are more likely to have hematogenous osteomyelitis, especially with acute
symptoms. Chronic osteomyelitis is more common with underlying diseases such as diabetes.
Contiguous focus osteomyelitis occurs when organisms are introduced from a puncture
wound, foreign body, or adjoining soft tissue infection. Peripheral vascular causes are more
common in chronically ill patients.
3. A patient has osteomyelitis related to vascular insufficiency. Which initial consultation is
necessary?
a. Infectious disease consultation
b. Neurosurgical consultation
c. Surgical consultation
d. Wound care specialist consultation
ANS: C
Because patients with vascular insufficiency who develop osteomyelitis may need
debridement or draining of lesions, a surgical consult is necessary. Infectious disease consults
are obtained for patients with resistant organisms or complex wounds. Neurosurgical consults
are needed for patients with epidural abscess. Wound care consults are needed for patients
with progressive or chronic wounds.

Chapter 167: Shoulder Pain


Buttaro: Primary Care: A Collaborative Practice, 6th Edition
MULTIPLE CHOICE
1. A 45-year-old patient reports a recent onset of unilateral shoulder pain which is described as
diffuse and is associated with weakness of the shoulder but no loss of passive range of motion.
What does the provider suspect as the cause of these symptoms?
a. Acromioclavicular joint disease
b. Cervical radicular pain
c. Glenohumeral arthritis
d. Rotator cuff injury
ANS: D
Rotator cuff injury is usually characterized by diffuse pain, weakness of the joint, but no
change in range of motion. Acromioclavicular joint disease is associated with
anterior-superior shoulder pain. Glenohumeral arthritis has similar symptoms, but with loss of
range of motion. Cervical radicular pain is characterized by pain distal to the elbow.
2. An examiner is evaluating a patient who reports unilateral shoulder pain and notes limited
active and passive range of motion in the affected shoulder along with erythema and bulging
on the anterior shoulder. What diagnosis is likely with this presentation?
a. Acromioclavicular joint disease
b. Adhesive capsulitis
c. Inflammatory bursitis
d. Rotator cuff tear
ANS: C
Both bursitis and adhesive capsulitis will present with decreased active and passive range of
motion, but patients with inflammatory bursitis will exhibit erythema and bulging of the
anterior shoulder. Acromioclavicular joint disease does not cause erythema or bulging of the
joint.
3. A patient reports a deep ache in one shoulder and the provider suspects tendonitis secondary
to repetitive activity. To determine whether the pain is caused by impingement on the
acromion, the provider will ask the patient to
a. abduct the arm.
b. adduct the arm.
c. internally rotate the shoulder.
d. shrug the shoulders.
ANS: A
If pain is caused by impingement on the acromion, the patient will reflexively shrug when
asked to abduct the arm to reduce the pain. Adduction of the arm does not elicit this response.
The shrug elicited is reflexive and not intentional. Internal rotation may be performed to
evaluate generalized muscle weakness.

Chapter 168: Sprains, Strains, and Fractures


Buttaro: Primary Care: A Collaborative Practice, 6th Edition
MULTIPLE CHOICE
1. A soccer player is brought to the emergency department after twisting an ankle during a game.
An examination of the affected joint reveals ecchymosis and edema of the ankle and limited
joint laxity along with pain on weight-bearing, although movement with pain is intact. Which
grade sprain is likely?
a. Grade I
b. Grade II
c. Grade III
d. Grade IV
ANS: B
This patient probably has a grade II sprain, which involves incomplete tear of a ligament with
some functional impairment, ecchymosis, and pain with weight-bearing. A grade I sprain
causes only pain and edema. A grade III sprain is a full or complete tear of the ligament with
loss of ligament integrity. A grade IV sprain results in severe weakness with loss of function.
2. A school-age child falls off a swing and fractures the humerus close to the elbow joint. What
is the most important assessment for this patient to evaluate possible complications of this
injury?
a. Evaluation of pain with extension
b. Palpation for joint laxity
c. Salter-Harris classification
d. The presence of a spiral fracture
ANS: C
Salter-Harris classification identifies the degree of epiphyseal, or growth plate involvement
and is important to evaluate in children and adolescents, since damage to the growth plate can
result in shortening of the long bone. The other assessments are part of the exam but have less
importance than assessment of growth plate involvement.
MULTIPLE RESPONSE
1. An emergency department provider is giving instructions for rest, ice, compression, and
elevation (RICE) treatment in a patient with a sprain. What is included in teaching about this
home care? (Select all that apply.)
a. An elastic bandage is enough for compression.
b. Apply ice packs for 20 minutes three times daily.
c. Proximal joints should be elevated higher than distal joints.
d. The injured extremity should be raised above the level of the heart.
e. Place a cloth between the ice pack and the skin.
ANS: A, D, E
There is good evidence supporting use of an economical elastic bandage for support and
compression. Elevation of the extremity above the level of the heart after the injury helps to
reduce the edema and thereby decrease initial pain. If using an ice pack, a cloth should be
placed between the ice pack and the skin to prevent cold burn. Ice packs should be applied at
20-minute intervals, allowing the skin to return to normal temperature before each application.
Distal joints should be higher than proximal joints.

Chapter 169: Elbow Pain


1. A patient reports elbow pain and the examiner elicits pain with resisted wrist flexion, forearm
pronation, and passive wrist extension on the affected side. What is a likely cause of this pain?

c. Medial epicondylitis
Medial epicondylitis will produce pain as described above. Lateral epicondylitis may result in
pain with passive wrist flexion and active wrist extension.

1. A patient has chronic elbow pain associated with arthritis. What is included in management of
this condition? (Select all that apply.)
a. Avoidance of certain activities
b. Balanced rest and exercise
d. Occupational therapy
Patients with arthritis may be managed by avoiding pain-causing activities, a program of
balanced rest and exercise, and occupational therapy to improve function. NSAIDs are used
for short periods. Splinting is not recommended.

Chapter 170: Hand and Wrist Pain


1. A 40-year-old woman reports pain at the thumb base in one hand radiating to the distal radius.
The provider learns that the woman knits for a hobby and is able to elicit the pain by asking
the patient to pour water from a pitcher. Which condition is suspected in this patient?
c. De Quervain’s Tenosynovitis
De Quervain’s tenosynovitis causes pain as described and occurs more in women between 30
and 59 years who engage in activities requiring excessive repetitive motions, such as knitting.
Carpal tunnel syndrome presents with intermittent wrist pain, numbness, and tingling
radiating from the palm to the thumb, index finger, middle finger, and medial aspect of the
ring finger. Palmar fibrosis causes contractures, usually of the ring finger. Trigger finger
causes nodules in tendons that catch on the finger pulley and impede movement.

2. A patient reports numbness and weakness of the wrist with pain focuses on the radial aspect
of the joint. During physical examination, what will the examiner do to help diagnose this
condition?
a. Applying press to the focus area
Applying pressure to the volar or radial aspect of the wrist will elicit pain in patients with a
ganglion cyst which has symptoms described above. Flexing the thumb while the examiner
places a finger on the metacarpophalangeal joint will elicit a pop when the digit is extended in
patients with trigger finger. Puckering of the skin occurs with palmar fibrosis. Placing the
patient’s thumb on the palm while deviating the hand toward the ulna will elicit pain in
patients with tenosynovitis.
MULTIPLE RESPONSE
1. A patient has symptoms of carpal tunnel syndrome. Which diagnostic tests will help confirm
this disorder? (Select all that apply.)
b. Electromyography d. Nerve conduction studies
While diagnosis may be made on history and physical findings, electromyography and nerve
conduction studies can be helpful to confirm or exclude carpal tunnel syndrome. ANA and
ESR testing are useful when rheumatoid arthritis is suspected. Plain radiographs are not
useful.

Chapter 171: Amyotrophic Lateral Sclerosis


1. A patient with a family history of amyotrophic lateral sclerosis (ALS) begins to have
symptoms that include asymmetric weakness in the arms and difficulty walking. The
neurologist recognizes these symptoms as characteristic of involvement of which portion of
the nervous system?
a. Lower motor neurons (LMN)
Lower motor neuron involvement and early LMN cell death leads to an insidious onset of
asymmetric weakness that is evident initially in the limbs, usually in the arms. Upper motor
neuron cell death may result in hyperreflexia, spasticity, incoordination, and weakness. Bulbar
signs include dysarthria, dysphagia, and tongue fasciculations. The corticospinal tracts are
part of the UMN cells.

2. The spouse of a patient newly diagnosed with amyotrophic lateral sclerosis (ALS) asks about
long-term care. What will the provider include when teaching the family about this disease?
c. Preventing malnutrition is a key element in care.
Prevention of malnutrition may improve both the quality and length of life. Bowel and bladder
function and sensation remain intact. Positive-pressure ventilation helps to relieve sleep
disturbance.

Chapter 172: Bell’s Palsy


1. A 35-year-old patient reports suddenly experiencing an asymmetric smile along with drooping
and tearing in one eye. The patient has a history of a recent viral illness but is otherwise
healthy. During the exam, the provider notes that there is unilateral full-face paralysis on the
right side. What is the initial intervention for this patient?
b. Prescribe oral corticosteroids.
Steroids are highly effective and increase the probability of complete nerve recovery and
should be started within 72 hours of onset. There are no confirmatory diagnostic tests, but
other tests may be performed to rule out certain causes. Patients may be instructed to tape the
eye closed at night, but eye patches are not recommended. A neurology referral is needed only
if patients have an atypical presentation or other comorbid conditions.

2. What is recommended to prevent ophthalmic complications in patients with Bell’s palsy?


b. Lubricating eye drops
Exposure keratitis from drying of the eye can result in blindness. Lubricating eye drops should
be used every 2 hours. Acupuncture has not been sufficiently studied. Patching is not
recommended. Protective eyewear to prevent moisture loss is recommended.
MULTIPLE RESPONSE
1. Which symptoms may occur with Bell’s palsy? (Select all that apply.)
a. Alteration in taste
c. Drooling
e. Tinnitus
Bell’s palsy may cause altered taste, drooling, and tinnitus. It causes increased sensitivity to
noises and an inability to close the eye.

Chapter 173: Cerebrovascular Events


Buttaro: Primary Care: A Collaborative Practice, 6th Edition
MULTIPLE CHOICE
1. A patient exhibits visual field defect, ataxia, and dysarthria and complains of a mild headache.
A family member reports that the symptoms began several hours prior. An examination
reveals normal range of motion of the neck. What type of cerebrovascular event is most
likely?
a. Hemorrhagic stroke
b. Hypertensive intracerebral hemorrhage
c. Ischemic stroke
d. Transient ischemic attack (TIA)
ANS: C
Patients with ischemic stroke typically do not have headache; if they do, it is milder than with
hemorrhagic stroke. A TIA resolves within minutes.

2. An elderly patient is brought to the emergency department after being found on the floor after
a fall. The patient has unilateral sagging of the face, marked slurring of the speech, and
paralysis on one side of the body. The patient’s blood pressure is 220/190 mm Hg. What is the
likely treatment for this patient?
a. Carotid endarterectomy
b. Close observation until symptoms resolve
c. Neurosurgical consultation
d. Thrombolytic therapy
ANS: C
This patient has signs consistent with hemorrhagic stroke and will need consultation with a
neurosurgeon to determine whether surgical intervention will be beneficial. Carotid
endarterectomy is performed in patients with carotid stenosis and is used in patients with
hemispheric ACVS (TIA). Patients with TIA may be observed to monitor symptoms.
Thrombolytic therapy is given to patients with ischemic stroke.

3. A previously healthy 30-year-old patient is brought to the emergency department with signs of
stroke. Diagnostic testing determines an ongoing ischemic cause. The patient’s spouse reports
that symptoms began approximately 2 hours prior to transport. What is the recommended
treatment?
a. Administration of low-molecular-weight heparin
b. Neurosurgical consultation for possible surgery
c. Observation for complications prior to initiating tPA
d. Tissue plasminogen activator (tPA) administration
ANS: D
This patient meets the criteria for tPA administration and it should be begun within 4.5 hours
after onset of symptoms. This patient has had symptoms for over 2 hours, so tPA should begin
immediately. LMW heparin is not indicated. Neurosurgical intervention is recommended for
patients with hemorrhagic stroke.

Chapter 174: Dementia


Buttaro: Primary Care: A Collaborative Practice, 6th Edition
MULTIPLE CHOICE
1. A previously lucid patient with early-stage Alzheimer’s disease is hospitalized after a surgical
procedure and exhibits distractibility and perceptual disturbances that occur only in the late
afternoon. The patient has difficulty sleeping at night and instead sleeps much of the morning.
What is the likely cause of these symptoms?
a. Hyperactive delirium
b. Hypoactive delirium
c. Sundowner syndrome
d. Worsening dementia

ANS: C
Patients with dementia are at increased risk of sundowner syndrome, characterized by the
symptoms above and which typically appear in late afternoon and early evening. Hyperactive
delirium is manifested by agitation and restlessness. Hypoactive delirium includes patients
with decreased alertness, lethargy, and slowed speech. Delirium and worsening of dementia
would cause symptoms around the clock, not just in the late afternoon or evening.

2. An 80-year-old patient becomes apathetic, with decreased alertness and a slowing of speech
several days after hip replacement surgery alternating with long periods of lucidity. What is
the most likely cause of these symptoms?
a. Anesthesia effects
b. Delirium
c. Pain medications
d. Stroke

ANS: B
An acute presentation of these symptoms is most likely delirium since they alternate with
lucid periods. The other causes may contribute to delirium by intensifying it.

3. An elderly patient has symptoms of depression and the patient’s daughter asks about possible
Alzheimer’s disease (AD) since there is a family history of this disease. A screening
evaluation shows no memory loss. What is the initial step in managing this patient?
a. Order brain imaging studies such as CT or MRI.
b. Perform genetic testing to identify true risk.
c. Prescribe a trial of an antidepressant medication.
d. Recommend a trial of a cholinesterase inhibitor drug.

ANS: C
Elderly patients with depression who do not have other signs of AD may be given a trial of
antidepressant medications initially in order to evaluate these symptoms. Brain imaging
studies are not indicated initially. Genetic testing is not indicated. Once the degree of
depression is determined and if other symptoms appear, an anticholinesterase inhibitor may be
ordered.

4. A patient with dementia experiences agitation and visual hallucinations and is given
haloperidol with a subsequent worsening of symptoms. Based on this response, what is the
likely cause of this patient’s symptoms?
a. Alzheimer’s disease
b. Lewy body dementia
c. Pseudodementia
d. Vascular neurocognitive disorder

ANS: B
Patients with Lewy body dementia may present with these symptoms and will have an
increased sensitivity to neuroleptics; when given haloperidol for agitation, will actually
worsen. The other causes do not have these characteristic symptoms and are not sensitive to
neuroleptics in this manner.

5. A patient with Alzheimer’s disease (AD) is taking donepezil to treat cognitive symptoms. The
patient’s son reports noting increased social withdrawal and sleep impairment. What is the
initial step to manage these symptoms?
a. Encourage activity and exercise.
b. Prescribe a selective serotonin reuptake inhibitor (SSRI).
c. Recommend risperidone.
d. Referral to a neurologist for evaluation.

ANS: A
Patients with AD may have improvement in depression with nonpharmacologic management,
including exercise and increased activity. If this is not effective, an SSRI may be prescribed.
Risperidone and other antipsychotics should not be prescribed.

MULTIPLE RESPONSE
1. What are initial approaches when managing delirium in a hospitalized patient who is agitated
and confused? (Select all that apply.)
a. Administer medications for sleep.
b. Apply physical restraints.
c. Attend to hydration and toileting needs.
d. Decrease stimulation.
e. Discontinue any non-essential medications.

ANS: C, D, E
Patients with delirium should be assisted with hydration and toileting needs. Stimulation
should be decreased. Any non-essential medications should be discontinued. Giving
medications for sleep may exacerbate the delirium. Physical restraints should be avoided
unless necessary for safety.

Chapter 175: Dizziness and Vertigo


Buttaro: Primary Care: A Collaborative Practice, 6th Edition
MULTIPLE CHOICE
1. A patient reports a recurrent sensation of spinning associated with nausea and vomiting.
Which test will the provider order to confirm a diagnosis for this patient?
a. Electroencephalogram (ECG)
b. Holter monitoring and electrocardiogram
c. Neuroimaging with computerized tomography (CT)
d. The Hallpike-Dix positioning maneuver
ANS: D
This patient has symptoms consistent with a vestibular lesion, so the provider will order a
Hallpike-Dix positioning maneuver to evaluate vestibular function. If seizure activity is
suspected, an electroencephalogram will be ordered. Holter monitoring and ECG are used if
patients report syncope or lightheadedness. Neuroimaging with CT is used when patients
possibly have a central lesion which would present with difficulty balancing.
2. An older adult patient reports sensations of being off balance when walking but does not
experience dizziness. The provider will refer this patient to which specialist for further
evaluation?
a. Audiologist
b. Cardiologist
c. Neurologist
d. Otolaryngologist
ANS: C
This patient has problems of balance without dizziness, suggestive of a central neural lesion
and should be referred to a neurologist. Patients with vertigo are likely to have vestibular
dysfunction and would be referred to an otolaryngologist and possibly an audiologist if
hearing is affected. Patients with syncope or lightheadedness are more likely to have an
underlying cardiac disorder and would be referred to a cardiologist.

Chapter 176: Guillain-Barré Syndrome


Buttaro: Primary Care: A Collaborative Practice, 6th Edition
MULTIPLE CHOICE
1. Following an upper respiratory infection, a patient begins to develop ataxia and distal
paresthesias, along with oculomotor symptoms and double vision. Based on these presenting
symptoms which type of Guillain-Barré syndrome (GBS) does this patient have?
a. Acute inflammatory demyelinating polyradiculoneuropathy (AIDP)
b. Acute motor axonal neuropathy (AMAN)
c. Classic Guillain-Barré syndrome
d. Miller Fisher syndrome (MFS)
ANS: D
Miller Fisher syndrome has oculomotor symptoms. Patients with this type tend to peak sooner
and recover more completely and quickly.
2. Which diagnostic test helps confirm a diagnosis of Guillain-Barré syndrome (GBS) in a
patient who is developing muscle weakness and paresthesias?
a. Lumbar puncture
b. MRI imaging
c. Nerve conduction studies
d. Screening for systemic infection
ANS: A
A lumbar puncture is the most important confirmatory test showing albuminocytologic
disassociation. MRI imaging typically is not necessary unless there is concern for spine
pathology but does not diagnose GBS. Nerve conduction studies are not necessary for the
diagnosis. Screening for systemic infection is based on history and does not diagnose GBS.
MULTIPLE RESPONSE
1. Which monitoring parameters are necessary when caring for a patient with Guillain-Barré
syndrome (GBS)? (Select all that apply.)
a. Bladder scans
b. Cardiac telemetry
c. Imaging studies
d. Fever
e. Vital capacity measures
ANS: A, B, D, E
Urinary retention can cause discomfort and infection, so assessment of urinary retention is
necessary. Cardiac telemetry is essential, as are measures of pulmonary function. Imaging
studies are not essential.

Chapter 177: Headache


Buttaro: Primary Care: A Collaborative Practice, 6th Edition
MULTIPLE CHOICE
1. A patient reports recurrent headaches occurring 1 or 2 times per month that generally occur
with weather changes or when sleep patterns are disrupted. They are described as severe, with
throbbing on one side of the head and sometimes accompanied by nausea. What is the
recommended abortive treatment for this type of headache?
a. Gabapentin
b. Propranolol
c. Ergotamine tartrate
d. Topiramate
ANS: C
This patient describes migraine headache without aura and has fewer than 4 per month. An
abortive medication, such as ergotamine tartrate, is recommended. The other medications are
preventive medications and are used for patients having more than 4 per month.
2. A patient has recurrent cluster headaches and asks about abortive therapy. Which therapy is
effective for most patients with cluster headaches?
a. Lithium
b. NSAIDs
c. Oxygen
d. Verapamil
ANS: C
Oxygen works as abortive therapy for cluster headaches in 75% of patients and should be
inhaled at the start of an attack. Lithium and verapamil work well as preventive medications
for cluster headaches but are not given for abortive treatment. NSAIDs are not useful.
MULTIPLE RESPONSE
1. Which medications may be useful in treating tension-type headache? (Select all that apply.)
a. Triptan drugs
b. Lithium
c. Muscle relaxants
d. NSAIDs
e. Oxygen
ANS: A, C, D
Triptan drugs, muscle relaxants, and NSAIDs may all be used to treat tension-type headaches.
Lithium and oxygen are not used.

Chapter 178: Infections of the Central Nervous System


Buttaro: Primary Care: A Collaborative Practice, 6th Edition
MULTIPLE CHOICE
1. A patient is brought to the emergency department with fever, lethargy, and headache. No
meningeal signs are noted. The examination reveals hypotension and lethargy and the
examiner notes petechiae on the patient’s trunk. What do these findings indicate?
a. Progressing meningococcemia
b. Encephalitis
c. Increased intracranial pressure (ICP)
d. Probable viral infection
ANS: A
Petechiae are an ominous sign, indicating a rapidly progressing meningococcemia. Patients
with encephalitis or other viral infection will not usually have petechiae and severe symptoms.
Increased ICP will present with hypertension.
2. A provider suspects that a patient has bacterial meningitis. When should antibiotics be given?
a. If the serum C-reactive protein is greater than 10 mg/L
b. Immediately after blood and spinal cultures are obtained
c. Prior to obtaining a computed tomography (CT) scan or lumbar puncture (LP)
d. When initial spinal fluid gram stain results are available
ANS: C
In all cases of suspected meningitis, the first dose of antimicrobials should be administered
immediately after blood cultures are obtained and prior to the CT and LP to avoid critical
delays in treatment. The CSF culture will still yield bacteria for 1 to 2 hours after the first
dose. Waiting for any laboratory results may delay effective treatment.

Chapter 179: Movement Disorders and Essential Tremor


Buttaro: Primary Care: A Collaborative Practice, 6th Edition
MULTIPLE CHOICE
1. A patient reports trembling of both hands causing difficulty performing tasks with the hands.
The provider notes symmetric, rhythmic movements which are present at rest and no other
neurological findings. A history reveals that the trembling decreases when the patient has a
glass of wine with dinner. What is the initial action?
a. Evaluation in an emergency department (ED)
b. Prescribing a beta blocker medication
c. Reassurance that these will subside
d. Referral to a neurologist
ANS: B
This patient has symptoms consistent with essential tremors. Reassurance may be the first
action, but the symptoms will not subside. Beta blockers are used when the tremor is
functionally or socially problematic. It is not necessary to refer to an ED or a specialist.
MULTIPLE RESPONSE
1. Which are common hyperkinetic movement disorders? (Select all that apply.)
a. Dystonia
b. Essential tremor
c. Parkinson’s disease
d. Progressive supranuclear palsy
e. Tourette’s syndrome
ANS: A, B, E
Dystonia, essential tremor, and Tourette’s are hyperkinetic disorders. Parkinson’s disease and
progressive supranuclear palsy are hypokinetic disorders.

Chapter 180: Multiple Sclerosis


1. A patient reports two episodes of visual disturbances and eye pain that lasted 1 to 2 days each
about 2 months apart. Which diagnostic testing will the provider order initially?
a. Lumbar puncture
b. Magnetic resonance imaging (MRI)
c. Optical Coherence Tomography (OCT)
d. Visual evoked potential
ANS: B
Visual disturbances and eye pain may be the only presenting symptoms and should be
investigated. The MRI is the gold standard for diagnosis of multiple sclerosis (MS). The other
tests may be performed if the diagnosis is unclear or if MRI is not readily available.
2. A patient diagnosed with multiple sclerosis and begins disease modulating therapy (DMT)
drugs. As part of the counseling about this therapy, the provider will tell the patient that this
regimen will likely result in what outcome?
c. A reduction in the exacerbation rate

Disease modulating therapy will reduce the rate of exacerbations of symptoms. It does not
decrease the need for other medications, induce long-term remission, or stop the progression
of the disease.

Chapter 181: Parkinson’s Disease


Buttaro: Primary Care: A Collaborative Practice, 6th Edition
MULTIPLE CHOICE
1. A primary care provider suspects Parkinson’s disease (PD) in a patient. Which tests may be
performed to diagnose this disorder?
a. Neuroimaging to identify specific midbrain lesion
b. Neuromuscular studies to identify reflex function
c. Presence of 2 cardinal signs which improve with levodopa
d. Serum creatine phosphokinase levels
ANS: C
The diagnosis of idiopathic PD is made based on clinical presentation and examination
findings with 2 of 3 cardinal manifestations present which respond to dopaminergic therapy.
Neuroimaging that identifies Lewy bodies is the gold standard but is performed post-mortem.
Neuromuscular studies and serum laboratory studies are not useful unless excluding other
causes of symptoms.
2. A patient diagnosed with Parkinson’s disease (PD) has been prescribed carbidopa-levodopa
with good results but develops increased dyskinesia. Which drug will be added to this
patient’s regimen to help control this symptom?
a. Amantadine
b. Benztropine
c. Ropinirole
d. Tolcapone
ANS: A
Amantadine is an antiviral agent that has antiparkinsonian activity. It is useful for controlling
dyskinesia as adjunctive therapy to levodopa and is more effective than anticholinergic drugs.
Benztropine is an anticholinergic drug that may be used for this purpose but is less effective
and is more commonly used to treat antipsychotic-induced parkinsonism. Ropinirole is used
as a first-line agent in patients with young-onset PD. Tolcapone is a COMT agent to prolong
and potentiate the effects of levodopa to help prevent “wearing off” periods.
MULTIPLE RESPONSE
1. What are common symptoms noted in patients diagnosed with Parkinson’s disease? (Select all
that apply.)
a. Bradykinesia
b. Festination
c. Hyperphonia
d. Rigidity
e. Symmetric tremor
ANS: A, B, D
Symptoms of Parkinson’s disease include bradykinesia, or loss of automatic movement,
festination, or an impulse to take much quicker and shorter steps, and rigidity. Hypophonia,
not hyperphonia occurs. Tremors are unilateral or asymmetric.

Chapter 182: Seizure Disorder


Buttaro: Primary Care: A Collaborative Practice, 6th Edition
MULTIPLE CHOICE
1. A patient with a seizure disorder has seizures which begin with a gastric sensation and a
feeling of déjà vu. Which site in the brain is the seizure focus?
a. Frontal
b. Occipital
c. Parietal
d. Temporal
ANS: D
Temporal sites cause epigastric and déjà vu sensations. Occipital sites causing complex partial
seizures will have visual auras that may begin with eye twitching and visual hallucinations.
Frontal sites cause dizziness or fear. Parietal sites cause sensory changes, such as numbness,
tingling, or pain.
2. A patient who has a seizure disorder and takes levetiracetam is brought to an emergency
department as two consecutive 15-minute seizures occur. What is the priority action for this
patient?
a. Administer a dose of levetiracetam now and repeat in 10 minutes.
b. Administer lorazepam and monitor cardiorespiratory status.
c. Administer phenytoin and phenobarbital along with oxygen.
d. Admit the patient to the hospital for a diagnostic work up.
ANS: B
This patient has status epilepticus, which should be treated with benzodiazepines and close
monitoring of airway, breathing, and circulation. The other interventions are not appropriate
for acute seizure management.
3. Which drug is used to treat patients with focal epilepsy and complex partial seizures?
a. Carbamazepine
b. Ethosuximide
c. Lamotrigine
d. Topiramate
ANS: A
Carbamazepine is used for focal and complex partial seizures. Ethosuximide is useful for petit
mal seizures in children. Lamotrigine has a wide range of effectiveness but has an increased
risk for Stevens-Johnson syndrome. Topiramate is not a first-line drug because of cognitive
side effects.

Chapter 183: Trigeminal Neuralgia


1. A patient reports paroxysms of burning, shock-like pain on both sides of the face usually
triggered by chewing or talking. The provider suspects trigeminal neuralgia. Based on these
presenting symptoms, what testing is indicated?
c. Magnetic resonance imaging (MRI)
Trigeminal neuralgia is a clinical diagnosis. Pain on both sides of the face raises a suspicion
for multiple sclerosis and MRI is done to corroborate the presence of MS. Autoimmune
laboratory pane is performed if alternative diagnoses are suspected. Inflammatory markers are
not diagnostic. Plain radiographs are not indicated.

2. A patient is diagnosed with trigeminal neuralgia and reports having paroxysms several times
each day. What is the initial treatment for this patient?
b. A high dose of carbamazepine with subsequent titration downward

Anticonvulsants are first-line treatments for trigeminal neuralgia – carbamazepine is started at


the maximum therapeutic dose and titrated down to the lowest effective dose. Combination
drug therapy is begun if the initial treatment is not effective or if the single drug regimen has
intolerable side effects. Botox injections and intranasal lidocaine are used as adjuncts to
anticonvulsants for acute pain relief.

3. A patient diagnosed with trigeminal neuralgia has tried several medication regimens to control
pain without success. What is the next step in management for this condition?
d. Referral to a neurosurgeon

Referral to a neurosurgeon is indicated after medical therapies have been exhausted. The other
options may be included in long-term care, but a neurosurgery referral is warranted.

Chapter 184: Intracranial Tumors


1. A patient develops a gait disorder and the patient’s spouse reports noticing recent personality
changes. The provider suspects a brain lesion. Which evaluation is especially important in the
initial physical examination?
c. Examination of the optic fundi

Gait disorders and personality changes are more typical presentations with nonfocal lesions.
Examination of the optic fundi for papilledema may be the only finding to indicate increased
intracranial pressure. The other assessments help determine focal involvement.

2. Which is the preferred treatment for primary brain tumors?


d. Surgical resection

Surgical resection is the most effective treatment for brain tumors. Chemotherapy is limited in
effectiveness because of difficulty crossing the blood-brain barrier. Radiation therapy is used
as a primary, adjuvant, or palliative therapy. Palliative care is not the preferred treatment;
many patients with brain tumors live for many years.

Chapter 185: Adrenal Gland Disorders


Buttaro: Primary Care: A Collaborative Practice, 6th Edition
MULTIPLE CHOICE
1. A patient has new-onset hypertension with a systolic blood pressure of 180 mm Hg. Which
test will the provider order to diagnose this patient?
a. ACTH suppression testing
b. Adrenal antibody tests
c. Cortisol excretion studies
d. Fractionated metanephrine levels
ANS: D
Patients with pheochromocytoma may present with new-onset hypertension with systolic
pressure >170 mm Hg. Fractionated metanephrine will be elevated when the diagnosis is
confirmed. ACTH suppression testing and cortisol excretion studies are performed to
diagnose Cushing’s syndrome. Adrenal antibody tests are performed as part of the evaluation
for Addison’s disease.
2. A patient has rapid weight gain, amenorrhea without pregnancy, and mild hypertension. Once
confirmatory tests are performed, what is a possible treatment for this patient?
a. Antihypertensive therapy
b. Mineralocorticoid replacement
c. Oral hydrocortisone
d. Pituitary tumor resection
ANS: D
This patient has symptoms of Cushing’s syndrome. When indicated, pituitary tumor resection
is performed as the first choice. Antihypertensive therapy is initiated in patients with
pheochromocytoma. Mineralocorticoids and glucocorticoids are given to patients with
Addison’s disease.
MULTIPLE RESPONSE
1. A patient has unexplained weight loss and the provider notes increased skin pigmentation on
light-exposed skin folds along with darkened palmar creases. Which laboratory tests will the
provider order? (Select all that apply.)
a. Serum ACTH
b. Serum cortisol
c. Serum electrolytes
d. TB skin testing
e. Urine cortisol
ANS: A, C, D
This patient has symptoms of Addison’s disease. Serum ACTH will be elevated in patients
with Addison’s disease. Hyponatremia and hyperkalemia may occur and are sometimes the
initial finding. TB skin testing is done to exclude tuberculosis. Serum and urine cortisol levels
are evaluated with Cushing’s syndrome is suspected.

Chapter 186: Diabetes Mellitus


Buttaro: Primary Care: A Collaborative Practice, 6th Edition
MULTIPLE CHOICE
1. An obese patient has recurrent urinary tract infections and reports feeling tired most of the
time. What initial diagnostic test will the provider order in the clinic at this visit?
a. C-peptide level
b. Hemoglobin A1C
c. Random serum glucose
d. Thyroid studies
ANS: B
HbA1C, along with fasting plasma glucose or a 2-hour plasma glucose during an oral glucose
tolerance test (OGTT), is diagnostic of diabetes. This patient is probably not fasting, so a
glucose level will not be helpful. C-peptide levels help to distinguish type 1 from type 2
diabetes and may be performed after a diagnosis of diabetes is made and if there is uncertainty
about the cause. Thyroid studies are helpful in evaluating comorbidity.

2. A patient diagnosed with type 2 diabetes mellitus becomes insulin dependent after a year of
therapy with oral diabetes medications. When explaining this change in therapy, the provider
will tell the patient
a. it is necessary because the patient cannot comply with the previous regimen.
b. that strict diet and exercise measures may be relaxed with insulin therapy.
c. the use of insulin therapy may be temporary.
d. this is because of the natural progression of the disease.
ANS: D
Even after several years of therapy for type 2 DM well controlled with oral diabetic
medications, diet, and exercise, the natural progression of the disease may require patients to
become insulin dependent. Patients must understand that this does not represent failure on
their part. Adding insulin may cause weight gain, so continuing lifestyle measures is essential.
The addition of insulin is not temporary.

3. A patient diagnosed with diabetes has a blood pressure of 140/90 mm Hg and albuminuria.
Which initial action by the primary care provider is indicated for management of this patient?
a. Consulting with a nephrologist
b. Limiting protein intake
c. Prescribing an antihypertensive medication
d. Referring to an ophthalmologist
ANS: A
Patients with diabetes who have elevated blood pressure and reduced renal function should be
referred to a nephrologist. Limiting protein intake and giving an antihypertensive medication
may be recommended, but evaluation by a nephrologist is essential. Ophthalmology referral
will be made as well to assess concurrent ocular damage.

4. A patient recently diagnosed with type 1 diabetes mellitus is in clinic for a follow-up
evaluation. The provider notes that the patient appears confused and irritable and is sweating
and shaking. What intervention will the provider expect to perform once the point of care
blood glucose level is known?
a. Dipstick urinalysis for ketones
b. Giving a rapid-acting carbohydrate
c. Injection of rapid-acting insulin
d. Performing a hemoglobin A1C
ANS: B
This patient has signs of hypoglycemia, so a rapid-acting carbohydrate should be given once
this is confirmed. Assessing for ketones is done if the patient is hyperglycemic, as is insulin
administration. Hemoglobin A1C gives information about long-term and not immediate
glucose control.

Chapter 187: Hirsutism


Buttaro: Primary Care: A Collaborative Practice, 6th Edition
MULTIPLE CHOICE
1. An obese adolescent female patient reports irregular periods and excessive acne. The provider
notes an increased amount of hair on her upper back, shoulders, and upper abdomen. What
will the provider do, based on these findings?
d. Refer to an endocrinologist for evaluation

All patients with suspected hirsutism should be referred to a specialist to determine the cause.
OCPs, lifestyle changes, and cosmetic treatments may be part of the treatment, but the
underlying causes must be determined first to ensure that a life-threatening condition is not
present.

2. A young adult woman is unable to conceive after trying to get pregnant for over 6 months.
The woman reports having had irregular periods since the onset of menarche. The provider
notes that the woman is overweight, has acanthosis nigricans, and an excess hair distribution.
What does the provider suspect as the most likely primary cause of these symptoms?
c. Polycystic ovary syndrome (PCOS)

PCOS is the most likely cause of oligo- or amenorrhea, so this is the most likely cause. The
other conditions are possible, but less likely.

3. A woman who has hirsutism with acne, and oligomenorrhea will most likely be treated with
which medication to control these symptoms?
c. Norgestimate

Norgestimate is a progestin with low androgenic activity and is used to suppress testosterone
and control symptoms. Finasteride, which decreases the peripheral conversion of testosterone
to dihydrotestosterone (DHT), is not approved for this use. Levonorgestrel is an androgenic
oral contraceptive pill (OCP) and should be avoided. Spironolactone is a second-line
medication approved for this purpose.

Chapter 188: Hypercalcemia and Hypocalcemia


Buttaro: Primary Care: A Collaborative Practice, 6th Edition
MULTIPLE CHOICE
1. A patient is in the emergency department with confusion and fatigue and a corrected serum
calcium concentration is 10.8 mg/dL. What is the initial treatment for this patient prior to
admission to the inpatient unit?
a. Administration of furosemide
b. Correction of potassium and magnesium levels
c. Parenteral salmon calcitonin
d. Rapid administration of intravenous normal saline
ANS: D
To help the kidneys excrete calcium, intravenous normal saline should be given initially.
Furosemide may not be effective as once thought and is used less often today. Correction of
other electrolytes may be done when these imbalances are assessed. Parenteral salmon
calcitonin may be used later to enhance calcium losses.

2. A patient experiences a carpal spasm when a blood pressure cuff is inflated. Which diagnostic
testing will the provider consider evaluating to determine the cause of this finding?
a. Calcitriol level
b. C-reactive protein
c. Magnesium and vitamin D
d. Protein electrophoresis
ANS: C
The Trousseau’s sign indicates neuromuscular irritability, which occurs with hypocalcemia.
Because hypomagnesemia and vitamin D deficiency may cause hypocalcemia, these should
be evaluated to help determine a cause. Calcitriol levels are used to assess hypercalcemia.
Inflammatory markers are not indicated. Protein electrophoresis is used in the evaluation of
hypercalcemia.

3. A patient has low serum calcium associated with low serum albumin. What is the
recommended treatment for this patient?
a. Calcium supplementation only
b. Correction of other serum electrolytes
c. Thiazide diuretics and sodium restriction
d. Vitamin D and calcium supplementation
ANS: B
Patients with hypocalcemia associated with hypoalbuminemia do not require calcium
replacement. Serum pH, potassium, magnesium, and phosphorus levels should be monitored
and corrected if needed. Thiazide diuretics with sodium restriction may be used to lower
urinary calcium excretion to allow lower dosing of calcium and vitamin D when these are
given.

Chapter 189: Hyperkalemia and Hypokalemia


Buttaro: Primary Care: A Collaborative Practice, 6th Edition
MULTIPLE CHOICE
1. A patient has a serum potassium level of 3 mEq/L and a normal blood pressure. Which test
should be performed initially to assist with the differential diagnosis?
a. Plasma aldosterone
b. Plasma renin activity
c. Serum bicarbonate
d. Serum magnesium
ANS: D
Hypomagnesemia often accompanies hypokalemia indicating the importance of also obtaining
a serum magnesium level. Sodium bicarbonate is occasionally used in the treatment of
hyperkalemia and is most effective when hyperkalemia is a result of metabolic acidosis.
Plasma aldosterone and renin activity are assessed in patients with hypokalemia who are
hypertensive.

2. A patient with normal renal function has a potassium level of 6.0 mEq/L. Which underlying
cause is possible in this patient?
a. Adrenocortical deficiency
b. Alcoholism
c. Hypertension
d. Malabsorption syndrome
ANS: A
Hyperkalemia without underlying renal disorder may be caused by Addison’s disease, which
is an adrenocortical deficiency. Alcoholism, hypertension, and malabsorption syndromes all
contribute to hypokalemia.

3. A hospitalized patient with renal failure is accidentally given parenteral potassium and has a
potassium level of 7.0 mEq/L. An ECG reveals a normal QRS interval. What is the initial
recommended treatment for this patient?
a. Calcium chloride
b. Insulin and glucose infusion
c. Sodium bicarbonate
d. Sodium polystyrene sulfate
ANS: B
Patients with severe hyperkalemia should have IV administration of glucose and insulin to
lower potassium levels quickly. If life-threatening sequelae, such as a widening QRS interval,
are present, calcium chloride is given. Sodium bicarbonate is occasionally used, but should be
used cautiously to prevent metabolic alkalosis. Sodium polystyrene sulfate is used when oral
medications may be given.

Chapter 190: Hypernatremia and Hyponatremia


Buttaro: Primary Care: A Collaborative Practice, 6th Edition
MULTIPLE CHOICE
1. A high school athlete is brought to the emergency department after collapsing during outdoor
practice on a hot day. The patient is weak, irritable, and confused. Serum sodium is 152
mEq/L and has dry mucous membranes and tachycardia. What is the initial approach to
rehydration in this patient?
a. Hypotonic intravenous fluid replacement
b. Intravenous fluid resuscitation with an isotonic solution
c. Loop diuretics and hypotonic intravenous fluids
d. Oral water replacement
ANS: B
This patient is dehydrated and has hypernatremia because of heat exposure and sweating.
Because the patient is confused, oral fluid replacement is not recommended, although it is the
safest in cognitively intact patients who are able to swallow safely. The initial fluids should
correct the hypovolemia and isotonic solutions such as normal saline (0.9%) or Ringer’s
lactate are given. Hypotonic fluids are then given once vital signs and urine output have
normalized in patients with hypernatremia caused by fluid loss. Loop diuretics are added for
patients who have hypernatremia caused by sodium gain.

2. An elderly patient who is taking a thiazide diuretic has been ill with nausea and vomiting and
is brought to the emergency department for evaluation. An assessment reveals oliguria,
hypotension, and tachycardia and serum sodium is 118 mEq/L. What is the treatment?
a. A single infusion of hypertonic saline
b. Addition of spironolactone
c. Emergency volume repletion with 3% NaCl.
d. Fluid and dietary sodium restriction
ANS: C
This patient has hypovolemic hyponatremia with a sodium less than 120 mEq/L and requires
fluid resuscitation with 3% NaCl. Diuretics and fluid restriction are part of treatment for
hypervolemic hyponatremia.

3. A patient has euvolemic hyponatremia secondary to chronic syndrome of inappropriate


antidiuretic hormone (SIADH) and is hospitalized for fluid replacement. When preparing to
discharge the patient home, what will be included in teaching?
a. Limiting dietary protein intake
b. Limiting fluids to 500 mL/day for several days
c. Restriction of sodium intake
d. The importance of adherence to vaptan therapy
ANS: B
Patients with chronic hypovolemia secondary to SIADH require fluid restriction for several
days. Sodium and protein are not restricted. Vaptan therapy is started for those whose serum
sodium fails to normalize in 24 to 48 hours.

Chapter 191: Lipid Disorders


Buttaro: Primary Care: A Collaborative Practice, 6th Edition
MULTIPLE CHOICE
1. When using the 2013 ACC/AHA “Guideline on the Assessment of Cardiovascular Risk” to
treat patients with hyperlipidemia, the practitioner understands that it will provide what
information?
a. Goals for treatment for low-density lipoprotein levels
b. How to titrate statin drugs to achieve goal levels
c. Use of non-statin therapy for primary prevention
d. Which patients will benefit from statin therapy
ANS: D
The 2013 guidelines identify four groups of patients who will benefit from statin therapy to
lower low-density lipoprotein cholesterol (LDL-C). The guidelines do not identify goal levels
for treatment, do not recommend titration of statin drugs to achieve results, and do not
recommend non-statin therapies for primary prevention.

2. A patient with type 2 diabetes has a low-density lipoprotein (LDL) level of 110 gm/dL. What
is recommended to manage this patient?
a. Dietary and lifestyle changes to modify risk
b. Initial treatment with a low intensity statin medication
c. Prescription of a moderate or high intensity statin
d. Statin therapy until the LDL level is below 75 mg/dL
ANS: C
This patient is in one of the four groups of patients identified in current guidelines as one who
would benefit from statin therapy because of type 2 diabetes. A moderate to high intensity
statin should be prescribed. Statins will be used in conjunction with dietary and lifestyle
changes, but these treatments alone do not reduce risk in this patient. Titration of statins is not
recommended, and goal levels are no longer part of the protocol.

3. A patient who is taking a statin drug to treat dyslipidemia has begun a diet and exercise
program. The patient reports new onset of muscle pain several weeks after beginning therapy.
What is the initial action by the provider?
a. Discontinue the statin drug immediately
b. Obtain a creatine kinase level
c. Prescribe acetaminophen or ibuprofen
d. Recommend reducing exercise intensity
ANS: B
A potential serious side effect of statin drugs is drug-induced myopathy. Patients who report
new-onset muscle pain should have creatine kinase levels evaluated. If this is elevated, the
drug should be stopped, and renal function should be evaluated. It is not safe to assume that
the muscle pain is related to the exercise until CK levels are determined.

Chapter 192: Metabolic Syndrome


Buttaro: Primary Care: A Collaborative Practice, 6th Edition
MULTIPLE CHOICE
1. What is important about increased PAI-1 levels in patients?
a. They cause increased insulin resistance.
b. They are associated with metabolic syndrome.
c. They lower the risk of hypertension.
d. They predispose patients to dyslipidemia.
ANS: B
Increased PAI-1 levels increase the risk of atherothrombosis. They are correlated, but do not
cause insulin resistance and do not affect the relative risk of hypertension or dyslipidemia.

2. Which medication given for patients with metabolic syndrome is most likely to lower PAI-1
levels?
a. Aspirin
b. Atorvastatin
c. Metformin
d. Niacin
ANS: C
Metformin is given not only to reduce hyperinsulinemia and lower insulin resistance, but also
to lower plasma PAI-1 levels. Aspirin is given to reduce MI risk. Atorvastatin helps with
dyslipidemia. Niacin may be given to lower triglycerides.

MULTIPLE RESPONSE
1. Which findings are part of the diagnostic criteria for metabolic syndrome? (Select all that
apply.)
a. Decreased plasminogen activator inhibitor 1 levels
b. Elevated waist circumference
c. Fasting plasma glucose 􀁴 100 mg/dL
d. HDL cholesterol 􀁴􀀃45 mg/dL
e. Triglycerides 􀁴􀀃150 mg/dL
ANS: B, C, E
The current criteria for diagnosing metabolic syndrome include increased waist
circumference, elevated fasting plasma glucose, and elevated triglycerides. According to these
criteria, patients will have HDL levels <40 mg/dL. The old criteria included elevated
plasminogen activator inhibitor 1 levels.

Chapter 193: Parathyroid Gland Disorders


Buttaro: Primary Care: A Collaborative Practice, 6th Edition
MULTIPLE CHOICE
1. Which laboratory values representing parathyroid hormone (PTH) and serum calcium are
consistent with a diagnosis of primary hyperparathyroidism?
a. Appropriately high PTH along with hypocalcemia
b. Appropriately increased PTH and low or normal serum calcium
c. Inappropriate secretion of PTH along with hypercalcemia
d. Prolonged inappropriate secretion of PTH with subsequent hypercalcemia
ANS: C
Primary hyperparathyroidism is characterized by the inappropriate secretion of PTH in the
setting of hypercalcemia. Appropriately high PTH with hypocalcemia characterizes
hypoparathyroidism. An appropriately increased secretion of PTH with low or normal serum
calcium is characteristic of secondary hyperparathyroidism. Prolonged inappropriate secretion
of PTH in which hypercalcemia develops is tertiary hyperparathyroidism.

2. A 40-year-old patient with primary hyperparathyroidism has increased serum calcium 0.5
mg/dL above normal without signs of nephrolithiasis. What is the recommended treatment for
this patient?
a. Annual monitoring of calcium, creatinine, and bone density
b. Avoidance of weight-bearing exercises
c. Decreasing calcium and vitamin D intake until values normal
d. Parathyroidectomy
ANS: A
Medical management of primary hyperparathyroidism involves close monitoring of serum
calcium and creatinine and bone density screenings. Weight-bearing exercises should be
encouraged, and vitamin D and calcium intake should be adequate, not decreased. This patient
does not meet criteria for parathyroidectomy because of age less than 50 years and serum
calcium less than 1 mg/dL above the upper limit of normal.

MULTIPLE RESPONSE
1. Which findings are symptoms of hyperparathyroidism? (Select all that apply.)
a. Chvostek’s sign
b. Cognitive impairment
c. Left ventricular hypertrophy
d. Perioral paresthesias
e. Renal calculi
ANS: B, C, E
Cognitive impairment, left ventricular hypertrophy, and renal calculi all occur with
hyperparathyroidism. Chvostek’s sign and perioral paresthesias occur with
hypoparathyroidism.

Chapter 194: Thyroid Disorders


Buttaro: Primary Care: A Collaborative Practice, 6th Edition
MULTIPLE CHOICE
1. Which thyroid-stimulating hormone (TSH) level indicates hyperthyroidism?
a. 0.2 uIU/L
b. 0.4 uIU/L
c. 2.4 uIU/L
d. 4.2 uIU/L
ANS: A
A TSH less than 0.3 uIU/L indicates hyperthyroid; greater than 4.0 uIU/L indicates
hypothyroid, and between 0.3 and 4.0 uIU/L indicates euthyroid.

2. A patient has thyroid nodules and the provider suspects thyroid cancer. To evaluate thyroid
nodules for potential malignancy, which test is performed?
a. Radionucleotide imaging
b. Serum calcitonin
c. Serum TSH level
d. Thyroid ultrasound
ANS: D
Thyroid ultrasound evaluation should be performed for all patients with known thyroid
nodules; high-resolution sonography can clearly distinguish between solid and cystic
components. Radionucleotide imaging is not specific; many cold nodules are benign. The
routine measurement of serum calcitonin levels is not useful or cost-effective. TSH levels are
not specific to malignancy.

3. A 20-year-old female patient with tachycardia and weight loss but no optic symptoms presents
with the following laboratory values: decreased TSH, increased T3, and increased T4 and free
T4. A pregnancy test is negative. What is the initial treatment for this patient?
a. Beta blocker medications
b. Radioiodine therapy
c. Surgical resection of the thyroid gland
d. Thionamide therapy
ANS: A
Beta blockers should be initiated for patients with Graves’ disease to alleviate the
alpha-adrenergic symptoms of the hyperthyroidism. Radioiodine therapy is used for patients
with Graves’ ophthalmopathy. Surgical resection is performed for pregnant women who
cannot be managed with thioamides or for patients who refuse radioiodine therapy. Thioamide
therapy is recommended for patients younger than 20 years old, pregnant women, those with a
high likelihood of remission, and those with active Graves’ Orbitopathy.

4. A postpartum woman develops fatigue, weight gain, and constipation. Laboratory values
reveal elevated TSH and decreased T3 and T4 levels. What will the provider tell this patient?
d. This condition may be transient.

Postpartum hypothyroidism may be a transient condition and does not require surgical
intervention, referral to a specialist, or lifelong medication unless it proves to be long-standing
or refractory to treatment.
Chapter 195: Polymyalgia Rheumatica and Giant Cell Arteritis
Buttaro: Primary Care: A Collaborative Practice, 6th Edition
MULTIPLE CHOICE
1. A patient is diagnosed with polymyalgia rheumatica (PMR) with giant cell arteritis. Which
dose of prednisolone will be given initially?
a. 15 mg daily
b. 20 mg daily
c. 30 mg daily
d. 60 mg daily
ANS: D
Although the usual starting dose to treat PMR is 15 to 20 mg daily, a higher dose of 60 mg
daily is used when there is evidence of concomitant giant cell arteritis.

2. A patient who is taking prednisolone 20 mg daily to treat polymyalgia rheumatica reports


blurred vision. What will the provider do?
a. Discontinue the medication
b. Increase the prednisolone dose to 60 mg daily
c. Prescribe NSAIDs to treat the inflammation
d. Refer to a rheumatologist immediately
ANS: D
Sudden vision loss, diplopia, and other visual disturbances may indicate giant cell arteritis
(GCA) and requires immediate referral to rheumatology. The primary provider should not
change the medication regimen without a consult.

MULTIPLE RESPONSE
1. A 60-year-old patient reports new onset of bilateral shoulder pain with morning stiffness
lasting approximately 1 hour. Which will be included in initial diagnostic testing for this
patient? (Select all that apply.)
a. Antinuclear antibodies
b. Erythrocyte sedimentation rate (ESR) and C-reactive protein (CRP)
c. Liver function tests (LFTs)
d. Protein electrophoresis
e. Serum calcitonin
ANS: B, C, D
ESR, CRP, and protein electrophoresis are included in the initial diagnostic workup when
polymyalgia rheumatica is suspected. ANA testing is not specific for this disorder. Serum
calcitonin is not indicated.

Chapter 196: Raynaud’s Phenomenon


Buttaro: Primary Care: A Collaborative Practice, 6th Edition
MULTIPLE CHOICE
1. A provider performs a nail fold capillaroscopy on a patient who reports marked color changes
of both hands with cold exposure and notes tortuous and dilated capillary loops. This finding
is consistent with what condition?
a. Polymyositis
b. Primary Raynaud’s phenomena
c. Scleroderma
d. Secondary Raynaud’s phenomena
ANS: D
Microvascular abnormalities like tortuous of dilated capillary loops are observed in secondary
Raynaud’s phenomena and capillaroscopy is used to differentiate primary from secondary
Raynaud’s. These findings are not present with polymyositis or scleroderma.

2. A patient has secondary Raynaud’s phenomena with severe digital ischemia. Which treatment
is indicated for this patient?
a. Ginkgo biloba
b. Intravenous prostaglandin E1
c. Oral nifedipine
d. Sildenafil as needed
ANS: B
Intravenous prostaglandin E1 is reserved for patients with secondary Raynaud’s phenomenon
who have severe digital ischemia. Ginkgo biloba is associated with adverse effects and has not
been shown to be effective. Nifedipine is used to prevent vasospasm in milder cases.
Sildenafil may be used as a vasodilator in milder cases.

Chapter 197: Rheumatoid Arthritis


1. A patient has swelling and tenderness in the small joints of both hands and reports several
weeks of malaise and fatigue. A rheumatoid factor (RF) test is negative. What will the
primary care provider do next?
b. Order radiographic tests, a CBC, and acute-phase reactant levels
The patient has signs of rheumatoid arthritis (RA); the RF test may be negative initially but
will become positive in 70% to 80% of patients. The provider’s next step is to order tests to
confirm the diagnosis and to provide a baseline to monitor disease progress and response to
treatment. DMARDs may be ordered when the disease is confirmed. The PCP may treat in
consultation with a rheumatologist who will order medications and will refer the patient for
physical therapy, occupational therapy, and psychotherapy.
2. A patient is diagnosed with rheumatoid arthritis (RA) after a review of systems, confirmatory
lab tests, and synovial fluid analysis. What will the provider order initially to treat this
patient?
a. Disease-modifying anti- rheumatic (DMARDs) drugs
Treatment with DMARDs should be initiated as soon as the diagnosis of RA is established to
achieve disease modification. Long-term glucocorticoid therapy is not recommended because
of adverse effects. NSAIDs are not first-line drugs and increase the risk of cardiac and renal
complications. NSAIDs are used as adjunctive and not first-line therapy.

MULTIPLE RESPONSE
1. Which are symptoms of rheumatoid arthritis (RA) that distinguish it from osteoarthritis (OA)?
a. Extra-articular inflammatory signs
c. Morning stiffness of at least 1 hour
d. Symmetric tender, swollen joints

The clinical presentation of RA includes extra-articular symptoms, morning stiffness lasting at


least 1 hour, and symmetric, bilateral joint involvement. OA often has a history of previous
injury and is usually asymmetric and may be unilateral.

Chapter 198: Seronegative Spondyloarthropathies


1. A patient is diagnosed with ankylosing spondylitis and begins taking a COX-2 inhibitor with
minimal pain and inflammation relief. What will the provider order initially to manage this
patient’s symptoms?
c. Changing to a COX-1 inhibitor medication

NSAIDs have been shown to reduce pain and stiffness and reduce progression of structural
damage if administered continuously. Patients should try at least two NSAIDs before other
medications are attempted. Sulfasalazine and methotrexate have not been shown to be
significantly effective for axial disease. Biologic anti-tumor necrosis factor medications are
given only after failure of two NSAIDs. Corticosteroid injections are not indicated.

2. A patient is treated for a urinary tract infection and, 3 weeks later, presents with pain and
swelling of one knee and in one hand, along with inflammation in both eyes. What will the
provider suspect as the cause of these symptoms?
d. Reactive arthritis

Reactive arthritis can cause arthritis, urethritis, and inflammation of the eyes 1 to 6 weeks
after a prior infection. Ankylosing spondylitis generally presents with lower back
inflammation. Psoriatic arthritis is associated with psoriasis. Reactive arthritis is not related to
infection in the involved joints.

MULTIPLE RESPONSE
1. A patient reports a history of recurrent lower back pain for 6 months. The patient describes the
pain as a deep ache and stiffness that is worse upon awakening and improves after walking.
Which findings will the examiner elicit to help make a clinical diagnosis of ankylosing
spondylitis? (Select all that apply.)
a. Assessment of the degree of lumbar lordosis
b. Evaluation of lateral thoracic spine flexion
c. Measurement of chest expansion

Examination of the spine will show loss of the normal lumbar lordosis, decreased thoracic
spine flexion, and diminished chest expansion. Cervical kyphosis is not assessed. Scapular
asymmetry evaluates for scoliosis.

Chapter 199: Systemic Lupus Erythematosus


Buttaro: Primary Care: A Collaborative Practice, 6th Edition
MULTIPLE CHOICE
1. A patient with systemic lupus erythematosus (SLE) develops end-stage renal disease. Because
of the underlying SLE, what treatment is recommended for this patient?
a. Dialysis only
b. Immunosuppressant therapy
c. Kidney transplantation
d. Palliative care
ANS: C
Patients with SLE who develop renal failure may require dialysis and then kidney
transplantation; most who undergo transplant do relatively well because of the
immunosuppression given to prevent graft rejection. Immunosuppressant therapy is given for
graft rejection and does not treat end-stage renal failure. Palliative care is not the only option
for this patient.

2. Which laboratory tests may help distinguish systemic lupus erythematosus (SLE) from other
systemic rheumatologic disorders?
a. Antinuclear antibody titer
b. C-reactive protein
c. Rheumatoid factor
d. Serum complement levels
ANS: D
With SLE, complement levels may decrease because of the activation and deposition of
immune complexes in tissues. The other tests are non-specific tests for inflammation and
rheumatologic disorders.

3. A patient with systemic lupus erythematosus (SLE) has frequent symptoms and has been
taking prednisone for each episode. The provider plans to start hydroxychloroquine and the
patient asks why this medication is necessary. What will the provider tell this patient about
this medication?
a. It is effective in reducing disease flares and for tapering steroids.
b. It is given in conjunction with steroids to improve outcomes.
c. It lowers blood pressure and decreases the risk for renal disease.
d. It prevents the need for bisphosphonate therapy.
ANS: A
Hydroxychloroquine is effective in managing musculoskeletal, cutaneous, and serosal
manifestations of SLE and allows tapering of steroids and reduces disease flares.
Cyclophosphamide is given with prednisone to improve renal outcomes. Hydroxychloroquine
is not given for effects on blood pressure and kidneys. Calcium and vitamin D are given to
prevent the need for bisphosphonates.

You might also like